72
Micii MATEMATICIENI 1 De ce să învăŃăm la matematică? Prof. Dr. BOGDAN CRISTESCU În ziua de azi, în discuŃiile sau discursurile legate de studiul matematicii în şcoală, apar în principal două „tipuri” de opinii: pe de o parte discursul de tipul „competenŃa matematică este una din cele opt competenŃe-cheie definite la nivel european, aşa că nevoia de studiu al conŃinuturilor matematice este evidentă şi indiscutabilă”,iar pe de altă parte discursul de tipul „deja nu se mai poate vorbi doar de matematică şi ştiinŃe, sunt opt competenŃe-cheie la nivel european şi ele nu pot fi obŃinute dacă se pune accent pe aceeaşi disciplină, pe acelaşi tip de predare bazată pe conŃinut, pe ceea ce elevul trebuie să ştie sau, mai rău, să memoreze”. În mod clar, ambele „tabere” au partea lor de dreptate şi poate că în raŃionamentul fiecăreia se pot identifica mai greu „greşeli”. Dar mulŃi dintre noi ştim, e drept de la fizică, faptul că studiul evoluŃiei unui proces depinde de sistemul de referinŃă, adică de un sistem de coordonate, de axe ortogonale sau nu. Ceea ce înseamnă…matematică. Reintrând într-un registru mai serios, ne putem gândi la acea parabolă care prezintă descrierile pe care doi oameni legaŃi la ochi le fac unui elefant (nu văzuseră până atunci un asemenea animal), unul atingând piciorul şi unul atingând urechea. Aşa cum era de aşteptat, fiecare dintre ei a făcut o descriere extrem de diferită şi amănunŃită, ambii fiind extrem de convinşi de dreptatea lor pentru că se bazau pe acurateŃea cu care făcuseră observaŃiile. Extrapolând, putem afirma că studiul matematicii în şcoală are foarte multe faŃete. Şi trebuie să nu fim legaŃi la ochi şi să le vedem pe cât mai multe. Partizan fiind, cred că studiul matematicii în şcoală este esenŃial. Dar, atunci când mă refer la studiul matematicii, mă refer atât la frumuseŃea unor probleme, a unor idei accesibile în primul rând celor pasionaŃi şi celor care aprofundează prin muncă acest studiu, dar mai ales, şi aici mă refer la majoritatea elevilor, este vorba de studiul care nu pleacă de la o problemă formulată în termeni matematici. Este vorba de studiul unei situaŃii reale (chiar dacă matematizată), la evaluarea corectă şi completă a unei situaŃii, la extragerea unei idei de urmărit sau a unui rezultat care trebuie obŃinut, la argumentarea prin exemple şi contraexemple a alegerii ideii şi la rigoarea raŃionamentului făcut. Uităm adesea faptul că studiul matematicii trebuie să pună accentul pe proces în aceeaşi măsură în care se pune accentul pe finalitate. În viaŃa de zi de zi apar judecăŃi de valoare, se face apel la o societate civilă bine organizată dar câŃi dintre noi ne-am gândit la faptul că „respectul pentru părerea celuilalt” poate începe prin ascultarea unei soluŃii alternative la o problemă sau prin alegerea unei metode de rezolvare adecvate după o discuŃie argumentată avută cu colegii sau …. exemplele pot continua. Matematica se studiază pentru întregirea sufletului. Şi aici prin suflet înŃeleg ceea ce ne face oameni, fărâma de divinitate, ceea ce este just şi perfect în noi. Vă doresc putere şi ardoare în studiul matematicii ! Inspector General adjunct al Inspectoratului Şcolar JudeŃean, Iaşi

De ce să învăŃăm la matematică? Word - 2011 Revista Mici matematicieni...coordonate, de axe ortogonale sau nu. Ceea ce înseamnă…matematică. Reintrând într-un registru

  • Upload
    others

  • View
    3

  • Download
    0

Embed Size (px)

Citation preview

Page 1: De ce să învăŃăm la matematică? Word - 2011 Revista Mici matematicieni...coordonate, de axe ortogonale sau nu. Ceea ce înseamnă…matematică. Reintrând într-un registru

Micii MATEMATICIENI

1

De ce să învăŃăm la matematică?

Prof. Dr. BOGDAN CRISTESCU

În ziua de azi, în discuŃiile sau discursurile legate de studiul matematicii în şcoală, apar în principal două „tipuri” de opinii: pe de o parte discursul de tipul „competenŃa matematică este una din cele opt competenŃe-cheie definite la nivel european, aşa că nevoia de studiu al conŃinuturilor matematice este evidentă şi indiscutabilă”,iar pe de altă parte discursul de tipul „deja nu se mai poate vorbi doar de matematică şi ştiinŃe, sunt opt competenŃe-cheie la nivel european şi ele nu pot fi obŃinute dacă se pune accent pe aceeaşi disciplină, pe acelaşi tip de predare bazată pe conŃinut, pe ceea ce elevul trebuie să ştie sau, mai rău, să memoreze”.

În mod clar, ambele „tabere” au partea lor de dreptate şi poate că în raŃionamentul fiecăreia se pot identifica mai greu „greşeli”. Dar mulŃi dintre noi ştim, e drept de la fizică, faptul că studiul evoluŃiei unui proces depinde de sistemul de referinŃă, adică de un sistem de coordonate, de axe ortogonale sau nu. Ceea ce înseamnă…matematică. Reintrând într-un registru mai serios, ne putem gândi la acea parabolă care prezintă descrierile pe care doi oameni legaŃi la ochi le fac unui elefant (nu văzuseră până atunci un asemenea animal), unul atingând piciorul şi unul atingând urechea. Aşa cum era de aşteptat, fiecare dintre ei a făcut o descriere extrem de diferită şi amănunŃită, ambii fiind extrem de convinşi de dreptatea lor pentru că se bazau pe acurateŃea cu care făcuseră observaŃiile. Extrapolând, putem afirma că studiul matematicii în şcoală are foarte multe faŃete. Şi trebuie să nu fim legaŃi la ochi şi să le vedem pe cât mai multe. Partizan fiind, cred că studiul matematicii în şcoală este esenŃial. Dar, atunci când mă refer la studiul matematicii, mă refer atât la frumuseŃea unor probleme, a unor idei accesibile în primul rând celor pasionaŃi şi celor care aprofundează prin muncă acest studiu, dar mai ales, şi aici mă refer la majoritatea elevilor, este vorba de studiul care nu pleacă de la o problemă formulată în termeni matematici. Este vorba de studiul unei situaŃii reale (chiar dacă matematizată), la evaluarea corectă şi completă a unei situaŃii, la extragerea unei idei de urmărit sau a unui rezultat care trebuie obŃinut, la argumentarea prin exemple şi contraexemple a alegerii ideii şi la rigoarea raŃionamentului făcut.

Uităm adesea faptul că studiul matematicii trebuie să pună accentul pe proces în aceeaşi măsură în care se pune accentul pe finalitate. În viaŃa de zi de zi apar judecăŃi de valoare, se face apel la o societate civilă bine organizată dar câŃi dintre noi ne-am gândit la faptul că „respectul pentru părerea celuilalt” poate începe prin ascultarea unei soluŃii alternative la o problemă sau prin alegerea unei metode de rezolvare adecvate după o discuŃie argumentată avută cu colegii sau …. exemplele pot continua.

Matematica se studiază pentru întregirea sufletului. Şi aici prin suflet înŃeleg ceea ce ne face oameni, fărâma de divinitate, ceea ce este just şi perfect în noi.

Vă doresc putere şi ardoare în studiul matematicii !

Inspector General adjunct al Inspectoratului Şcolar JudeŃean,

Iaşi

Page 2: De ce să învăŃăm la matematică? Word - 2011 Revista Mici matematicieni...coordonate, de axe ortogonale sau nu. Ceea ce înseamnă…matematică. Reintrând într-un registru

Micii MATEMATICIENI

2

Matematica, un “esperanto” al celor riguroşi şi performanŃi, un argument pentru o carieră de succes !

Prof. Univ. Dr.ing. ALEXANDRU SǍLCEANU DorinŃa de a impune o comunicare facilǎ în cadrul uriaşei diversităŃi numită “omenire” a

preocupat vizionarii fiecărei generaŃii. Ce semnifică din punct de vedere lingvistic esperanto (limbaj artificial extrem de simplificat propus la sfârşitul secolului XIX) îşi găseşte un mult mai natural echivalent în limbajul matematic.

O idee corectă poate fi uşor transpusă în limbaj matematic. Fără modelare şi simulare matematică, ingeniozitatea indispensabilă profesiilor inginereşti ar rămâne la nivelul unui empirism limitat, greu de argumentat sau de explicat.

Demersul pe care tinerii matematicieni de la Liceul “Ştefan cel Mare” din Hîrlău îl întreprind cu pasiune şi entuziasm, beneficiind de coordonarea unui nucleu profesoral de cea mai bună calitate, va rodi cu siguranŃă în momentele importante care jalonează devenirea profesională şi inserŃia socială.

Un tânăr care a deprins o abordare logică şi coerentă a problemelor cărora trebuie să le găsească potrivit răspuns, are şanse sporite să atingă excelenŃa în profesia aleasă, fie ea medicală, socio-umană, didactică sau, în primul rând, inginerească.

Ingineria electrică şi electronică, telecomunicaŃiile şi tehnologiile informatice implică un grad de abstractizare sporit prin comparaŃie cu alte domenii ale tehnicii. Tainele acestor profesii extrem de solicitate pe actuala piaŃă a muncii sunt mult mai accesibile celor pentru care gândirea şi logica matematică au devenit o normalitate încă de pe băncile primilor ani de şcoală.

Este omenesc sǎ te întrebi, atunci când depui un efort îndelung susŃinut, dacă vei fi răsplătit, dacă vei culege nişte roade, sub o formă sau alta, într-un viitor mai apropiat sau mai depărtat.

În cazul micilor matematicieni din Hârlău, unul dintre posibilele foarte bune răspunsuri la această întrebare îl poate constitui admiterea de succes la o facultate centenară prin tradiŃie, dar pe deplin ancorată în realitatea economicǎ a începutului de mileniu III. E vorba de Facultatea de Inginerie Electrică, Energetică şi Informatică Aplicată, Universitatea Tehnică „Gheorghe Asachi” din Iaşi. Aici aveŃi posibilitatea opŃiunii pentru unul din patru domenii, toate extrem de căutate în Europa, (inclusiv România) sau America de Nord. Aici vă oferim condiŃii excelente (cazare în cămine modern dotate, burse de merit, de studii sau sociale, tabere naŃionale şi internaŃionale, stagii de pregătire în cele mai prestigioase universităŃi din Europa), pentru a pătrunde tainele curentului electric. Ingineria în general, cea electrică în mod special, este o profesie în care excelenŃa este accesibilă doar celor cu o pregătire matematică solidă.

Apreciez în mod deosebit munca pe care o depuneŃi în cadrul unui liceu de tradiŃie şi vă aşteptăm la Facultatea de Inginerie Electrică, Energetică şi Informatică Aplicată, în calitate de oaspeŃi dragi, aflaŃi într-o vizită care se va dovedi rodnică din punctul de vedere al unei investiŃii în viitorul vostru socio-profesional !

Decan al FacultăŃii de Inginerie Electrică, Energetică şi Informatică Aplicată,

Universitatea Tehnică „Gheorghe Asachi”, Iaşi

Page 3: De ce să învăŃăm la matematică? Word - 2011 Revista Mici matematicieni...coordonate, de axe ortogonale sau nu. Ceea ce înseamnă…matematică. Reintrând într-un registru

Micii MATEMATICIENI

3

ARTICOLE ŞI NOTE MATEMATICE

GEOMETRIE CONCRETĂ (1)

DAN BRÂNZEI ALEXANDRU NEGRESCU

Această notă are cauze, scopuri, sugestii, planuri, secrete (de serviciu), explicaŃii (succinte); le vom contura succesiv, adresându-le cui trebuie: unor semi-adolescenŃi interesaŃi. Se deduce că accesul persoanelor majore la aceste pagini este contraindicat. RestricŃia vizează binele adulŃilor: tot nu ar pricepe mare lucru! ContravenienŃii vor fi pedepsiŃi: li se va interzice o săptămână să iasă la joacă! • 1. Cauze. Societatea de ŞtiinŃe Matematice din România a declarat 2010 – Anul matematicii în şcoală. Filiala Ieşeană a acestei societăŃi îmbrăŃişează cu entuziasm această iniŃiativă. Autorii au convingerea că excesele de abstractizare în şcoală au efecte negative (momentane şi de perspectivă). ExperienŃe recente au confirmat astfel de impresii. Invers, o anume zăbavă pe „un concret atractiv” aduce în timp beneficii. Autorii cunosc generozitatea acŃiunilor locale împletite sub sigla „Micii Matematicieni” şi le apreciază. •••• 2. Scopuri. O anume colaborare susŃinută între persoane şi instituŃii poate genera un „Atelier de Geometrie Concretă” funcŃionând în Liceul Teoretic Ştefan cel Mare Hîrlău. Atelierul ar avea un comandament alcătuit din profesori, informaticieni, învăŃători, poate şi educatori, un grup de iniŃiative alcătuit din elevi „mai copŃi”, consilieri onorifici (autorii) şi un detaşament de „inimoşi”. Atelierul ar coordona „jocuri” de realizare (prin desenare, îndoire, decupare, lipire) a unor obiecte geometrice cu calităŃi predominant estetice: etichete, plăci de hârtie, celuloid, carton sau placaj, cutii, coşuleŃe, insigne, ecusoane, seturi de piese mobile etc. Noi autorii gândim că până la momentul festiv al desfăşurării concursului Micii Matematicieni 2011 bună parte dintre aceste deziderate să poată fi expuse. Dacă va fi să nu fie aşa, ne-om aminti şi noi zicala: Mai bine mai târziu decât niciodată. • 3. Sugestii. Comandamentul ar detalia oportunităŃi organizatorice şi ar realiza la imprimantă (pe diverse tipuri de carton şi hârtie colantă) pentagoane şi decagoane regulate şi stelate pregătind ecusoane de membri (diferenŃiate pentru comandament şi „instructori”). (Lămuriri pentru inimoşi: formele desenate au denumiri oficiale: pentagonul regulat, pentagonul stelat, decagonul regulat, decagonul stelat. Sunt însă prietenii voştri şi nu se supără dacă le spuneŃi de exemplu: Platul, SlăbuŃul, Grasul, ColŃosul. Poate cândva veŃi învăŃa şi cum le puteŃi desena geometric). Poate că o abreviereAGECON – HIRLAU s-ar dovedi mobilizatoare. De la instructori ar merge spre inimoşi forme regulate de carton (variat dimensionate) pentru a fi returnate desenate, colorate, eventual „lipite în relief”. Autorii se îngrijesc ca în afara acestor figuri mici din text să ajungă la informaticieni figuri mari, precise. • 4. Planuri. Credem că de la şefi la realizatori vor porni (la dimensiuni adecvate) „flori” ca cea din figura 5. (Noi zicem că această floare Penta este foarte frumoasă; după câte ştim, nuexistă în realitate, dar nu este nevoie să o căutaŃi, fiindcă o veŃi avea pe masă). Inimoşii vor învăŃa cum să decupeze atent. (Nu vă grăbiŃi să tăiaŃi prea mult; unele margini vor fi întoarse şi lipite). Apoi, vor îndoi, vor lipi şi vor avea un coşuleŃ elegant. Noi i-am spune unui astfel de coşuleŃ KosoFlo, deoarece este exotic, este făcut dintr-o floare şi pare a fi dornic să mai găzduiască flori. Zicem că de la comandament vor pleca spre câte un inimos amator (sau inimoasă amatoare) câte două flori Penta. (Aşa este pe la unii uzanŃa: florile să fie oferite în număr par).

Page 4: De ce să învăŃăm la matematică? Word - 2011 Revista Mici matematicieni...coordonate, de axe ortogonale sau nu. Ceea ce înseamnă…matematică. Reintrând într-un registru

Micii MATEMATICIENI

4

Vor fi confecŃionate câte două coşuleŃe KosoFlo şi ne permitem să anticipăm surpriza plăcută: răsturnând un coşuleŃ, drept capac pentru celălalt, ele se vor potrivi „miraculos de perfect”; „corpul” rezultat are o denumire oficială haină: duodecaedru regulat. Dar voi nu vă speriaŃi de cuvinte lungi şi veŃi îndrăgi obiectul; poate îi veŃi spune Dudu sau Dodo (cum era numită o pasăre – vag asemănătoare - care a cam fost mâncată definitiv de marinari). Unii vor spune că seamănă cu mingea de la campionatul mondial de fotbal, alŃii cor spune că este un bostan foarte potrivit pentru Haloween, poate altele vor spune că este un bujor perfect înflorit. Poate un comandant mărinimos vă va spune poveşti neasemuite (dar adevărate) despre „cele 5 solide platonice” şi vă va mai sugera distracŃii. Sperăm că cititorul nu se declară plictisit de referirile noastre la Penta, KosoFlo, Dudu şi continuăm. Gândim acum un balon Dudu. (Poate din cauciuc mai subŃire şi mai elastic decât cel în care dădeau unii cu piciorul). Bine dezumflat, un astfel de balon se aranjează ca floarea Diamanta (sau Dodica) care apare în figura 6. Ar fi oare comandamentul AGeCon atât de câinos încât să refuze dulci RugăminŃi inimoase: mi-ar place să colorez câteva Diamante? •••• 5. Alte planuri. Avertisment pentru eventuali inimoşi oscilanŃi: Dacă ai intrat în horă trebuie să joci! Nu apreciază nimeni pe unul care se instalează în mijlocul horei ca un prepeleag Ńeapăn! Cei care intră în AGeCon trebuie să se străduie zi de zi, ceas de ceas şi proporŃie de masă să devină mai hâtri. (Cam aşa se exprima un fost locatar al unui mausoleu, fie-i lespedea uşoară). Asta înseamnă să pregătiŃi şi ceva jocuri prin care să testaŃi isteŃimi (ale prietenilor sau pizmaşilor). Zăbovim un pic să vă spunem ceva despre hexagonul regulat (pe care îl putem dezmierda Hexi). Arată ca în figura 7. PuteŃi obŃine unul stricând un ceas vechi (rotund) şi tăind după linii care unesc 1 cu 3, 3 cu 5, 5 cu 7, 7 cu 9, 9 cu 11, 11 cu 1. Dacă părinŃii nu se arată înŃelegători (cum se mai întâmplă şi vă ameninŃă cu corecŃii nepedagogice dacă îndrăzniŃi să stricaŃi ceasul), o să puteŃi desena unul numai cu compasul. (După ce aŃi împuns în primul centru, aveŃi mereu grijă să înfingeŃi vârful numai pe „circumferinŃă”). Dacă aveŃi grijă să nu-i modificaŃi deschiderea iniŃială, veŃi obŃine o „margaretă” ca în figura 8; o să o dezmierdăm Hexina; puteŃi să o vopsiŃi după plac, fără să solicitaŃi sprijin de la comandament. (Asta numai dacă sunteŃi dintre aceia cărora le plac Hexinele vopsite). Zicem împreună că acum ştiŃi cine este Hexi; dacă doriŃi amănunte despre el, solicitaŃi comandamentul. (Nu vă încurajăm să întrebaŃi dacă Hexi este blond sau brunet). Acum, inimoşilor, veŃi căpăta de la şefi un pătrat ca în figura 9 cu linii rasateîn interior. O să fim mai precişi: şefii vă vor da o hârtie mai mare, convenabil trasată. Voi o veŃi lipi frumos pe un carton foarte gros sau pe placaj, sau pe o bucată rigidă de linoleum. VeŃi tăia apoi după liniile subŃiri cu un cutter sau traforaj. (Se recomandă ferm să nu tăiaŃi şi degete; ale voastre sau ale altuia care vă ajută la imobilizare). Setului de 5 piese astfel obŃinut îi spunem între noi PatraHex. PregătiŃi din timp (tot cu imagine de la şefi) şi golul hexagonal din figura 10. Apoi, când aveŃi un voluntar la dispoziŃie, îl îndemnaŃi să reaşeze setul PatraHex în golul hexagonal. Stop! Unul dintre autori se scuză! Celălalt a gafat şi a introdus în text o figură 10 neautorizată, pe care se poate vedea şi o distribuŃie posibilă a setului PatraHex. (Nu cunoaştem explicaŃia fizică a fenomenului, dar am observat de foarte multe ori: gafele sunt aproape totdeauna făcute de celălalt). Voi trebuie să fiŃi vigilenŃi şi să nu-i acordaŃi voluntarului posibilitatea de acces la o astfel de informaŃie! Prima măsură, strict necesară, este să ascundeŃi acest număr din revista Micii

Page 5: De ce să învăŃăm la matematică? Word - 2011 Revista Mici matematicieni...coordonate, de axe ortogonale sau nu. Ceea ce înseamnă…matematică. Reintrând într-un registru

Micii MATEMATICIENI

5

Matematicieni, să nu fie văzut de eventualii voluntari: sub un pat, după un dulap, sub covoruldin dormitor, unde ştiŃi voi că nu mai poate fi găsit de nimeni. În continuare, ne abŃinem să vă spunem ce este aceea triunghi echilateral, dezmierdat Trichi. Dacă bănuiŃi un înŃeles, aveŃi şanse mari să nimeriŃi. Dacă nu, întrebaŃi pe cine trebuie. Acum veŃi căpăta un şablon pentru un pătrat ca în figura 11, pe care sesizaŃi liniile subŃiri după care veŃi tăia setul de plăci PatraTri. Voluntarul testat (acelaşi sau altul), va reaşeza acest set în golul triunghiular din figura 12. Pentru acest nou test trebuie să fiŃi fairplay şi să anunŃaŃi că este permisă răsturnarea unor piese din setul PatraTri. Din nou scuze! Celălalt a gafat din nou, punând în text o figură 12 cu deconspirări de secrete! Este recalcitrant şi recidivist! Devine şi mai necesar să ascundeŃi bine (şi definitiv) numărul de revistă! •••• 6. Secrete. Am evidenŃiat deja unele secrete, pe care le-am numi personale: unde anume pitiŃi voi numărul acesta de revistă! Să nu spuneŃi că le puneŃi în biblioteca voastră, deoarece acolo nu umblă nimeni! Nu veŃi fi crezuŃi! Gândul autorilor este să mai continue cu astfel de idei ciudate. Deci, dacă găsiŃi un loc bun de pitit reviste, este bine să fie destul de mare să mai încapă şi numere viitoare! Venim astfel pe secretele care le-am numi de serviciu. Unii (comandanŃi, intermediari sau inimoşi) ar putea deveni super-ultra-curioşi şi să pună întrebări nepotrivite: cum anume se construiesc diverse figuri, cum se dovedesc potrivirile. Asta înseamnă că o apucă pe drumul rău a călcării de secrete. Vor avea multişor de învăŃat din geometria (abstractă)! Poate vor fi trecut de toate manualele şcolare în vigoare şi tot nu vor fi aflat ce vor! Să fie cuminŃi! Dacă acea super-ultra-curiozitate îi persecută tare de tot, ar putea găsi ceva într-un volum roşu de CompetenŃă şi PerformanŃă în Geometrie, cam după pagina 76. Dacă îi furnică o curiozitate mai cuminte, să mai afle poveşti despre cele de mai sus, ne vom strădui să ne întâlnim pe undeva pe lângă şcoală şi să povestim. Totuşi! Ca să nu ne certe matematicienii că noi dezvăŃăm copii de a căuta argumentări, înserăm o problemă. (SoluŃiile pot fi trimise lângă cele ale altor probleme din revistă; autorii vor da o soluŃie în numărul viitor). Se dau plăci dreptunghiulare ABCD, KLMN. Se presupune că avem .KL AB< Se ştie că au aceeaşi arie, deci .AB AD KL KN⋅ = ⋅ Se cere să se taie primul dreptunghi cu două tăieturi rectilinii şi să se reasambleze piesele rezultate pentru a forma al doilea dreptunghi. • 7. ExplicaŃii. Autorii sunt obligaŃi să se cam oprească! Mărturisesc (ei) că sunt dependenŃi de o organizaŃie foarte puternică, având ramificaŃii cam pe tot globul şi care se numeşte aproximativ MECET. (Depinde şi de Ńară şi de limba în care se scrie şi se citeşte). Această organizaŃie veghează să nu fie călcate în picioare drepturile sacre ale copiilor. De exemplu, un adult nu este slobod să trăncănească (monoton şi neîntrerupt) în faŃa copiilor mai mult de 30 de minute fără să le acorde pauza necesară să se mai buşească şi între ei! SunteŃi astfel îndemnaŃi să observaŃi că şi adulŃii recunosc unele drepturi inalienabile ale copiilor. Socotind şi viteza medie de lectură a semi-adolescenŃilor autorii au cam ajuns la limita tolerată de lege şi trebuie (imperativ) să se oprească. Altfel, pot fi pedepsiŃi să nu mai meargă la serviciu! • 8. Încheiere. Ştim cu toŃii ce frumoase sunt florile care smălŃuiesc câmpiile şi grădinile; poate oare cineva garanta că minŃile şi mânuŃele de la AGeCon nu vor zămisli altele, încă mai frumoase? La revedere.

Profesor universitar Doctor, Universitatea “Alexandru Ioan Cuza”, Iaşi

Profesor, Şcoala nr. 9 “Elena Cuza”, Iaşi

Page 6: De ce să învăŃăm la matematică? Word - 2011 Revista Mici matematicieni...coordonate, de axe ortogonale sau nu. Ceea ce înseamnă…matematică. Reintrând într-un registru

Micii MATEMATICIENI

6

ÎmpărŃirea prin metoda complementelor

PETRU ASAFTEI

AMALIA ROMILA

În cele ce urmează vom demonstra o teorema ce poate fi utilizatǎ în metodologia predării operaŃiei de împărŃire la clasele III-V. Teoremă: Dacă se adună la deîmpărŃit produsul câtului cu complementul împărŃitorului, suma este numărul care începe cu cifrele câtului, urmate de atâtea zerouri cât este diferenŃa dintre numărul cifrelor împărŃitorului şi numărul cifrelor restului, iar după aceste zerouri se adaugă cifrele restului. Dacă restul este nul, după cifrele câtului se adaugă atâtea zerouri câte cifre are

împărŃitorul ( Complementul lui 1 2 3... na a a a este numărul 1 2 310 ...nna a a a− ).

DemonstraŃie: Pentru numerele naturale d şi b cu 0b ≠ şi b d≤ , din teorema împărŃirii cu

rest obŃinem d b c r= ⋅ + cu 0 r b≤ < . Considerăm 1 2... mb bb b= , 1 2... pc c c c= , 1 2... kr r r r= cu

, ,m p k ∗∈ℕ şi k m≤ . Ar trebui să demonstrăm că: ( ) �1 2 1 210 ... 00...0 ...mp k

m k

d b c c c c r r r−

+ − ⋅ = .

Într-adevǎr, avem ( ) ( ) �1 2 1 210 10 10 ... 00...0 ...m m mp k

m k

d b c b c r b c c r c c c r r r−

+ − ⋅ = ⋅ + + − ⋅ = ⋅ + = .

Dacă 0r = atunci ( ) ( ) �1 210 10 10 ... 00...0m m mp

m

d b c b c b c c c c c+ − ⋅ = ⋅ + − ⋅ = ⋅ = .

ObservaŃie: Dacă b d> , atunci 0c = şi d r= . ***

Regula după care se vor face împărŃirile prin metoda complementelor are la bază următorul raŃionament:

� Fie 1 2... nd a a a= şi 1 2... ma a a sau 1 2 1... ma a a + , cel mai mic număr format cu cifrele

deîmpărŃitului, luate de la stânga spre dreapta, în care împărŃitorul se cuprinde cel puŃin o

dată şi cel mult de 9 ori. Numărul 1 2... ma a a sau 1 2 1... ma a a + se numeşte primul deîmpărŃit

parŃial. � Din teorema precedentă se deduce că prima cifră a câtului este prima cifră a numărului

obŃinut prin efectuarea sumei ( )1 2 1... 10mma a a b c+ − ⋅ sau a sumei ( )1 2 1 1... 10mma a a b c+ + − ⋅ .

Pentru a face împărŃirea a două numere naturale prin metoda complementelor, � se scrie deasupra împărŃitorului complementul său, � apoi se separă la stânga deîmpărŃitului atâtea cifre astfel încât, în numărul format,

împărŃitorul să se cuprindă cel puŃin o dată şi cel mult de 9 ori. � Se împarte acest număr la împărŃitor şi se obŃine astfel prima cifră a câtului. � Se înmulŃeşte această cifră cu complementul împărŃitorului şi rezultatul se adună la

deîmpărŃitul parŃial. � La aceasta sumă, prima cifră se subliniază, iar la dreapta celorlalte se coboară cifra 1ma + sau

2ma + , după caz, formându-se în acest fel al doilea deîmpărŃit parŃial.

De exemplu: ( )1 2 1 1 2... 10 ...mm la a a b c s s s+ − ⋅ = , unde l poate fi m sau 1m + .

În acest caz, 1 1s c= , iar numărul 1 2... ls s s se numeşte primul rest parŃial.

Asupra celui de-al doilea deîmpărŃit parŃial aplicăm din nou algoritmul precedent.

Page 7: De ce să învăŃăm la matematică? Word - 2011 Revista Mici matematicieni...coordonate, de axe ortogonale sau nu. Ceea ce înseamnă…matematică. Reintrând într-un registru

Micii MATEMATICIENI

7

Exemplul 1: Numărul 4353 este primul deîmpărŃit parŃial, iar 2187 este al doilea deîmpărŃit parŃial. Analog, numărul 218 este primul rest parŃial, iar 533 este al doilea rest parŃial, care este chiar restul împărŃirii iniŃiale. Rezultatele sunt corecte deoarece şi Deoarece m – k = 0, între cifrele câtului şi cifrele restului nu avem niciun zero.

Exemplul 2: La această împărŃire restul final este 989, iar m – k =1, ceea ce înseamnă că între cifrele câtului şi cifrele restului se interpune un zero. Intr-adevăr,

Exemplu 3: În acest exemplu restul este 0. Conform rezultatului stabilit în teoremă, după cifrele câtului trebuie să adăugăm cinci zerouri şi astfel

Această alternativă la operaŃia clasică de împărŃire are avantajul că înlocuieşte scăderile succesive prin adunări succesive. Prima cifră de la rezultatul fiecărei adunări indică cifra corespunzătoare a câtului, obŃinându-se astfel un element de control asupra operaŃiei efectuate.

Bibliografie:

1. Abatele E. Gelin, ÎmpărŃirea prin metoda complementelor, “ RecreaŃii Stiintifice”, 1887, nr. 7 şi 8, pag.1-3

Profesori, Şcoala Normalǎ “Vasile Lupu”,

Iaşi

Exemplul 1: 173 43537: 827 = 52 865 52187 346 2533

Exemplul 2: 1675 32568389: 8325 = 3912 5025 375933 15075 910088 1675 117639 3350 20989

Exemplu 3: 21746 7825478254 : 78254 = 100001 21746 1000007 0 0000078 0 0000782 0 0007825 0 0078254 21746 100000

Page 8: De ce să învăŃăm la matematică? Word - 2011 Revista Mici matematicieni...coordonate, de axe ortogonale sau nu. Ceea ce înseamnă…matematică. Reintrând într-un registru

Micii MATEMATICIENI

8

O expresie algebricǎ

generatoare de noi probleme IOAN SǍCǍLEANU

Numerelor , , ,a b x y∈ℝ le asociem expresia ( ) 2a bE x

ax by bx ay x y= + −

+ + +.

Grupǎm convenabil şi efectuǎm calculele: ( ) 1 1a bE x

ax by x y bx ay x y

= − + − + + + +

, de

unde, obŃinem forma echivalentǎ cu ( )( ) ( ) ( ) ( )ax ay ax by bx by bx ay

E xax by x y ax by x y

+ − − + − −= +

+ ⋅ + + ⋅ + , adicǎ

( ) ( )( ) ( )

( )( ) ( )

a b y a b yE x

ax by x y ax by x y

− ⋅ − ⋅= −

+ ⋅ + + ⋅ + ⇔ ( ) ( ) 1 1a b y

E xx y ax by bx ay

− ⋅ = ⋅ − + + +

( ) ( )( ) ( )

a b y bx ay ax byE x

x y ax by bx ay

− ⋅ + − −= ⋅

+ + ⋅ + ⇔ ( ) ( ) ( ) ( )

( ) ( )a b y y a b x a b

E xx y ax by bx ay

− ⋅ − − −= ⋅

+ + ⋅ +. Prin urmare

( ) ( ) ( )( ) ( ) ( )

2y a b y x

E xx y ax by bx ay

⋅ − ⋅ −=

+ ⋅ + ⋅ + în condiŃiile de existenŃǎ necesar impuse.

În concluzie: Avem ( ) 0E x = dacǎ şi numai dacǎ a b= sau 0y = sau x y= .

*** 1. Pentru 2x = şi 3y = se obŃine problema C.R.VI.1 din nr. 2/2009, revista “ Alpha”:

“Dacǎ , 0,a b > atunci 2

2 2 3

a b

a b b a+ =

+ + dacǎ şi numai dacǎ a b= ”

D.M. BǎtineŃu-Giurgiu, prof. Bucureşti

2. Pentru 1x = şi 1

yab

= se obŃine 8.25 din nr. 4/2010, revista “Micii matematicieni”:

Numerele întregi a şi b validează egalitatea: 2 2

2 2

2

1 1 1

a b ab

a b ab+ =

+ + + . DemonstraŃi că

produsul numerelor a şi b este un pătrat perfect. Ioan Sǎcǎleanu, prof. Hirlău

3. Pentru lnx a= şi lny b= se obŃine problema 10.26 din nr. 4/2010, revista “Micii …”:

CalculaŃi 2010 2010a b− ştiind cǎ( ) ( ) ( )2

1 1 1

loglog loga b

a b b aee ea ba b a b

+ =⋅⋅ ⋅

; , 0a b > , 1b ≠ .

Ioan Sǎcǎleanu, prof. Hirlău 4. Pentru 2010

ax C= şi 2010by C= se obŃine problema 10.25 din nr. 4/2010, “Micii …”:

“ DeterminaŃi numerele naturale a şi b având produsul 21005 ştiind cǎ verificǎ

2010 2010 2010 2010 2010 2010

2a b a b a b

a b

a C b C b C a C C C+ =

⋅ + ⋅ ⋅ + ⋅ + .”

Aurel Neicu, prof. Hirlău 5. Pentru a f= , /x f= ,b F= şi / /y f= se obŃine 12.20 din nr. 4/2010, “Micii …”:

“Sǎ se determine funcŃia de douǎ ori derivabilǎ ( ): 0,f → ∞ℝ ştiind cǎ verificǎ

/ / / / / / / / /

2f F

f f F f f f F f f f+ =

⋅ + ⋅ ⋅ + ⋅ + , unde F este o primitivǎ a funcŃiei f .”

Ioan Sǎcǎleanu, prof. Hirlău

Page 9: De ce să învăŃăm la matematică? Word - 2011 Revista Mici matematicieni...coordonate, de axe ortogonale sau nu. Ceea ce înseamnă…matematică. Reintrând într-un registru

Micii MATEMATICIENI

9

Matematica aplicată

în rezolvarea unor probleme de fizică

ANCA-MARIA BOBÎRNǍ La nivel liceal există numeroase probleme de fizică în a căror rezolvare sunt necesare serioase cunoştinŃe de matematică. Dintre principalele noŃiuni de matematică utilizate în fizica amintim: calcul vectorial, noŃiuni de trigonometrie, calcul fazorial, derivate, integrale, numere complexe, etc. Din păcate profesorul de fizică este pus în faŃa unor dificultăŃi cauzate de lipsa corelării materiilor la matematică şi fizică. În aceastǎ notǎ vom aborda, folosind diverse tehnici matematice douǎ probleme tipice de extrem, des întâlnite la concursurile de fizică şi chiar în variantele de bacalaureat. Problema 1: La bornele unui generator cu t.e.m. E şi rezistenŃa r se cuplează un rezistor cu rezistenŃa variabilă. Care este valoarea rezistenŃei acestui rezistor pentru care puterea debitată pe circuitul exterior să fie maximă? Rezolvare 1:Din expresia puterii disipate (P) pe circuitul exterior a unui generator de tensiune, în funcŃie de rezistenŃa electrică (R) şi intensitatea curentului electric (I), avem:

2P R I= ⋅ . Conform legii lui Ohm: E

IR r

=+

obŃinem ( )

2

2

EP R

R r= ⋅

+ de unde gǎsim cǎ

2

2

EP R

rR R

R

= ⋅ +

=2

2

+

R

rR

E. Se observă că numitorul este o sumă de doi termeni

a căror produs este constant, deci au sumă minimă când termenii sunt egali, adicǎ r

RR

= .

Prin urmare valoarea rezistenŃei acestui rezistor este rR = . Rezolvare 2: Din expresia puterii disipate (P) pe circuitul exterior a unui generator de tensiune, în funcŃie de tensiunea electrică (U) şi intensitatea curentului electric (I), avem: P U I= ⋅ . Dar, tensiunea electricǎ este U E r I= − ⋅ . Deci ( ) 2P E rI I r I E I= − ⋅ = − ⋅ + ⋅ , care este o funcŃie de

gradul II cu variabila I . ObŃinem cǎ P este o funcŃie de gradul al II-lea cu : 0a r= − < şi

:b E= , care admite punct de maxim în max 2 2

b EI

a r= − =

⋅. Cum

rR

EI

+= atunci

2

E E

R r r=

+ ⋅2 E r E R E r⇔ ⋅ ⋅ = ⋅ + ⋅ , de unde rR = .

Rezolvare 3: Avem( )

22

2

EP R I R

R r= ⋅ = ⋅

+. Se observǎ cǎ P este o funcŃie în R . Derivata

acestei funcŃii este: ( )( ) ( )

( )( ) ( )

( ) ( )/

2 2 22 2 2

2 2 2

2R

R r R r R R r r R r RP E E E

R r R r R r

+ − ⋅ + ⋅ + − −= ⋅ = ⋅ = ⋅

+ + +. Cum

( )/ 0P R R r= ⇔ = ± atunci din tabelul de variaŃie

rezultǎ cǎ valoarea maximǎ a rezistenŃei este rR = . Problema 2: Se consideră o lentilă biconvexă a cărei distanŃă focală este f. Să se afle distanŃa minimă dintre obiect şi imaginea sa reală.

R

( )/RP

( )RP

r

0

max

−−−−−−++++++

Page 10: De ce să învăŃăm la matematică? Word - 2011 Revista Mici matematicieni...coordonate, de axe ortogonale sau nu. Ceea ce înseamnă…matematică. Reintrând într-un registru

Micii MATEMATICIENI

10

Rezolvare 1: Notǎm cu x1 distanŃa dintre obiect şi lentilă, iar cu x2 distanŃa dintre lentilă şi imagine. Atunci distanŃa dintre obiect şi imaginea reală a sa prin lentilă este 21 xxd +−= .

Din reprezentarea graficǎ, obŃinem ( )fax +−=1 ;

fbx +=2 ( unde f este distanŃa focală a lentilei ). Gǎsim fbad 2++= . Pentru ca d sǎ fie minimǎ trebuie ca a b+ sǎ fie minimǎ. Se realizeazǎ atunci când produsul a b⋅ este constant şi mai mult, trebuie ca a b= . Din formula lentilelor subŃiri avem:

fxx

111

12

=− ⇒ffafb

111=

++

+. Aducând la

acelaşi numitor obŃinem succesiv egalitǎŃile:

( ) ( ) ( ) ( )a f f b f f a f b f+ ⋅ + + ⋅ = + ⋅ + , de unde 2 2 2af f bf f ab af bf f+ + + = + + + . Prin urmare 2 .a b f const⋅ = = ⇒ a b f= = . Atunci distanŃa

minimǎ cǎutatǎ este fd 4min = .

Rezolvare 2: Înlocuind pe dxx −= 21 în fxx

111

12

=− , obŃinem cǎ 0222 =+− fddxx . Prin

urmare ecuaŃia 2 0X d X f d− ⋅ + ⋅ = are pe 2x ca soluŃie şi deci discriminantul ei este în mod

necesar 0∆ ≥ .Dar 2 24 4 0 4d fd d fd d f∆ = − ⇒ − ≥ ⇒ ≥ . Minimul distanŃei d este

fd 4min = .

Rezolvare 3: Din fxx

111

12

=− rezultǎ cǎ 1

2 1

1 f x

x f x

+=

⋅, de unde 1

21

x fx

x f

⋅=

+. Înlocuind în

1 2d x x= − + obŃinem 11

1

x fd x

x f= − +

+⇔

21

1

xd

x f= −

+. Prin urmare distanŃa d este o funcŃie în

necunoscuta 1x având derivata:

( ) ( )( ) ( )

2 21 1 1' 1 1

1 2 2

1 1

2 2x x f x x x fd x

x f x f

− + + − − ⋅ ⋅= =

+ +. Din

tabelul de variaŃie al funcŃiei d deducem fd 4min = . Bibliografie:

1. R. Sfichi, Probleme de limită ş extrem, E.D.P. 1979 2. M. Atanasiu, V. Drobotă, Fizica pentru admitere în facultate, Editura Albatros, 1974 3. S. Talpalaru, D. Haralamb, C. Corega, Manual de fizică, cl. a Xa, Ed. Polirom, 2007

Profesor de fizicǎ,

Liceul teoretic “Ştefan cel Mare”, Hîrlǎu

00

2 f−

−−−+ + + +−−−

maxmin

( )/1d x

( )1d x 4 f 0

1x 0

f ba

2x1x−

2F

f1F

/B

/A

A

B

d

Page 11: De ce să învăŃăm la matematică? Word - 2011 Revista Mici matematicieni...coordonate, de axe ortogonale sau nu. Ceea ce înseamnă…matematică. Reintrând într-un registru

Micii MATEMATICIENI

11

VIAłA MATEMATICĂ ZONALĂ

Această rubrică conŃine în acest număr informaŃii despre: • concursul “Micii MATEMATICIENI” , ediŃia a V a din 20 martie 2010 ; • subiecte date la “ Testarea elevilor “ de clasa a IV a în vederea înscrierii în clasa a V a ; • proiectul educaŃional “SUPER MATE” în anul şcolar 2009-2010; • raport de activitate a cercului de matematicǎ “Clubul matematicienilor”.

Concursul “Micii matematicieni” EdiŃia a V-a, 20 martie 2010

În ziua de 20 martie 2010, s-a desfǎşurat la Hîrlǎu a V a ediŃie a concursului “Micii matematicieni” organizat de Liceul Teoretic “Ştefan cel Mare”, Hîrlău în parteneriat cu Inspectoratul Şcolar JudeŃean, Iaşi şi AsociaŃia “RecreaŃii matematice”, Iaşi . Acest concurs s-a nǎscut din dorinŃa de a veni în sprijinul acelor elevi care manifestǎ aptitudini pentru matematicǎ, disponibilitate pentru muncǎ şi dorinŃǎ pentru a se înscrie într-o competiŃie deschisǎ ce vizeazǎ performanŃa. Ne face o deosebită plăcere să menŃionǎm prezenŃa unor invitaŃi importanŃi din judeŃ, îndrumători activi pentru multe generaŃii de elevi şi studenŃi:

� Prof. Dr. Dan Brânzei, Universitatea “Alexandru Ioan Cuza”, Iaşi � Prof. Dr. Temistocle Bârsan, Universitatea Tehnicǎ “Gh. Asachi”, Iaşi � Prof. Dr.Rodica Luca Tudorache, Universitatea Tehnicǎ “Gh. Asachi”, Iaşi � Prof. Mihai Haivas, Institutul de Cercetări Economice „Gheorghe Zane” Iaşi

Organizatorii cu sprijinul sponsorilor au oferit premii în bani şi diplome tuturor câştigǎtorilor concursului. Este o datorie de onoare sǎ mulŃumesc pe aceastǎ cale sponsorilor celei de a V-a ediŃii a concursului „Micii matematicieni” :

� ASOCIAłIA PĂRINłILOR „ŞTEFAN CEL MARE”, HÂRLĂU; � PRIMĂRIA ORAŞULUI HÎRLĂU; � S.C. COTNARI S.A. ; � C.M.I. Dr. STELA NEICU; � S.C. BEST COLOR S.R.L. � RESTAURANT łĂPUŞĂ

şi contăm în continuare de sprijinul lor. Prin participarea a 329 elevi din cel puŃin 28 de unităŃi şcolare din judeŃ se poate spune că am încheiat cu un real succes a V-a ediŃie a concursului. Aşteptăm cu interes profesorii şi învăŃătorii care doresc să se implice în buna organizare a ediŃiei a VI a din 26 martie 2011 şi să ne contacteze . Vă mulŃumim anticipat pentru participare . Prezentǎm în continuare lista premianŃilor şi subiectele propuse spre rezolvare.

Rezultatele concursului „Micii matematicieni”, ediŃia a V-a, Hîrlău, 20 martie 2010

Cls Nume si prenume Şcoala de provenienŃǎ Profesorul

îndrumǎtor Puncte

Pre-miul

III TENCHIU TEODOSIA Şc.”P.Rareş”, Hîrlǎu BUDACEA MARIA 50 I III IOSUB MARIAN Şc.”P.Rareş”, Hîrlǎu BUDACEA MARIA 48 I III DELEANU RADU Şc.”P.Rareş”, Hîrlǎu MUSEI MARIETA 46,25 I

Page 12: De ce să învăŃăm la matematică? Word - 2011 Revista Mici matematicieni...coordonate, de axe ortogonale sau nu. Ceea ce înseamnă…matematică. Reintrând într-un registru

Micii MATEMATICIENI

12

III ROSU RADU ANDREI S.A.M.”N.Iorga”Paşcani COVRIG TEODORA 45 II III PETRESCU BOGDAN Şc.”P.Rareş”, Hîrlǎu MUSEI MARIETA 43,5 II III OLARU M. ANDREEA Şc.”I. Creangǎ”,Tg.Frumos BOTEZATU MARIA 37,75 III III MORARU EDUARD Şc.”P.Rareş”, Hîrlǎu MUSEI MARIETA 36,5 III III MUSTEATA ROBERT Şc.”P.Rareş”, Hîrlǎu MUSEI MARIETA 34,5 III III TIMOFTE F. BIANCA Şc.”I. Creangǎ”,Tg.Frumos BOTEZATU MARIA 32,5 M III ANITOAIE ANDRA IOANA S. A.M. , LUNGANI IGANT CONSTANTIN 32 M III GRADINARU DRAGOS IONUT S. A.M. , LUNGANI IGANT CONSTANTIN 31,5 M III PRICOP E. ANA Şc.”I. Creangǎ”,Tg.Frumos BOTEZATU MARIA 30,25 M III PRISACARIU STEFAN LIC. „V. ALECSANDRI” IASI CRAUS ELENA 30,25 M III BARSAN CRISTIANA IOANA Lic.”M. Costin”, Paşcani PRODAN NATALIA 30 M III IONESCU PAUL - ANDREI LIC. „V. ALECSANDRI” IASI CRAUS ELENA 29 M III VICOL ŞTEFAN Şc.”P.Rareş”, Hîrlǎu MUSEI MARIETA 29 M III LUPU CEZAR S.A.M.”N.Iorga”Paşcani APETROAIE GABRIELA 28,5 M III BEZEDICA ROBERT Şc.”P.Rareş”, Hîrlǎu MUSEI MARIETA 27,5 M III SAVIN STEFAN Lic.”M. Costin”, Paşcani PRODAN NATALIA 27,5 M III MANELICI ELENA Şc.”P.Rareş”, Hîrlǎu BUDACEA MARIA 27 M III STANGA MARIA Şc.”P.Rareş”, Hîrlǎu MUSEI MARIETA 26,5 M III POLEAC ALEXANDRA Lic.”M. Costin”, Paşcani PRODAN NATALIA 26,25 M III ANGHELUS IONUT VLADUT Şcoala TANSA PRICOP OVIDIU 26 M III ONOFREI TUDOR Şc.”P.Rareş”, Hîrlǎu BUDACEA MARIA 26 M IV SURUNIUC CONSTANTIN Şc.”P.Rareş”, Hîrlǎu CRETU MARIA 45,5 I IV PUHA ALEXANDRU Şc.”P.Rareş”, Hîrlǎu CRETU MARIA 44 I IV TEHANIUC TEONA Lic.”M. Costin”, Paşcani PASAT CARMEN 44 I IV CIOBANU LAURA MARIA Şc.”P.Rareş”, Hîrlǎu CRETU MARIA 41 II IV COTNAREANU ALEXANDRU Şc.”P.Rareş”, Hîrlǎu MUNTEANU MIRELA 39,5 II IV BUTUNOI BOGDAN Şc.”P.Rareş”, Hîrlǎu OPREA EMILIA 38,5 II IV CIOLAC BOGDAN Lic.”M. Costin”, Paşcani TOMA SANDINA 36,5 III IV BUZAMURGA RALUCA Şc.”P.Rareş”, Hîrlǎu CRETU MARIA 35,5 III IV VAVILOV M. ANDREI Şc.”I. Creangǎ”,Tg.Frumos MIHOC MIHAELA 35,5 III IV LUCHIAN ANDREEA AMALIA Şc. cu cl. I-VIII PARCOVACI MUSEI IOAN 34 M IV BORTAS IONUł Şc.”P.Rareş”, Hîrlǎu OPREA EMILIA 33,5 M IV BLAGA G. ALEXANDRA Şc. cu cl. I-VIII SATU NOU BIRLADEANU MARIA 31,5 M IV LOGHIN ANDREI FLORIN Şc.”P.Rareş”, Hîrlǎu MUSEI LUMINITA 31,5 M IV CLOPOTEL MARIAN GABRIEL Lic.”B. Vodǎ”,Hǎlǎuşeşti MIRT SILVIA 30,5 M IV COSTEA DENISA Lic.”M. Costin”, Paşcani TOMA SANDINA 30,5 M IV COJOCARU COSMIN Şc.”P.Rareş”, Hîrlǎu OPREA EMILIA 28,5 M IV ISMANA IULIANA Şc. „Cezar Petrescu”, Hodora UNGUREANU MILICA 28,5 M IV BOBOC PETRU Lic.”M. Costin”, Paşcani ALECSA ECATERINA 28 M IV CRACIUN LORENA-ELENA Lic.”M. Costin”, Paşcani ALECSA ECATERINA 26,5 M IV HABDULEA RAZVAN Şc.”P.Rareş”, Hîrlǎu MUSEI LUMINITA 25 M

V GRIGORIU MARIA C.N.”M.Sadoveanu”Paşcani POPESCU CLAUDIA 43 I V PADURARU TEODOR Colegiul NaŃional, Iaşi ANITA ALICE 37,5 II V POPESCU FLAVIUS C.N.”M.Sadoveanu”Paşcani POPESCU CLAUDIA 35 II V CIUBUC M. REMUS MIHAIL Lic.”Şt.cel Mare”Hîrlǎu OANCEA GHEORGHE 28 III V COJOCARU ELISABETA Lic.”Şt.cel Mare”Hîrlǎu OANCEA GHEORGHE 28 III V CURCĂ G. IULIANA Lic.”Şt.cel Mare”Hîrlǎu OANCEA GHEORGHE 28 III V CERNESCU N. BOGDAN Lic.”Şt.cel Mare”Hîrlǎu OANCEA GHEORGHE 23,75 M V PRAJNU ANDREEA C.N.”M.Sadoveanu”Paşcani POPESCU CLAUDIA 23,75 M V CHERSAN ANDREI DANIEL Lic.”I.Neculce” Tg.Frumos ANTON MARIA 23 M V CIUBOTARU ADINA C.N.”M.Sadoveanu”Paşcani POPESCU CLAUDIA 23 M V ICHIM TEODORA C.N.”M.Sadoveanu”Paşcani POPESCU CLAUDIA 23 M VI VINTUR CRISTIAN C.N.”M.Sadoveanu”Paşcani CRACIUN MIHAI 48,5 I

Page 13: De ce să învăŃăm la matematică? Word - 2011 Revista Mici matematicieni...coordonate, de axe ortogonale sau nu. Ceea ce înseamnă…matematică. Reintrând într-un registru

Micii MATEMATICIENI

13

VI SACALEANU GABRIEL EMILIAN Lic.”Şt.cel Mare”Hîrlǎu SACALEANU IOAN 31,5 II VI MURARIU I. MARIA Lic.”Şt.cel Mare”Hîrlǎu SACALEANU IOAN 26,5 III VI ANCUTA STEFANA C.N.”M.Sadoveanu”Paşcani PRICOP VASILE 24,5 III VI SERBAN ROBERTA ANDREEA C.N.”M.Sadoveanu”Paşcani CRACIUN MIHAI 24,5 III VI ASTEFANEI RARES Lic.”I.Neculce” Tg.Frumos TURNEA MIHAELA 20 M VII CALINESCU D. ANA IOANA Lic.”Şt.cel Mare”Hîrlǎu SACALEANU IOAN 45 I VII CHIUARIU TRAIAN C.N.”M.Sadoveanu”Paşcani MARCOVSCHI IONICA 42 I VII LUPU ANDREI Şc.“I. Cantacuzino”Paşcani MIRON ANISOARA 31,5 II VII FLOREA MIHAELA Şc.”P.Rareş”, Hîrlǎu RAUTU IOAN 28 III VII LEONTE CODRIN GABRIEL Lic.”I.Neculce” Tg.Frumos DOCA LAURENTA 28 III VII MATCOVICI STEFAN C.N.”M.Sadoveanu”Paşcani MARCOVSCHI IONICA 27,5 III VII SCRIPCARIU C. GABRIEL Lic.”Şt.cel Mare”Hîrlǎu SACALEANU IOAN 27 III VII COROIANU IOANA Şc.”P.Rareş”, Hîrlǎu RAUTU IOAN 26 M VII IFRIM G. RARES CRISTIAN Lic.”Şt.cel Mare”Hîrlǎu SACALEANU IOAN 26 M VII PORUSNIUC COSMIN Şc.”P.Rareş”, Hîrlǎu RAUTU IOAN 25,5 M VII CLIZOR DANIEL S.A.M. „Şt. Cel Mare”, Cotnari MINCIUNA ADRIANA 25 M VII CONDURACHE VLAD Şc.“I. Cantacuzino”Paşcani MIRON ANISOARA 22,5 M VII PUIU SEBASTIAN Şc.“I. Cantacuzino”Paşcani MIRON ANISOARA 22 M VIII BIVOL EUSEBIU Lic.”I.Neculce” Tg.Frumos ANTON MARIA 27,25 I VIII MAFTEI SIMONA Şc.”P.Rareş”, Hîrlǎu RAUTU IOAN 25,25 II VIII MOCANU DIANA Lic.”B. Vodǎ”, Hǎlǎuceşti BALAN MARIA 24,5 III VIII DANAILA SILVIU Şc.”P.Rareş”, Hîrlǎu DORNEANU BOGDAN 23,25 M VIII CEUCA V. RAZVAN STEFAN Lic.”Şt.cel Mare”Hîrlǎu SACALEANU IOAN 20,25 M

Probleme de concurs. Bareme de corectare „Micii matematicieni”,

ediŃia a V-a, 20 martie 2010

Clasa a III-a : EnunŃuri I (20 p) :

1. ( )10p CalculaŃi :

( )3:3 9 4 :3a = + − , ( )10 3: 5 4 : 8 12 : 2 8b = − − − − şi 0 2010 2c = × + .

2. ( )10p În anul 2010 o persoană a împlinit 40 de ani. Care este data de naştere a acelei

persoane ştiind că nu a putut să-şi sărbătorească ziua în fiecare an ?

II (20 p) :

1. ( )10p Cele trei fete ale unei familii au vârstele reprezentate prin numere consecutive, iar cei

patru băieŃi prin numere impare consecutive. Cel mai mic dintre băieŃi este cu 2 ani mai mare decât cea mai mare dintre fete. AflaŃi suma vârstelor copiilor acestei familii dacă fata cea mai mică are vârsta de un an.

2. ( )10p Câte numere de cel mult 3 cifre se pot forma cu cifrele 0, 5 și 6.

III (20 p) :

1. ( )10p În timp ce Mircea mănâncă 2 struguri, Ovidiu mănâncă 3 şi Diana 5. CâŃi struguri

revin fiecăruia dacă împreună au 60 de struguri?

Page 14: De ce să învăŃăm la matematică? Word - 2011 Revista Mici matematicieni...coordonate, de axe ortogonale sau nu. Ceea ce înseamnă…matematică. Reintrând într-un registru

Micii MATEMATICIENI

14

2. ( )10p În patru lăzi sunt cantităŃi egale de portocale. Dacă din fiecare ladă se iau 30 kg de

portocale, rămân în toate lăzile, la un loc, atâtea kg de portocale câte erau la început în fiecare ladă. Câte kg de portocale erau la început în cele patru lăzi la un loc?

Subiecte elaborate/ modificate/ selectate şi propuse de: prof. Gheorghe Oancea, înv. Maria CreŃu şi înv. Mirela Munteanu.

SoluŃii. Barem de corectare I 1. Avem: ( )1 5 :3 6 : 3 2a = + = = (3p); ( ) [ ]10 3: 5 4 : 8 6 10 3: 5 4 : 2b = − − − = − − (2p);

[ ]10 3: 5 2 10 3:3 10 1 9b = − − = − = − − (3p); şi 0 2 2c = + = (2p).

2. Anul de naştere este 2010 40 1970− = (5p). Cum persoana nu a putut sǎ-şi sǎrbǎtoreascǎ ziua de naştere în fiecare an rezultǎ cǎ s-a nǎscut pe 29 februarie (5p). II 1. Cum cea mai micǎ fatǎ are 1 an şi fetele au vârste consecutive rezultǎ cǎ ele au vârstele: 1, 2, 3 (2p). Deoarece cel mai mic bǎiat este cu 2 ani mai mare decât cea mai mare fatǎ deducem cǎ vârstele bǎieŃilor sunt: 5, 7, 9, 11 (5p); . Suma vârstelor copiilor acestei familii este 1 2 3 5 7 9 11 38+ + + + + + = (3p). 2. Numerele de o cifrǎ sunt 0, 5 şi 6 (1p). Numerele de 2 cifre sunt 50, 55, 56, 60, 65 şi 66 (3p). Numerele de 3 cifre sunt 500, 505, 506, 550, 555, 556, 560, 565, 566, 600, 605, 606, 650, 655, 656, 660, 665 şi 666 (5p). În total, avem 3 6 18 27+ + = numere formate cu cel mult 3 cifre (1p). III 1. Planul logic al rezolvǎrii este urmǎtorul : � Câte piersici mǎnâncǎ împreunǎ într-un anumit interval de timp ?

� Mircea: 2 ; Ovidiu: 3 şi Diana: 5. În total 10 piersici…(2p); � În câte intervale de timp mǎnâncǎ împreunǎ cele 60 piersici ?

� 10 se cuprinde în 60 de 5 ori ( 60 :10 5= )…(2p); � Câte piersici revine fiecǎruia ? Mircea: 5 2 10× = …(2p); Ovidiu:5 3 15× = … (2p), iar Diana: 5 10 50× = piersici …(2p). 2. Notǎm cu a , cantitate de portocale din fiecare ladǎ . Avem: 4 4 30a a⋅ − ⋅ = . Scrierea relaŃiilor sau reprezentarea graficǎ (5p). Rezolvarea: 4 4 30 3 120a a a⋅ − = ⋅ ⇒ ⋅ = şi aflarea lui

40a kg= (3p). În cele 4 lǎzi sunt 4 40 160 kg⋅ = (2p).

Clasa a IV-a : EnunŃuri

I (20 p)

1. ( )16p ArătaŃi că: ( ) ( ){ }5 : 2 3 1 14 :15 3 : 1 2 45 :15 2 : 2 2008 2009⋅ − + + + ⋅ − + = .

AlegeŃi una dintre cifrele care intră în alcătuirea numerelor din exerciŃiu şi eliminaŃi-o, astfel încât egalitatea să rămână adevărată. VerificaŃi noua egalitate. 2. ( )4p Numerele de patru cifre distincte, a şi b au suma cifrelor fiecăruia egală cu 11. Ştiind

că a este cel mai mic număr având exact trei cifre pare, iar b este cel mai mic număr având exact trei cifre impare, să se arate că numărul 2009a b+ − are suma cifrelor egală cu 11.

II (20 p) : CitiŃi cu atenŃie enunŃurile următoarelor probleme pentru a le putea rezolva corect!

1. ( )10p DiferenŃa a două numere este 2009. AflaŃi numerele, ştiind că unu este rezultatul

scăderii lui 2010 din suma lor. 2. ( )10p DiferenŃa a două numere este 2009. AflaŃi numerele, ştiind că unul este rezultatul

scăderii lui 2010 din suma lor.

Page 15: De ce să învăŃăm la matematică? Word - 2011 Revista Mici matematicieni...coordonate, de axe ortogonale sau nu. Ceea ce înseamnă…matematică. Reintrând într-un registru

Micii MATEMATICIENI

15

III (20 p) :

1. ( )10p Profesorul Aritmel a inventat o nouă operaŃie notată cu semnul „$” şi care operează

astfel: $ a b este suma dintre produsul cifrelor nenule ale numărului a şi suma cifrelor numărului b. CalculaŃi cu noua regulă: 2009 $ 2008 şi ( )29 $ 28 $ 27 .

2. ( )10p În timp ce Rareş mănâncă 2 piersici, Mateea mănâncă 3, iar Teodor 10. Câte piersici

revin fiecăruia dacă împreună au 60 piersici. Subiecte elaborate/ modificate/ selectate şi propuse de:

prof. Ioan Sǎcǎleanu, înv. Tereza Ruginǎ, înv. Marieta Muşei. SoluŃii. Barem de corectare

I 1. Avem succesiv : ( ) ( ){ }5 : 6 1 14 :15 3 : 1 2 3 2 : 2 2008 2009− + + + ⋅ − + = (2p) ⇔

[ ] [ ]{ }5 : 5 14 :15 3 : 1 2 1 : 2 2008 2009+ + + ⋅ + = (2p) [ ] [ ]{ }1 14 :15 3 : 1 2 : 2 2008 2009+ + + + =

(2p) { }15 :15 3:3 : 2 2008 2009+ + = (2p) { }1 1 : 2 2008 2009+ + = (2p) 2 : 2 2008 2009+ =

1 2008 2009⇔ + = 2009 2009⇔ = , evident adevǎratǎ. (2p). Prin eliminarea unei cifre se înŃelege ştergerea ei oriunde apare în egalitate. Eliminǎm cifra 0

(2p). Avem ( ) ( ){ }5 : 2 3 1 14 :15 3 : 1 2 45 :15 2 : 2 28 29⋅ − + + + ⋅ − + = . Cum acolada are

valoarea 2 se verificǎ uşor adevǎrul egalitǎŃii obŃinute. (2p) 2. Pentru a fi cele mai mici, numerele a şi b au cifra miilor 1, iar cifra sutelor 0, rǎmânând ca suma dintre cifra zecilor şi a unitǎŃilor sǎ fie egalǎ cu 10. Prin urmare 1028a = (1p) şi 1037b = (1p). Avem 2009 1028 1037 2009 56a b+ − = + − = (1p). Se verificǎ cǎ suma cifrelor lui

2009a b+ − este 5 6 11+ = (1p) II Notǎm cu a şi cu b numerele cǎutate. Avem 2009a b− = (2p).

1. Stabilirea relaŃiilor şi aflarea sumei (3p): ( )1 2010a b= + − 2011a b⇒ + = . Reprezentarea

graficǎ sau scrierea şi rezolvarea algebricǎ (3p): se adunǎ relaŃiile şi se obŃine 2 4020a⋅ = , de unde 2010a = , iar 1b = (2p). 2. Cazul I: dacǎ ( ) 2010a a b= + − atunci 2010a a b+ = + , de unde 2010b = , iar 4019a = .

Cazul II: dacǎ ( ) 2010b a b= + − atunci 2010b a b+ = + , de unde 2010a = , iar 1b = .

Elevii obŃin punctaj maxim pentru rezolvarea ambelor cazuri. Rezolvarea corectǎ a oricǎrui caz se puncteazǎ cu (7p), iar gǎsirea şi rezolvarea celui de al II-lea caz (3p). Indiferent de abordare, rezolvarea parcurge etapele:

� Reprezentarea graficǎ sau stabilirea relaŃiilor ....... 4 puncte / 2 puncte � Aflarea numerelor ....... 3 puncte / 1 punct

III 1. Produsul cifrelor lui 2009 este 18 (1p), iar suma cifrelor lui 2008 este 10 (1p). Atunci

2009$2008 18 10 28= + = (3p). Calculul ( )29$28 2 9 2 8 18 10 28= ⋅ + + = + = (2p). Calculul

valorii expresiei ( ) ( )29$28 $27 28$27 2 8 2 7 16 9 25= = ⋅ + + = + = (3p).

3. Aflarea sumei piersicilor consumate la o „încercare”: 2 3 10 15+ + = piersici (2p). Stabilirea numărului de „ încercări” : 60 :15 4= încercǎri (2p). Fiecǎrui copil îi revin : Rareş 4 2 8⋅ = piersici (2p), Mateea 4 3 12⋅ = piersici (2p), iar Teodor 4 10 40⋅ = pirsici (2p).

Page 16: De ce să învăŃăm la matematică? Word - 2011 Revista Mici matematicieni...coordonate, de axe ortogonale sau nu. Ceea ce înseamnă…matematică. Reintrând într-un registru

Micii MATEMATICIENI

16

Clasa a V-a : EnunŃuri I (20 p):

1. ( )10p ArătaŃi că 3 2005 2 2006 2007 2008 4 2005 20093 2 3 2 3 2 2 3 2 2⋅ + ⋅ + ⋅ + = ⋅ − .

2. ( )10p Să se arate că suma a patru numere naturale consecutive, nedivizibile cu 5 este

divizibilă cu 5.

II (20 p) :

1. ( )11p Fie egalitatea: 284xy cy xc yx x+ + + + = . Ştiind că y şi c sunt consecutive,

demonstraŃi că numerele şi sunt consecutive.

2. ( )9p Aida este născută în luna februarie, ziua ax , anul abcd . Se ştie că ax este număr prim,

4a x+ = şi cǎ numǎrul abcd împărŃit la ax dă câtul 52a şi restul 2. DeterminaŃi ziua şi anul naşterii Aidei.

III (20 p) : Se consideră numerele naturale 22009 2009m = + şi n suma tuturor numerelor de

forma 2010

200c , unde c este o cifră din sistemul de numeraŃie zecimal. a) ( )14p Să se demonstreze că numerele m şi n se divid cu 5;

b) ( )3p AflaŃi cel mai mare număr natural k astfel încât numărul m să se dividă cu 7k ;

c) ( )3p Să se determine câtul şi restul împărŃirii numărului m la numărul 2747 .

Subiecte elaborate/ modificate/ selectate şi propuse de:

Prof. Ioan RǎuŃu, prof. Iuliana Blanariu şi prof. Oana Rǎdeanu. SoluŃii.Barem de corectare I 1. Membrul stâng al egalitǎŃii din enunŃ este egal cu ( )2005 3 2 2 32 3 2 3 2 3 2⋅ + ⋅ + ⋅ + (3p), adicǎ

20052 65⋅ (2p), iar membrul drept este egal cu ( )2005 4 42 3 2⋅ − (3p), adicǎ 20052 65⋅ (1p). Prin

urmare are loc egalitatea (1p). 2. Numerele nedivizibile cu 5 au forma 5 1k + , 5 2k + , 5 3k + sau 5 4k + (5p). Suma acestora

este ( )20 10 5 4 2k k+ = ⋅ + (4p), adicǎ suma este divizibilǎ cu 5 (1p).

II 1. Avem 10 10 10 10 284x y c y x c y x x+ + + + + + + + = , adicǎ 22 12 11 284x y c+ + =

22 11 11 275 9x y c y⇔ + + − = − ( )11 2 25 9x y c y⇔ ⋅ + + − = − (3p), de unde deducem cǎ cifra

119 y− ∈Μ . ObŃinem 9y = (2p). Rezultǎ cǎ 2 25 0 2 16x y c x c+ + − = ⇔ + = , de unde c este

cifrǎ parǎ (1p). Gǎsim cǎ perechilor ( ) ( ) ( ) ( ) ( ){ }, 8;4 , 6;5 , 4;6 , 2;7c x ∈ (2p). Cum cifrele y şi c

sunt consecutive şi cum 9y = rezultǎ cǎ 8c = (1p). Atunci 2 2 80x c+ = (1p). Cum 2 81y =

deducem cǎ numerele 2 2x c+ şi 2y sunt consecutive (1p)..

2. Luna februarie are maxim 29 zile. Deci 29ax ≤ . Din ax număr prim

{ }11,13,17,19, 23, 29ax⇒ ∈ (4p). Dar 4a x+ = , deci 13ax = (2p). Anul naşterii se obŃine din

52 2abcd ax a= ⋅ + , de unde 13 152 2abcd = ⋅ + (2p). Data este 13 februarie 1978 (1p).

Page 17: De ce să învăŃăm la matematică? Word - 2011 Revista Mici matematicieni...coordonate, de axe ortogonale sau nu. Ceea ce înseamnă…matematică. Reintrând într-un registru

Micii MATEMATICIENI

17

III a) Scoaterea factorului comun ne dă 22009 2009 2009 2010m = + = ⋅ , adică m se divide

cu 5 căci factorul 2010 are ultima cifră 0 (3p). Cum ultima cifră se repetă din 4 în 4

rezultă că ( ) ( ) ( )2010 2010 2200U c U c U c= = căci 42010 2= Μ + (4p). Avem că

( ) ( ) ( )0 1 4 9 6 5 6 9 4 1 25 5U n U U= + + + + + + + + + = = (6p). Deci n se divid cu 5 (1p).

b) Descompunerea lui 22009 2010 7 41 2 5 3 67m = ⋅ = ⋅ ⋅ ⋅ ⋅ ⋅ (2p) şi cum m să se dividă cu 7k rezultă că cel mai mare număr natural k este 2 (1p).

c) Avem ( ) ( )241 67 7 2 5 3 2747 1470m = ⋅ ⋅ ⋅ ⋅ ⋅ = ⋅ (2p) . Din unicitatea câtului şi restului rezultă

câtul 1470 şi restul egal cu 0 la împărŃirea lui m la numărul 2747 (1p).

Clasa a VI-a : EnunŃuri I (20 p) :

1. ( )10p DeterminaŃi numerele naturale mai mici decât 1000 care împărŃite pe rând la 18, 24,

42 şi 56 dau de fiecare dată restul 7.

2. ( )10p DeterminaŃi ştiind că ; ; sunt direct proporŃionale cu numerele

; ; şi că .

II (20 p) : 1. ( )10p Să se arate că dacă( ) ( )

1 1 1

0,00,x xx+ + ∈ℕ atunci 2 5x − este divizor

întreg al lui 2010. 2. ( )10p Trei elevi trebuiau să-şi plătească o excursie cu sume direct proporŃionale cu numerele

3, 4 şi 5. Dacă taxa fixată fiecăruia se majorează cu 10% , 15% , respectiv 20% atunci ei ar plăti fiecare cu 120 lei mai puŃin decât dacă taxele se majorează cu 20% , 25% , respective 30%. Ce sumă trebuia să plătească fiecare elev?

III ( ) 20 p : 1. ( )10p În jurul punctului O se consideră unghiurile AOB∢ , BOC∢ , COD∢ ,

DOE∢ şi EOA∢ astfel încât măsurile lor sunt direct proporŃionale cu cinci numere naturale

consecutive, iar ( )( )

1

3

m AOB

m BOE=

∢, demonstraŃi că punctele A , O şi D sunt coliniare.

2. ( )10p Se dă ABC△ cu [ ] [ ]AB AC≡ . Se iau punctele ( )D AB∈ şi ( )E AC∈ astfel încât D

este mijlocul lui AB şi AE EC= . Să se demonstreze că [ ] [ ]BE CD≡ .

Subiecte elaborate/ modificate/ selectate şi propuse de: Prof. Mircea Popa, prof. Corina Odochia şi prof. Ramona Darie.

SoluŃii.Barem de corectare I 1. Fie numărul căutat. Din enunŃ se obŃine cǎ : 1 1818 7 7x c x= ⋅ + ⇒ − ∈Μ ;

2 2424 7 7x c x= ⋅ + ⇒ − ∈Μ ; 3 4242 7 7x c x= ⋅ + ⇒ − ∈Μ şi 3 5656 7 7x c x= ⋅ + ⇒ − ∈Μ (3p).

Rezultǎ cǎ 18 24 42 567x − ∈Μ ∩Μ ∩Μ ∩Μ (2p). Deoarece [ ] 3 218,24,42,56 2 3 7 504= ⋅ ⋅ =

rezultǎ cǎ 5047x − ∈Μ (2p). Cum 1000x < rezultǎ cǎ (2p). { }7 0;504x − ∈ Prin urmare

{ }7;511x∈ (1p).

Page 18: De ce să învăŃăm la matematică? Word - 2011 Revista Mici matematicieni...coordonate, de axe ortogonale sau nu. Ceea ce înseamnă…matematică. Reintrând într-un registru

Micii MATEMATICIENI

18

2. Avem : 2 2 2

22 2 2

rapoarte

egale

a b b c c a a b c

a b b c c a a b c

+ + + + += = =

− − − + += (4p). Din 22

a b

a b

+=

− rezultǎ cǎ

4 2a b a b+ = − 3 3b a⇒ = b a⇒ = (1p). Din 22

b c

b c

+=

− obŃinem 4 2b c b c+ = − c b⇒ = (1p).

Folosind 81b a ba b c+ + = obŃinem cǎ 3 81bb⋅ = (1p)., de unde 33 3bb b= ⇒ = (2p). Deci

333abc = (1p).

II 1. Transformând în fracŃii ordinare obŃinem : 1 9 90 100

x x x x+ + ∈ ⇔ ∈ℕ ℕ (3p), de unde

rezultǎ cǎ cifra x este divizor al lui 100 (1p). Deci { }1,2,4,5x∈ (2p), de unde

{ }2 5 3, 1,3,5x − ∈ − − (1p). Cum avem descompunerea lui 2010 1 2 3 5 67= ⋅ ⋅ ⋅ ⋅ (2p) rezultǎ cǎ

2 5x − este divizor întreg al lui 2010 (1p).

2. Sumele a , b , c fiind direct proporŃionale cu 3, 4, 5⇒ .

3 4 5

nota b ck= = = (2p), de unde 3a k= ,

4b k= şi 5c k= . Avem10 15 20 20 25 30

120100 100 100 100 100 100

a b c a b c⋅ + ⋅ + ⋅ + = ⋅ + ⋅ + ⋅ (3p), relaŃia

dintre majorǎri. Atunci 10 3 15 4 20 5 12000 20 3 25 4 30 5k k k k k k⋅ + ⋅ + ⋅ + = ⋅ + ⋅ + ⋅ de unde 190 12000 310k k⇒ + = 12000 120k⇒ = 100k⇒ = (3p). Deci 300a = , 400b = şi

500c = (2p).

III 1. Avem ( ) ( ) ( ) ( ) ( ) 0360

1 2 3 4 5 10

m AOB m BOC m COD m DOE m EOA

n n n n n n= = = = =

+ + + + +

∢ ∢ ∢ ∢ ∢

(2p). Din ( )

( )

0360

2 5 2

m COD

n n=

+ ⋅ +

∢( ) 072m COD⇒ =∢ (1p). Din ipoteza

( )( )

1

3

m AOB

m BOE=

∢ rezultǎ

cǎ ( ) ( )3m BOE m AOB= ⋅∢ ∢ (1p). Dar ( ) ( ) ( )m BOE m BOA m AOE= +∢ ∢ ∢ , de unde

( ) ( ) ( ) ( )3 2m AOE m AOB m AOB m AOB= ⋅ − = ⋅∢ ∢ ∢ ∢ (1p). Înlocuind în

( ) ( )4

m AOB m EOA

n n=

+

∢ ∢ 1 24

4n

n n⇒ = ⇒ =

+ (1p). Deci ( ) ( )

2

nm AOB m COD

n= ⋅

+∢ ∢

( ) 0 0472 48

6m AOB⇒ = ⋅ =∢ şi ( ) 0 05

72 606

m BOC = ⋅ =∢ (2p). Prin urmare avem cǎ

( ) ( ) ( ) ( ) 0 0 0 048 60 72 180m AOD m AOB m BOC m COD= + + = + + =∢ ∢ ∢ ∢ . Rezultǎ cǎ AOD∢

este alungit, adicǎ A, O, D sunt puncte coliniare (2p). Clasa a VII-a : EnunŃuri

I (20 p) 1. ( )10p CalculaŃi: ( ) ( ) ( )( )2 2

3 2 2 2 3 2 3 3 2 2 3 3 2 2− + + − − + .

2. ( )10p DeterminaŃi numărul natural 1n > pentru care egalitatea de mai jos este adevărată:

( ) ( )1 124 3 1 6 5 6 594 81 2n n n− −⋅ − − ⋅ − = + ⋅ .

II (20 p) 1. ( )10p Fie numerele naturale nenule x şi y . Notǎm cu 3

4 3

x ya

y

+=

+ ,

4 1

9

yb

+= şi

cu 11

3c

x y=

+. Ştiind cǎ a b c= = , sǎ se arate cǎ x y> .

Page 19: De ce să învăŃăm la matematică? Word - 2011 Revista Mici matematicieni...coordonate, de axe ortogonale sau nu. Ceea ce înseamnă…matematică. Reintrând într-un registru

Micii MATEMATICIENI

19

2. ( )10p Fie ,x y ∗

+∈ℝ . DeterminaŃi media geometricǎ a numerelor x şi y ştiind cǎ 3x y+ = şi

cǎ ( ) ( )4 4

24x y x y+ − − = .

III (20 p) : 1. ( )10p Un romb are diagonalele de 18 cm şi , respectiv 24 cm. DeterminaŃi

perimetrul şi aria rombului. 2. ( )10p Fie paralelogramul ABCD, în care AD DB⊥ . Fie E şi N pe dreapta BC astfel încât

( )B CE∈ , ( )E BN∈ şi [ ] [ ] [ ]CB BE EN≡ ≡ . a) ( )4p Demonstrați că ABN△ este isoscel;

b) ( )6p Dacă M este simetricul lui B faŃă de D, arătaŃi că punctele N, A, M sunt coliniare.

Subiecte elaborate/ modificate/ selectate şi propuse de:

Prof. Constantin Nastasǎ, prof. Mihaela Turnea şi prof. Cristina Macovei. SoluŃii.Barem de corectare

I 1. Aplicǎm regulile de calcul prescurtat: ( )23 2 2 3 4 3 2 8 11 4 6− = − ⋅ + = − (3p),

( )22 3 2 12 4 3 2 2 14 4 6+ = + ⋅ + = + (3p), ( )( )3 3 2 2 3 3 2 2 27 8 19− + = − = (3p).

Prin urmare, expresia are valoarea 11 4 6 14 4 6 19 44− + + + = (1p). 2. Avem 1 124 3 24 30 6 6 594 81 2n n n− −⋅ − − + ⋅ = + ⋅ (3p), 8 3 6 648 81 2n n n⋅ + = + ⋅ (2p),

( ) ( )3 8 2 81 2 8n n n⋅ + = ⋅ + (3p). Cum 2 8 0n + ≠ obŃinem 3 81n = 43 3 4n n⇔ = ⇔ = (2p).

II 1. Din 3 4 1 11 3 5 12

14 3 9 3 3 5 12

x y y x ya b c

y x y x y

+ + + += = ⇒ = = = =

+ + + + (3p). Din

4 11

9

y += rezultǎ

4 1 9 4 8 2y y y+ = ⇒ = ⇒ = (3p). Din 11

1 11 3 2 3 9 33

x x xx y

= ⇒ = + ⇒ = ⇒ =+

(3p). Prin

urmare, x y> (1p).

2. DiferenŃa de pǎtrate : ( ) ( ) ( ) ( )2 2 2 2

24x y x y x y x y + − − + + − = (3p), de

unde ( )( )( )2 2 24x y x y x y x y x xy y x xy y+ + − + − + + + + − + = (3p), adicǎ

( )2 2 2 2 24x y x y⋅ ⋅ + = (1p) ( ) 3xy x y⇔ ⋅ + = (1p) 1xy⇔ = (1p). Prin urmare media

geometricǎ este 1gm xy= = (1p).

III

O

A

B

C

D

1 2

9

1fig u ra

M

N

AB

CD

E

2figura

Page 20: De ce să învăŃăm la matematică? Word - 2011 Revista Mici matematicieni...coordonate, de axe ortogonale sau nu. Ceea ce înseamnă…matematică. Reintrând într-un registru

Micii MATEMATICIENI

20

1. Realizarea figurii 1 se puncteazǎ cu (1p). Din ABCD romb obŃinem cǎ 24

122 2

ACOA = = =

şi 18

92 2

BDOB = = = ( 2p). . Cum rombul are AC BD⊥ rezultǎ cǎ în

AOB△ avem: 2 2 2 2 212 9 144 81 15AB OA OB AB= + ⇒ = + = + = (3p).

Perimetrul este egal cu 4 60AB⋅ = (2p), iar aria este egalǎ cu 24 18

2162 2

AC BD⋅ ⋅= = (2p).

2. a) Realizarea figurii cu elementele necesare rezolvǎrii se puncteazǎ cu (1p). Din

[ ] [ ]CB BE≡ ⇒cǎ B este mijlocul lui [ ]CE . Din faptul cǎ ABCD este paralelogram, aflǎm cǎ

O este mijlocul lui [ ]AC . Deducem cǎ OB este linie mijlocie în ACE△ (1p)., de unde OB AE�

şi cum OB BC⊥ (ip.) rezultǎ AE înǎlŃime în ABN△ (1p). Dar, [ ] [ ]BE EN≡ atunci AE este şi

medianǎ în ABN△ . Deci ABN△ este triunghi isoscel de bazǎ [ ]BN (1p).

b) Din ABCD paralelogram AD BC� , de unde AD BE� , dar şi ip

AD BC BE= = . Deducem cǎ

AEBD este paralelogram şi rezultǎ AEBD este dreptunghi ( ) 090m DAE⇒ =∢ (2p). Din ABN△

isoscel rezultǎ AD este bisectoarea BAN∢ , de unde ( ) ( )2m BAN m BAE= ⋅∢ ∢ (1p). Din M,

simetricul lui B faŃǎ de D şi cum AD DB⊥ (ip.) rezultǎ cǎ ABM∆ este isoscel, de unde AD este bisectoarea BAM∢ , de unde ( ) ( )2m BAM m BAD= ⋅∢ ∢ (1p). ObŃinem cǎ

( ) ( ) ( )m MAN m BAM m BAN= +∢ ∢ ∢ ( ) ( ) ( )( )2m MAN m BAD m BAE⇒ = ⋅ +∢ ∢ ∢

( ) ( ) 0 02 2 90 180m MAN m DAE⇒ = ⋅ = ⋅ =∢ ∢ MAN⇒∢ unghi alungit, de unde N, A, M sunt

puncte coliniare (2p). Clasa a VIII-a : EnunŃuri I (20 p) :

1. ( )12p Fie x, y numere reale cu 1y ≠ − . ArătaŃi că:

a) ( )6p dacă ( )2 2 2x y x y− = + atunci { }11, 1

1

x

y

−∈ − +

+

b) ( )6p dacă { }11, 1

1

x

y

−∈ − +

+ atunci ( )2 2 2x y x y− = +

2. ( )8p Suma tuturor muchiilor unui paralelipiped dreptunghic este 480 dm, iar diagonala

paralelipipedului este 5 2m. Să se afle aria totală a paralelipipedului în 2m .

II (20 p) :

1. ( )12p Fie funcŃia :f →ℝ ℝ , ( )f x ax b= + , ,a b∈ℝ , 0a ≠ cu proprietatea

( ) ( ) ( )3 2 2 11f x x f= + ⋅ − , oricare ar fi x număr real.

a) ( )4p DeterminaŃi funcŃia f.

b) ( )6p CalculaŃi perimetrul şi aria triunghiului determinat de axele de coordonate şi

graficul funcŃiei f. c) ( )2p AflaŃi distanŃa de la originea axelor la graficul funcŃiei.

Page 21: De ce să învăŃăm la matematică? Word - 2011 Revista Mici matematicieni...coordonate, de axe ortogonale sau nu. Ceea ce înseamnă…matematică. Reintrând într-un registru

Micii MATEMATICIENI

21

2. ( )8p DeterminaŃi n∈ℤ pentru care A să fie cel mai mare număr natural, pătrat perfect

mai mic decât 2010, unde 2 22 1 1 8 1

: 24 3 12 6 2

n n n n n nA

− + + + += − + + ⋅

III (20 p): Fie ABC△ , dreptunghic în A cu AB AC m= = . Se consideră un punct D exterior

planului ( )ABC astfel încât tetraedrul ABCD să aibă toate feŃele, triunghiuri dreptunghice şi

exact trei (din cele şase) muchii de lungime m . Să se determine valoarea lui m ştiind că valoarea numerică a ariei totale a tetraedrului este egală cu cea a volumului.

Subiecte elaborate/ modificate/ selectate şi propuse de: Prof. Aurel Neicu, prof. Oana Felicia Alexe şi prof. Bogdan Dorneanu.

SoluŃii.Barem de corectare

I 1. a) : Din ( )2 2 2x y x y− = + obŃinem 2 22 1 2 1x x y y− + = + + ( ) ( )2 21 1x y⇔ − = + (3p)

Cum 1y ≠ − rezultǎ 2

11

1

x

y

−= +

. Deci { }11, 1

1

x

y

−∈ − +

+ (3p).

b) Din { }11, 1

1

x

y

−∈ − +

+⇒

21

11

x

y

−= +

(2p) ( ) ( )2 21 1x y⇔ − = + ⇔ ( 2 22 1 2 1x x y y− + = + +

(2p), de unde ( )2 2 2x y x y− = + (2p).

2. Notǎm cu , ,a b c dimensiunile paralelipipedului dreptunghic. Avem: 4 4 4 48a b c+ + = , de

unde 12a b c+ + = (1p). Formula diagonalei este 2 2 2 2d a b c= + + (1p), iar aria totalǎ este egalǎ 2 2 2t ab ac bcΑ = + + (1p). Din pǎtratul unui trinom obŃinem cǎ

( )2 2 2 2 2 2 2a b c a b c ab ac bc+ + = + + + + + (2p)., adicǎ ( )2 2ta b c d+ + = + Α (1p). Înlocuind

gǎsim ( )2212 5 2 t= + Α , de unde 294tA m= (2p).

II 1. a): Pentru 2x = avem ( ) ( )2 12 2 11f f= ⋅ − (2p), de unde ( )2 1f = (1p). ObŃinem

legea funcŃiei datǎ de ( ) 3 5f x x= − (1p).

b) Coordonatele punctului de intersecŃie al graficului cu axa Ox se obŃine din ( ) 0f x = , adicǎ

punctul 5,0

3A

(1p), iar cu axa Ox : ( )0 5y f= = − , adicǎ ( )0, 5B − (1p). DistanŃa AB se

obŃine folosind formula distanŃei sau teorema lui Pitagora în ( ).OAB dr O△ şi anume

22 2 25 5 10

53 3

AB OA OB = + = + =

(2p). Perimetrul cǎutat este OABP OA OB AB= + +△

20 5 10

3 3OABP⇔ = +△ (1p), iar aria este 1 5 25

52 2 3 6OAB

OA OBA

⋅= = ⋅ ⋅ =△ (1p).

c) DistanŃa de la originea reperului la AB , graficul funcŃiei f este înǎlŃimea ( ).OAB dr O△ (1p).

Atunci ( )2 5 3 10

; 53 25 10

T h

f

OA OBd O G

AB

⋅⇒ = = ⋅ ⋅ = (1p).

2. Cel mai mare pǎtrat perfect mai mic ca 2010 este 21936 44= . Avem 1936A = (1p).

Page 22: De ce să învăŃăm la matematică? Word - 2011 Revista Mici matematicieni...coordonate, de axe ortogonale sau nu. Ceea ce înseamnă…matematică. Reintrând într-un registru

Micii MATEMATICIENI

22

Avem2 26 3 4 4 8 6

212 1 2

n n n n nA

n

− − − + + / += ⋅ + ⋅ / +

(1p)⇔( )

( )2 12 12

2 1 2

n nn nA

n

⋅ ++ += + ⋅

+

1p)( )( )

( )21 1

22 1 2

n n nA

n

/ + ⋅ +⇔ = + ⋅

+ (2p) ( ) ( )1 1A n n n⇔ = + + + .

Prin urmare ( )21A n= + (2p) 1 1936 1 1935A n n n⇔ = + ⇔ = + ⇔ = (1p).

III Triunghiul ABC△ având AB AC m= = şi fiind dreptunghic în A are 2BC m= (1p).

• Deoarece tetraedrul DABC are toate feŃele triunghiuri dreptunghice şi numai 3 muchii de lungime m , rǎmâne sǎ stabilim care dintre muchiile DA , DC sau DB are lungimea m astfel încât feŃele DCA, DAB şi DCB sǎ fie triunghiuri dreptunghice . • Deducem cǎ DA m≠ cǎci alfel DAC△ şi DAC△ sunt isocele şi dreptunghice, de unde

. .T P

⇒ 2DC DB m= = şi cum 2BC m= gǎsim cǎ DCB△ este echilateral, în contradicŃie cu faptul cǎ el este dreptunghic (3p). Vom analiza cazurile:

Cazul 1: Dacǎ DC m= atunci DCA△ este dreptunghic în C, de

unde DC CA⊥ şi 2DA m= (1p) Vom stabili vârful unghiului drept al triunghiului DAB.

• Dacǎ DAB△ este dreptunghic în D atunci ipotenuza

AB DA> 2m m⇒ > 1 2⇒ > , absurd (2p). • Dacǎ DAB△ este dreptunghic în B atunci

.2 2

T P

DB DA AB m⇒ = − = , contradicŃie cu ipoteza cǎ tetraedrul are numai trei muchii de lungime m (2p).

Prin urmare DAB△ este dreptunghic în A, deci DA AB⊥ şi cum DC CA⊥ şi CA AB⊥ rezultǎ dintr-o reciprocǎ a teoremei celor 3 perpendiculare cǎ ( )DC ABC⊥ (3p). Atunci volumul este

3

3 3 2 6ABCDC A m m m m

V⋅ ⋅

= = ⋅ = (2p), iar aria totalǎ este

( )222 2 1 2

2 2t

m m m mA m

⋅ ⋅= ⋅ + ⋅ = ⋅ + (3p). Cum tA V= rezultǎ ( )6 1 2m = ⋅ + (2p). Cazul

2: DB m= , se analizeazǎ „în oglindǎ” obŃinându-se acelaşi ( )6 1 2m = ⋅ + (1p).

∗∗∗

Zâmbetul ŞtiinŃei Gottfried Wilhelm Leibniz (1646 - 1716)

De la Leibniz ne-a rămas fraza: " Tout est pour le mieux dans le meilleur des mondes possibles ", adică "totul este pentru mai bine în cea mai bună dintre lumile posibile". Această maximă, considerată teorema fundamentală a optimismului, a fost ironizată şi răstălmăcită de Voltaire în lucrarea sa Candide. Până în anul 1672, Leibniz nu s-a ocupat deloc de matematici. La această vârstă a luat lecŃii cu Christian Huygens şi a ajuns ca în acelaşi timp cu Newton să fie descoperitorul calculului diferenŃial şi integral. Aceasta nu l-a împiedicat totuşi pe Leibniz să facă la un moment dat şi o eroare, deoarece a crezut că derivata produsului a două functii este egal cu produsul derivatelor. Eroarea însă a corectat-o el însuşi.

Culeasǎ de Bobîrnǎ Petru Costin

A

BC

D

mm

m2m

3m

2m

Page 23: De ce să învăŃăm la matematică? Word - 2011 Revista Mici matematicieni...coordonate, de axe ortogonale sau nu. Ceea ce înseamnă…matematică. Reintrând într-un registru

Micii MATEMATICIENI

23

Testarea absolvenŃilor de clasa a IV-a în vederea înscrierii în clasa a V-a

Varianta nr. 1, mai 2010

Subiectul I (23 puncte): CalculaŃi 3 5 12 :a b c× − × + ştiind cǎ:

( )3:3 9 4 :3a = + − ; ( )10 3: 5 4 : 8 12 : 2 8b = − − − − ; ( ) ( )14 6 :5 3 15 6 : 7 1c = + − ⋅ + − .

Subiectul II (28 puncte): GǎsiŃi valoarea lui a din egalitatea :

( )35 4 24 :3 13 :10 5 18 :3 11a− × − + × + =

Subiectul III (29 puncte): Mama are 2 gemeni. AflaŃi ce vârstǎ au bǎieŃii şi mama, ştiind cǎ împreunǎ cei trei au 60 de ani, iar vârsta mamei este de 3 ori mai mare decât vârsta unui bǎiat. Peste câŃi ani vârsta mamei va fi de 2 ori mai mare decât vârsta unui bǎiat ? Subiectul IV (10 puncte): Cele trei fete ale unei familii au vârstele reprezentate prin numere consecutive, iar cei patru băieŃi prin numere impare consecutive. Cel mai mic dintre băieŃi este cu 2 ani mai mare decât cea mai mare dintre fete. AflaŃi suma vârstelor copiilor acestei familii dacă fata cea mai mică are vârsta de un an.

Varianta nr. 2, mai 2010 Subiectul I (29 puncte): Se dau numerele:

6 7 5 6 72 :8a = × − × − ; 64 :8 3 4 4 4b = + × − × şi 42 : 6 16 :8c = − . CalculaŃi 4 3 2a b c× − × + × . Subiectul II (20 puncte): AflaŃi valoarea lui b din egalitatea:

( ){ }6 6 : 5 4 : 3 10 : 5 : 2007 2010b b− − − × = − .

Subiectul III (21 puncte): În anul 2004, Rǎzvan şi tatǎl sǎu aveau împreunǎ 48 de ani. Dacǎ în 2006 tatǎl sǎu va avea 39 ani, aflaŃi:

a) Ce vârstǎ va avea Rǎzvan în anul 2006 ? b) CâŃi ani vor avea împreunǎ în anul 2010 ?

Subiectul IV (20 puncte): a) (15 puncte) EfectuaŃi calculul: 201 5 67 5 67 10× + × + × . b) (5 puncte) Se ştie cǎ un calculator efectueazǎ o înmulŃire în 2 secunde şi o adunare într-o

secundǎ. Câte secunde îi sunt necesare pentru efectuarea calculului din exerciŃiul de la punctul a).

Varianta nr. 3, mai 2010

Subiectul I (25 puncte): CalculaŃi:

a) ( )3 5 39 5 49 2 16 6 : 2 : 7× − × − × + × =

b) ( )9 9 9 :9 9 2001 2010 0+ × − + − × =

Subiectul II (26 puncte): AflaŃi valoarea lui a din egalitatea:

( )37 3 10 48 : 32 :8 7 6 52 : 4a− × + − × + = .

Subiectul III (24 puncte): Suma a trei numere este 510. Sǎ se afle numerele ştiind cǎ primul este de trei ori mai mare decât al doilea, iar al treilea este diferenŃa dintre primul şi al doilea. Subiectul IV (15 puncte): Un elev are la matematicǎ patru note care reprezintǎ patru numere impare consecutive cu proprietatea cǎ dacǎ le adunǎm cu dublul lor obŃinem numǎrul 72. Care sunt notele pe care le are elevul ? (Se considerǎ notǎ un numǎr de la 1 la 10, inclusiv).

Subiecte elaborate/ modificate/ selectate şi propuse de: Aurel Neicu, Gheorghe Oancea, Ioan Sǎcǎleanu şi Constantin Nastasǎ.

Page 24: De ce să învăŃăm la matematică? Word - 2011 Revista Mici matematicieni...coordonate, de axe ortogonale sau nu. Ceea ce înseamnă…matematică. Reintrând într-un registru

Micii MATEMATICIENI

24

PROIECTUL EDUCAłIONAL

« SUPER MATE » În orice domeniu ar activa, omul societăŃii contemporane trebuie să posede solide cunoştinŃe de matematică, să fie înarmat cu algoritmi şi scheme logico-matematice, menite să-i permită orientarea adecvată în lumea valorilor ştiinŃifice şi tehnologice, în stăpânirea limbajului ştiinŃelor care, după majoritatea estimărilor actuale, va fi matematizat şi informatizat. Matematica nu se învaŃă numai de specialişti. Până la un anumit nivel, ea face parte din cultura generală a oricărui om. Nu învăŃăm matematică pentru a şti pur şi simplu, ci pentru a o folosi şi a ne ajuta în practică. De aceea, este necesar ca elevii să dobândească, nu simplă instruire matematică, ci educaŃie matematică. Aceasta îşi propune şi Proiectul educaŃional SUPER MATE, care îşi desfăşoară activitatea la Şcoala ,,Petru Rareş” Hîrlău . El se adresează elevilor cu capacităŃi aptitudinale înalte, la disciplina Matematică, prin parcurgerea unui curriculum specific, diferenŃiat.

În cele două grupe de la clasele a III-a şi a IV-a sunt cuprinşi elevi de la şcolile: Hîrlău, Pîrcovaci, Deleni, Maxut, Poiana Deleni .

Rezultatele muncii cadrelor didactice şi ale elevilor se oglindesc în premiile obŃinute la concursurile şcolare de matematică : *Concursul ,,Micii matematicieni”organizat de Liceul ,,Ştefan cel Mare” Hîrlău: clasa a III-a : Tenchiu Teodosia (locul I); Iosub Marian (locul I); Deleanu Radu (locul I); Petrescu Bogdan (locul II); MusteaŃă Robert (locul III); Vicol Ştefan (MenŃiune); Bezedică Robert (M); Manelici Elena (M); Stângă Maria (M); Onofrei Tudor (M) . clasa a IV-a : Suruniuc Constantin (locul I); Puha Alexandru (locul I); Ciobanu Laura ( II ); Cotnăreanu Alexandru – locul II; Butunoi Bogdan – locul II; Buzămurgă Raluca – locul III; Luchian Andreea(M); Bortaş IonuŃ (M); Loghin Andrei (M); Cojocaru Cosmin (M); Habdulea Răzvan (M) *Concursul naŃional de matematică ,,Lumina Math” Cotnăreanu Alexandru – premiul II şi Ciobanu Laura Maria – menŃiune *Concursul naŃional ,,Micul matematician” Buzămurgă Raluca, Cotiugă Adelina, Cotnăreanu Alexandru, Petraş Iolanda ( 100 p) *Concursul ,,Cangurul”- au participat toŃi elevii cuprinşi în proiect.

*Din cei 28 de elevi admişi în clasa a V-a la Liceul Teoretic ,, Ştefan cel Mare” Hîrlău, 22 elevi au făcut parte din acest proiect.

Întreaga activitate a Proiectului EducaŃional ,,SUPER MATE” este prezentată în revista centrului nr. 6 - Hîrlău, care a obŃinut MenŃiune la Concursul NaŃional al revistelor şcolare. Cuprinde opinii ale cadrelor didactice, părinŃilor şi elevilor, dar şi exerciŃii şi probleme propuse de elevi şi cadre didactice. Datorită faptului că acest proiect se bucură de o mare apreciere în rândul elevilor, atât ei cât şi părinŃii şi cadrele didactice şi-au exprimat dorinŃa ca el să se deruleze şi în anii următori.

Responsabil centru nr. 6, Înv.Mirela Munteanu,

Şcoala “Petru Rareş”, Hîrlǎu

Page 25: De ce să învăŃăm la matematică? Word - 2011 Revista Mici matematicieni...coordonate, de axe ortogonale sau nu. Ceea ce înseamnă…matematică. Reintrând într-un registru

Micii MATEMATICIENI

25

CLUBUL MICILOR MATEMATICIENI

În anul 2002, s-a înfiinŃat Cercul elevilor “Pro-matematica”, schimbându-şi, în 2006 denumirea în “Clubul micilor matematicieni” având ca scop pregǎtirea elevilor liceului nostru pentru concursurile şcolare, cât şi implicarea lor în viaŃa matematicǎ actualǎ prin rezolvarea problemelor propuse în reviste de specialitate. Vom prezenta activitatea şi rezultatele obŃinute de unii membri ai cercului. 1. Pregǎtirea concursurilor şcolare prin rezolvarea problemelor propuse în diverse reviste. Astfel, unii membri ai cercului au apǎrut la Rubrica rezolvitorilor în revistele:

“RecreaŃii matematice”, Iaşi o Antoci Bogdan (2003-2004) Burican Bogdan Alexandru (2003, 2004,2007) Mihulcǎ

Lucian (2003) Pascariu Dragoş Marian (2003) Rotaru Lucian (2003-2005) Ciofu Alexandra (2004,2005) Sava Cristina Amelia (2004,2005) Scripcariu Gabriela (2004,2005) Spiridon Florin (2004,2005) Surugiu IonuŃ (2004) Manolie Ioan (2005) Zamfir Carmen Adela (2005) Onofrei Andrei CodruŃ (2005) Spînu Cosmin Alexandru (2005) ApachiŃei Ana-Maria (2006) AtârgoviŃoiei Anca-Elena (2006) Cojocaru Iulia (2006) Curcǎ Ioana (2006) LenŃer Sonia (2006) Pintilii Anda (2006) Buzilǎ Andreea (2007-2008) IvǎnuŃǎ Andreea Simona (2007-2008) Matei Mǎdǎlina (2007) Barǎu Larisa Ionela (2007,2008,2009) Ceucǎ Rǎzvan (2007,2008) Jitariu Adina Diana (2007,2008) Brânzǎ Carla (2008) Cǎlinescu Ana Maria (2008,2009) HuŃanu MǎǎlinaGeorgiana (2008) Loghin Bianca (2008) Mititelu Melisa (2008,2009) Neicu Mara (2008,2009) Sava Diana Alexandra (2008) Pintilii Alina (2008) Pletan Denisa Elena (2008) Bobîrnǎ Petru Costin (2009) Ruginǎ Rareş Teodor (2009) Sǎcǎleanu Emilian Gabriel (2009)

o Pentru trei la apariŃii Pagina rezolvitorilor, revista a premiat urmǎtorii elevi: Burican Bogdan (2007), Buzilǎ Andreea (2007) Neicu Mara (2008) IvǎnuŃǎ Andreea Simona (2008)

“Revista de Matematicǎ din Timişoara” Onofrei Andrei IonuŃ (3 şi 4/2005) Spînu Cosmin Alexandru (3 şi 4/2005) Rotaru Lucian(3 şi 4/2005) Pascariu Dragoş Marian (4/2005) Ciofu Alexandra (4/2005) Sava Cristina Amalia(4/2005) Scripcariu Gabriela (4/2005) Zamfir Carmen Adela (4/2005)

“Gazeta matematicǎ”, Bucureşti Bobîrnǎ Petru Cosmin (1+2/2009) Lǎcǎtuş Elena Alexandra (1/2009) Ruginǎ Rarreş Teodor (1/2009) Sǎcǎleanu Emilian Gabriel (1+2/2009) Cǎlinescu Ana Ioana (1/2009) Mititelu Melissa Florina (1/2009) Loghin Bianca (1/2009) Sava Diana Alexandra (1/2009) Scripcariu Gabriel (1/2009) Neicu Mara (1/2009; 7+8+9+10/2010)

Revista de matematicǎ “Alpha”, Craiova o Bobîrnă Costin Petru (2009,2010) Săcăleanu Emilian Gabriel (2009,2010) Brânză

Carla Gabriela (2009,2010) Buzilă Maria Bianca(2009,2010) Călinescu Ana Ioana (2009,2010) Alistar Ştefania Andreea (2009) Fotea Mădălina (2009) HuŃanu Mădălina Georgiana (2009,2010) Mititelu Melissa Florina (2009,2010) Neicu Mara (2009,2010) şi Sava Alexandra Diana (2009) Chifan Lavinia Georgiana (2009) Lǎcǎtuş Elena Alexandra (2009) Manilici Dumitru Andrei (2009) Murariu Maria (2009) Ruginǎ Rareş Teodor(2009)

o La “CONCURSUL REZOLVITORILOR” , nr. 1/2009 al revistei ALPHA, 4 elevi au obŃinut DIPLOMA DE CASTIGǍTOR: Bobîrnă Costin Petru, Săcăleanu Emilian Gabriel, Mititelu Melissa Florina şi Neicu Mara, iar Călinescu Ana Ioana a câştigat premiul 3 (30 RON). 2. Dezvoltarea potenŃialului creativ prin creearea de noi probleme. În nr. 1/2006 în “RecreaŃii matematice” elevului Onofrei CodruŃ i s-a publicat o problemǎ originalǎ. Pentru dezvolta potenŃialul creativ al elevilor şi pentru a avea ocazia sǎ propunǎ probleme originale s-a înfiinŃat concursul de creaŃie matematicǎ “Cea mai frumoasǎ problemǎ”. Printre elevii care au participat la concurs menŃionǎm: Buzilǎ Andreea (P2/2008), IvǎnuŃǎ Andreea Simona (M/2010+M/2009), Jitariu Adina Diana(M/2009). Onofrei Andrei CodruŃ(P1/2008),

Page 26: De ce să învăŃăm la matematică? Word - 2011 Revista Mici matematicieni...coordonate, de axe ortogonale sau nu. Ceea ce înseamnă…matematică. Reintrând într-un registru

Micii MATEMATICIENI

26

Pletan Denisa Elena(P1/2010+P1/2009), Alistar Ştefania Andreea, Brânzǎ Carla Gabriela, Buzilǎ Bianca, Creangǎ Daniela Simona (M/2009), Maticiuc Diana, Mititelu Melissa Florina. 3. Rezultate obŃinute la Concursuri (premii şi menŃiuni)

• OLIMPIADA NATIONALǍ DE MATEMATICǍ, ETAPA JUDETEANǍ 2002-2003: SCRIPCARIU GABRIELA (P2), SPIRIDON FLORIN (P3) SI DARIE RAMONA (P3). 2003-2004: SAVA

CRISTINA AMELIA (M). 2004-2005: PASCARIU DRAGOS MARIAN (M). 2005-2006: SAVA CRISTINA AMELIA

(M). 2006-2007: CEUCǍ RǍZVAN (M), IVǍNUTǍ ANDREEA SIMONA (M), CREANGǍ ARYNA (M) SI PUHǍ

RǍZVAN (M). 2007-2008: NEICU MARA (M) . 2008-2009: BOBIRNǍ PETRU COSTIN (M), HUTANU MǍDǍLINA

GEORGIANA (P3), NEICU MARA (M), CǍLINESCU ANA IOANA (M), MITITELU MELISSA FLORINA(M), SCRIPCARIU

GABRIEL(M), SAVA SUZANA(M). 2009-2010: COZMA ROXANA ELENA (M), RUGINǍ RARES THEODOR (M), PUHǍ RǍZVAN PETRU (M) SI PONOR COSMINA GEORGIANA(M).

• CONCURSUL NATIONAL DE MATEMATICǍ APLICATǍ “ADOLF HAIMOVICI” 2004-2005: POPOVICI LILIANA (M). 2005-2006: POPOVICI LILIANA (M). 2006-2007: TINCU IULIAN (M), ANGHEL ALINA GEORGIANA (M), IVǍNUłǍ SIMONA ANDREEA (M), PROCOVANU ADINA (M) ŞI RǍłOI ALINA

ELENA (M). 2007-2008: ZAPAN ANA MARIA (M), SPIRIDON FLORIN TUDOR (M). 2008-2009: SCRIPCARIU

GABRIELA (M), ANGHEL ALINA GEORGIANA (M) ŞI LUNGU MIHAELA (M). 2009-2010: DORCU IRINA LUCREłIA

(M), SPIRIDON FLORIN (P2 JUD. +M NAłIONALǍ) ŞI ANGHEL ALINA GEORGIANA(M). • CONCURSUL DE MATEMATICǍ “FLORICA T. CAMPAN”

2006-2007: IVǍNUTǍ ANDREEA SIMONA (M). 2008-2009: BOBIRNǍ PETRU COSTIN (M), RUGINǍ RARES

THEDOR(M), CIUBOTARU RALUCA (M), HUTANU MǍDǍLINA GEORGIANA (M) SI MITITELU MELISSA FLORINA(M). 2009-2010: CIUBUC REMUS (M), HUTANU MǍDǍLINA (M) SI MITITELU MELISSA FLORINA(M).

• CONCURSUL INTERJUDETEAN DE MATEMATICǍ “SPERANTE OLIMPICE”, PASCANI 2006-2007: IVǍNUTǍ ANDREEA SIMONA (P3), PLETAN DENISA (M), BUZILǍ ANDREEA (M), JITARIU ADINA

DIANA(M), SIRBU IOANA (P3), UNGUREANU CARMEN (M), PETRAS ROXANA (M) SI PINTILII ANDA(M). 2007-2008: HUłANU MǍDǍLINA GEORGIANA (M), COTIUGǍ MǍDǍLINA ANDREEA (M), MITITELU MELISSA

FLORINA (M), SCRIPCARIU GABRIEL (M), BARǍU LARISA IONELA (PS), IVǍNUłǍ ANDREEA SIMONA (PS), BUZILǍ

ANDREEA (M), JITARIU ADINA DIANA (M), PETRAŞ ROXANA (PS), SÎRBU IOANA (M) ŞI UNGUREANU CARMEN

ELENA (M). 2008-2009: BOBÎRNǍ COSTIN PETRU (P2), CHIFAN LAVINIA GEORGIANA (PS), CIUBOTARU RALUCA

(M), SǍCǍLEANU EMILIAN GABRIEL (M), RUGINǍ RAREŞ THEODOR (M), LǍCǍTUŞ ALEXANDRA (M), HUłANU

MǍDǍLINA GEORGIANA (P3), MITITELU MELISSA FLORINA (M), SCRIPCARIU GABRIEL (M), CǍLINESCU ANA

IOANA (M) ŞI IVǍNUłǍ ANDREEA SIMONA (P3). • CONCURSUL ZONAL “MICII MATEMATICIENI”, HÎRLǍU

2005-2006: ZAHARIA DENIS (P1), ŞTEFǍNESCU ALEXANDRU (P3),LOGHIN ELENA MǍDǍLINA (P2), CURCǍ

IOANA (P3), OPREA OTILIA (M), APACHIłEI ANA MARIA (M), ATUDOSIEI CLAUDIA (M), CIOFU ALEXANDRA (P1), SAVA AMELIA CRISTINA(P2) ŞI ZAMFIR CARMEN ADELA(M). 2006-2007: PLETAN DENISA ELENA (P2), IVǍNUłǍ

ANDREEA SIMONA (P3), CREANGǍ ARYNA ALEXANDRA (M), PUHǍ RǍZVAN PETRU (P1), BUZEMURGǍ MIHAELA

(P2), HANUŞ ŞTEFAN (M), DANDEA IULIA ALEXANDRA (M), BURSUC IOAN CIPRIAN (P1),PINTILII ANDA DUMITRELA

(P2), ŞI ŞERBAN GEORGIANA (M). 2007-2008: MITITELU MELISSA FLORINA (P2), HUłANU MǍDǍLINA

GEORGIANA (M), LOGHIN BIANCA (M), IFRIM RAREŞ CRISTIAN (M), NEICU MARA (M), SCRIPCARIU GABRIEL

(M), ALISTAR ANDREEA (M), ROMAN VLAD (M), PLETAN DENISA ELENA (M), PUHǍ RǍZVAN (P1), PETRAŞ

ROXANA (P2), DANDEA ALEXANDRA (M), SÎRBU IOANA (M) ŞI UNGUREANU CARMEN ELENA (M). 2008-2009: BOBÎRNǍ COSTIN PETRU (P1), CIUBOTARU RALUCA (P2), MURARIU MARIA (M), RUGINǍ RAREŞ THEODOR (M), MITITELU MELISSA FLORINA (P2), NEICU MARA (P3), HUłANU MǍDǍLINA GEORGIANA (M), LOGHIN BIANCA

(M), CǍLINESCU ANA IOANA (M), PORUŞNIUC ANDREI (M), IVǍNUłǍ ANDREEA SIMONA (P2), PINTILII ALINA

SÎNZÎIANA (P3), PRIGOREANU GABRIEL (P2), ŞTEFǍNESCU ALEXANDRU (P2), BUCUR-BRÂNZǍ BIANCA (M), BUZEMURGǍ GABRIELA (M), SCRIPCARIU RǍZVAN (M), ROMAN CODRIN (M), LUCEAC MǍDǍLINA (M) ŞI PETRARU ALEXANDRA (M). 2009-2010: CIUBUC REMUS MIHAIL (P3), COJOCARU ELISABETA (P3), CURCǍ

IULIAN (P3), CERNESCU BOGDAN FLORIN (M), SǍCǍLEANU EMILIAN GABRIEL (P2), MURARIU MARIA (P3), CǍLINESCU ANA IOANA (P1), SCRIPCARIU GABRIEL (P3), IFRIM RAREŞ (M) ŞI CEUCǍ RǍZVAN ŞTEFAN (M).

CATEDRA DE MATEMATICǍ A LICEULUI TEORETIC “ŞTEFAN CEL MARE”, HÎRLǍU

Page 27: De ce să învăŃăm la matematică? Word - 2011 Revista Mici matematicieni...coordonate, de axe ortogonale sau nu. Ceea ce înseamnă…matematică. Reintrând într-un registru

Micii MATEMATICIENI

27

PROBLEME ŞI SOLUłII

Această rubrică conŃine enunŃurile şi soluŃiile Problemelor propuse în numărul 4 al revistei “Micii matematicieni” din martie 2010.

MATEMATICA PITICĂ

P. 47 : JustificaŃi cǎ suma numerelor cuprinse între 121 şi 176 cu cifra unitǎŃilor 3 are ultima cifrǎ cel mai mare numǎr par.

Inst. Elena Nicolae, Şcoala Pleşoi, Dolj SoluŃie : Numerele cuprinse între 121 şi 176 cu cifra unitǎŃilor 3 sunt : 123 ; 133 ; 143 ; 153 ; 163 şi 173, a cǎror sumǎ este 888. Cum 8 este cea mai mare cifrǎ parǎ rezultǎ cerinŃa problemei. P. 48 : Un copil are un număr de jetoane cu animale. În fiecare lună el primeşte un număr de jetoane egal cu un sfert din câte avea. După 3 luni copilul avea 125 jetoane. Câte jetoane a avut la început?

Înv. Maria Ilie, Şcoala « Vasile Conta », Iaşi SoluŃie : Notǎm cu a , numǎrul iniŃial de jetoane, cu b , numǎrul de jetoane după prima lună, cu c din a doua lunǎ, iar cu d pe cel din a treia lunǎ. Avem 125d = şi : 4d c c= + . ÎnmulŃind cu 4 gǎsim cǎ 4 4 4 125 5 500 :5d c c c c⋅ = ⋅ + ⇒ ⋅ = ⋅ ⇒ = . Deci 100c = şi cum : 4c b b= + rezultǎ cǎ 4 4 4 100 5 400 :5c b b b b⋅ = ⋅ + ⇒ ⋅ = ⋅ ⇒ = , de unde 80b = . Dar, : 4b a a= + . ÎnmulŃind cu 4 obŃinem: 4 4 4 80 5 320 :5b a a a a⋅ = ⋅ + ⇒ ⋅ = ⋅ ⇒ = . Deci elevul a avut la început 64 jetoane. P. 49 : În anul 2010 o persoanǎ a împlinit 40 ani. Care este data de naştere a acelei persoane ştiind cǎ nu a putut sǎ-şi sǎrbǎtoreascǎ ziua de naştere în fiecare an, deşi ar fi vrut aceasta ?

Înv. Ramona Mihaela Adam, Şcoala Miroslava, Iaşi SoluŃie : Anul de naştere este 2010 40 1970− = . Cum persoana nu a putut sǎ-şi sǎrbǎtoreascǎ ziua de naştere în fiecare an rezultǎ cǎ s-a nǎscut pe 29 februarie. Nota redacŃiei : Problema are un « neajuns » şi anume, 2010 nu este an bisect (nu existǎ 29 februarie în acest an).

P. 50 : De 8 Martie, Florin a cumpărat pentru colegele de clasă zambile şi frezii. După ce fiecare fată a primit o zambilă şi două frezii, i-au mai rămas două zambile şi 12 frezii. Câte colege are Florin, dacă numărul zambilelor cumpărate a fost de 3 ori mai mic decât al freziilor?

Inst. Corneliu Constantin Ilie, Şcoala « Vasile Conta », Iaşi SoluŃie : Notǎm cu a , numǎrul colegilor, cu z , numǎrul zambilelor, iar cu f pe cel al freziilor. Din enunŃ se deduc relaŃiile : 3f z= ⋅ ; 2z a= + şi 2 12f a= ⋅ + . Din primele douǎ rezultǎ cǎ

( )3 2 3 6f a f a= ⋅ + ⇒ = ⋅ + . Înlocuind în a treia gǎsim 3 6 2 12 3 2 12 6a a a a⋅ + = ⋅ + ⇒ ⋅ = ⋅ + −

3 2 6 6 3 2 6 6a a a a a a⇒ ⋅ = ⋅ + ⇒ = ⋅ − ⋅ ⇒ = ⇒ = . Prin urmare , Florin are 6 colege. P. 51 : Mama are doi bǎieŃi gemeni. AflaŃi ce vârstǎ au bǎieŃii şi mama, ştiind cǎ împreunǎ cei trei au 60 de ani, iar vârsta mamei este de 3 ori mai mare decât vârsta unui bǎiat. Peste câŃi ani vârsta mamei va fi de 2 ori mai mare decât vârsta unui bǎiat ?

Inst. Constantina Dinu, Şcoala Castranova, Dolj SoluŃie : a) Fie m , vârsta mamei şi b , vârsta bǎieŃilor. Avem 60m b b+ + = şi 3m b= ⋅ , de unde 3 60b b b⋅ + + = , adicǎ 5 60b⋅ = . Deci, bǎieŃii au vârsta de 60 :5 12b = = ani, iar mama de 36 ani. b) Notǎm cu x numǎrul anilor cǎutat. Din enunŃ se obŃine ecuaŃia : ( )36 2 12x x+ = ⋅ + ⇒

Page 28: De ce să învăŃăm la matematică? Word - 2011 Revista Mici matematicieni...coordonate, de axe ortogonale sau nu. Ceea ce înseamnă…matematică. Reintrând într-un registru

Micii MATEMATICIENI

28

24 2 36x x+ ⋅ = + 2 36 24x x⇒ ⋅ = + − 2 12x x⇒ ⋅ = + 12 2 x x⇒ = ⋅ − 12 12x x⇒ = ⇒ = ani. Peste 12 ani, vârsta mamei va fi de 2 ori mai mare decât vârsta unui bǎiat.

P. 52 : Din cel mai mic număr de 4 cifre diferite scad triplul numărului 125. La rezultat adun sfertul numărului 840, din noul rezultat scad dublul unui număr, împart rezultatul obŃinut la 5 şi obŃin câtul 60. La ce număr m-am gândit?

Înv. Mariana Chelaru, Şcoala „Petru Rareş”, Hîrlău SoluŃie : Cel mai mic numǎr de 4 cifre distincte este 1023. Notǎm numǎrul cǎutat cu a . Se

obŃine: ( ){ }1023 3 125 840 : 4 2 :5 60a− ⋅ + − ⋅ = ( )1023 375 210 2 60 5a⇔ − + − ⋅ = ⋅ , de unde

[ ]648 210 2 300 858 2 300 2 858 300 558 : 2 279a a a a a⇔ + − ⋅ = ⇔ − ⋅ = ⇔ ⋅ = − ⇔ = ⇔ = .

P. 53 : Elena şi Corina au un numǎr de timbre. Dacǎ Elena i-ar da Corinei 8 timbre, ele ar avea acelaşi numǎr de timbre. Dacǎ Corina i-ar da Elenei 8 timbre, atunci Elena ar avea de 5 ori mai multe timbre decât Corina. Câte timbre are fiecare fatǎ ?

Inst. Ovidiu-Constantin Munteanu, Şcoala MoŃǎŃei, Dolj SoluŃie : Notǎm cu E şi cu C , numǎrul de timbre ale Elenei, respectiv ale Corinei. Din enunŃ deducem egalitǎŃile : 8 8E C− = + şi ( )8 5 8E C+ = ⋅ − . Din prima gǎsim cǎ 16E C= + , pe care

o înlocuim în a doua obŃinând : 16 8 5 5 8 24 5 40 5 24 40C C C C C C+ + = ⋅ − ⋅ ⇔ + = − ⇔ = + + 5 64 64 5 64 4 64 : 4 16C C C C C C C⇔ = + ⇔ = − ⇔ = ⇔ = ⇔ = , iar 16 16 32E = + = .

Prin urmare, Corina are 16 timbre, iar Elena 32 timbre.

P. 54 : De Paşti, bunica a vopsit ouă roşii, galbene şi verzi. Ştiind că numai 35 nu sunt roşii, numai 49 nu sunt verzi , iar numărul celor verzi este de 2 ori mai mic decât numărul celor roşii, aflaŃi câte ouă de fiecare fel a vopsit bunica.

Înv. Maria CreŃu, Şcoala „Petru Rareş”, Hîrlău SoluŃie : Scǎzând egalitǎŃile 49r g+ = şi 35v g+ = se obŃine 14r v− = . łinând cont cǎ

2r v= ⋅ gǎsim cǎ 2 14v v− = ⇒ 14v = ouǎ verzi, de unde 2 14 28r = ⋅ = ouǎ roşii, iar 28 49g+ = , adicǎ 49 28 21g = − = ouǎ galbene.

P. 55 : În timp ce Rareş mănâncă douǎ piersici, Mateea trei şi Teodor zece. Câte piersici revin fiecăruia dacă împreună au 60 piersici ?

Înv. Maria-Tereza Rugină, Şcoala “Petru Rareş”, Hîrlău SoluŃie : Planul logic al rezolvǎrii este urmǎtorul : � Câte piersici mǎnâncǎ împreunǎ într-un anumit interval de timp ?

� Rareş: 2 ; Mateea: 3 şi Teodor: 10. În total 15 piersici. � În câte intervale de timp mǎnâncǎ împreunǎ cele 60 piersici ?

� 15 se cuprinde în 60 de 4 ori ( 60 :15 4= ). � Câte piersici revine fiecǎruia ?

� Rareş: 4 2 8× = ; Mateea: 4 3 12× = , iar Teodor “ mâncǎciosul”: 4 10 40× = piersici.

P. 56 : ArătaŃi că : ( ) ( ){ }5 : 2 3 1 14 :15 3: 1 2 45 :15 2 : 2 2008 2009⋅ − + + + ⋅ − + = .

EliminaŃi o cifrǎ din egalitatea precedentǎ astfel încât ea sǎ rǎmânǎ adevǎratǎ. Prof. Nicolae Ivăşchescu, Colegiul NaŃional ” FraŃii Buzeşti”, Craiova

SoluŃie : Calculând , se obŃine: ( ) ( ){ }5 : 6 1 14 :15 3: 1 2 3 2 : 2 2008 2009− + + + ⋅ − + =

[ ] [ ]{ }5 :5 14 :15 3: 1 2 1 : 2 2008 2009⇔ + + + ⋅ + = [ ] [ ]{ }1 14 :15 3: 1 2 : 2 2008 2009⇔ + + + + =

Page 29: De ce să învăŃăm la matematică? Word - 2011 Revista Mici matematicieni...coordonate, de axe ortogonale sau nu. Ceea ce înseamnă…matematică. Reintrând într-un registru

Micii MATEMATICIENI

29

{ }15 :15 3:3 : 2 2008 2009⇔ + + = { }1 1 : 2 2008 2009⇔ + + = 2 : 2 2008 2009⇔ + =

1 2008 2009⇔ + = 2009 2009⇔ = , evident adevǎratǎ. Prin eliminarea unei cifre se înŃelege ştergerea ei oriunde apare în egalitate. Eliminǎm cifra 0 .

Avem ( ) ( ){ }5 : 2 3 1 14 :15 3: 1 2 45 :15 2 : 2 28 29⋅ − + + + ⋅ − + = . Cum acolada are valoarea 2

se verificǎ uşor adevǎrul egalitǎŃii obŃinute. P. 57 : În prima zi de luni din ianuarie 2009, Emi îşi activeazǎ cǎsuŃa de e-mail. Lunea, marŃea, miercurea şi joia primeşte câte 6 mesaje pe zi, iar vinerea şi sâmbǎta câte 12 mesaje pe zi. Duminica le citeşte pe toate şi şterge jumǎtate din numǎrul lor. Dacǎ în fiecare duminicǎ se întâmplǎ la fel, câte mesaje vor fi în cǎsuŃa de e-mail pe 31 ianuarie 2009 la ora 23:59 ?

Prof. Ioan Sǎcǎleanu, Liceul ” Ştefan cel Mare”, Hirlău SoluŃie : În fiecare sǎptǎmânǎ, Emi primeşte 6 4 2 12 48⋅ + ⋅ = mesaje. În dumineca din 11 ianuarie din cele 48 şterge 24 mesaje. În a doua duminecǎ din 24 48 72+ = mesaje şterge 36, iar în ultima duminecǎ din 36 48 84+ = rǎmâne cu 42 mesaje. Cum în ultima sǎptǎmânǎ mai primeşte 48 mesaje pe care nu le şterge rezultǎ cǎ pe 31 ianuarie 2009 (sâmbǎtǎ), ora 23 :59 în cǎsuŃa sa de e-mail vor fi 42 48 90+ = mesaje. P. 58 : Lupul o urmăreşte pe ScufiŃa Roşie: aceasta are 15 paşi înaintea lui. Câte sărituri va face lupul ca să o ajungă, dacă în timp ce el face 2 sărituri, ea face 3 paşi, iar 2 sărituri de-ale lupului fac cât 5 paşi de-ai ScufiŃei Roşii?

Înv. Mirela Munteanu, Şcoala “Petru Rareş”, Hirlău SoluŃie : În timp ce lupul face 2 sǎrituri, adicǎ 5 paşi de-ai ScufiŃei , ea face 3 paşi. Prin urmare din 2 sǎrituri lupul recupereazǎ 2 paşi. Deducem cǎ la o sǎriturǎ a Lupului el recupereazǎ 1 pas. Deoarece el trebuie sǎ recupereze 15 paşi (« avansul » ScufiŃei) rezultǎ cǎ Lupul va face 15 sǎrituri pentru a o ajunge pe ScufiŃǎ.

P. 59 : Ana o întreabă pe Corina ce vârstă are mama sa. Corina răspunde: „ – Dacă la sfertul anilor ei vei adăuga 4, atunci vei afla câŃi ani a avut cu 20 de ani în urmă”. CâŃi ani are mama Corinei?

Înv. Maria CreŃu, Şcoala „Petru Rareş”, Hîrlău SoluŃie : Notǎm cu m , vârsta mamei. Dacǎ considerǎm segmentul-parte ca fiind sfertul lui m şi anume : 4m p= atunci 4m p= × şi avem 4 4 20p p+ = × − , de unde 4 4 20p p× = + + ⇔ 4 24 3 24 8p p p p× − = ⇔ × = ⇔ = . Vârsta mamei Corinei este 4 8 32m = ⋅ = ani. P. 60 : În timp ce cei 25 elevi ai unei clase urcau pe munte în şir indian, Radu – matematicianul clasei – exclamă „ Am în faŃa mea trei sferturi din totalul colegilor mei!” Al câtelea în şir este Radu?

Înv. Marieta Muşei, Şcoala „Petru Rareş”, Hîrlǎu SoluŃie : Radu are 25 1 24− = colegi. Un sfert dintre colegi reprezintǎ 24 : 4 6= , iar trei sferturi cât are în faŃa lui înseamnǎ 3 6 18⋅ = colegi. Prin urmare Radu ocupǎ al 19-lea loc în şir.

P. 61 : Cei şase membri ai unui echipaj care participă la un concurs de matematica au vârste diferite, de cel puŃin 10 ani şi cel mult 15 ani. În timpul concursului s-au aşezat la o masă în ordinea vârstelor. Să se afle ce vârstă are fiecare ştiind că Ioana este cea mai mică,Anca este cea mai mare, Bogdan se află lângă Ioana şi nu se află lângă Bianca şi Andrei şi că Alexandra se află între doi băieŃi.

Prof.ConstanŃa Tudorache, Şcoala ”St. Bârsănescu”, Iaşi

Page 30: De ce să învăŃăm la matematică? Word - 2011 Revista Mici matematicieni...coordonate, de axe ortogonale sau nu. Ceea ce înseamnă…matematică. Reintrând într-un registru

Micii MATEMATICIENI

30

SoluŃie : Vârstele celor şase membri ai echipajului pot fi 10, 11, 12, 13, 14 sau 15. Fiind cea mai micǎ, Ioana are 10 ani. Cum lângǎ Ioana este un singur loc, acesta este ocupat de Bogdan , deci el are 11 ani. Anca are 15 ani cǎci este cea mai mare. Bogdan nu se aflǎ lângǎ Bianca şi Andrei rezultǎ cǎ lângǎ Bogdan se aflǎ Alexandra. Ca urmare ea are 12 ani. Cum Alexandra se aflǎ între 2 bǎieŃi atunci locul de lângǎ ea, cel de 13 ani va fi ocupat de Andrei . Rǎmâne Bianca de 14 ani. P. 62 : Alba ca Zǎpada şi cei şapte pitici au suma vârstelor 172 ani. Ştiind cǎ Alba ca Zǎpada este cea mai tânǎrǎ dintre ei şi cǎ vârstele piticilor sunt numere naturale consecutive, aflaŃi vârsta fiecǎruia.

Înv. ConstanŃa Dumitriu, Şcoala “Petru Rareş”, Hîrlǎu SoluŃia autorului : Luând vârsta Albei-ca-Zǎpada ca segment-parte p atunci vârsta primului pitic este 1p + , al doilea 2p + , ... , al 7-lea este 7p + . Atunci se obŃine 8 1 2 ... 7 172p× + + + + = , de unde 8 172 28 144 :8 18p p p× = − ⇔ = ⇔ = . Prin urmare Alba-ca-Zǎpada are 18 ani, primul pitic are 19, al doilea 20,..., ultimul pitic 25 ani. Nota redacŃiei : Citind cu atenŃie enunŃul problemei, observǎm cǎ vârstele celor 8 protagonişti nu sunt numere consecutive, ci numai vârstele piticilor. Vom rezolva problema în acest sens. Notǎm cu a , vârsta Albei-ca-Zǎpada şi cu p , vârsta celui mai mic pitic. EnunŃul ne dǎ

egalitatea : ( ) ( ) ( )1 2 ... 6 172a p p p p+ + + + + + + + =

( )7 1 2 .. 6 172a p⇔ + ⋅ + + + + = 7 151p a⇔ ⋅ + = ( )1 . ÎmpǎrŃind pe 151 la 7 , se obŃine câtul

21c = şi restul 4r = . Cum a p< ( )2 , analizǎm situaŃiile :

� Dacǎ 0 7a≤ < atunci 4r a= = şi 21c p= = , din unicitatea câtului şi a restului unei împǎrŃiri.

� Dacǎ 7 14a≤ < şi cum ( )1 devine : ( ) ( )151 7 1 7p a= ⋅ + + − şi cum 0 7 7a≤ − < , atunci

7 4 11r a a= − = ⇒ = şi 1 21 20c p p= + = ⇒ = , din unicitatea câtului şi a restului unei împǎrŃiri. � Dacǎ 14 21a≤ < şi cum ( )1 devine : ( ) ( )151 7 2 14p a= ⋅ + + − şi cum 0 14 7a≤ − < , atunci

14 4 18r a a= − = ⇒ = şi 2 21 19c p p= + = ⇒ = , din unicitatea câtului şi a restului unei împǎrŃiri. � Dacǎ 21a ≥ atunci, din ( )2 avem 22p ≥ , de unde 7 154p⋅ ≥ 7 154 154p a a⋅ + ≥ + > de

unde ( )1151 154⇒ > , absurd. Prin urmare, problema are trei soluŃii şi anume :

1. Alba-ca-Zǎpada are 4 ani şi piticii : 21, 22, ... , 27 ; 2. Alba-ca-Zǎpada are 11 ani şi piticii : 20, 21, ... , 26 ; 3. Alba-ca-Zǎpada are 18 ani şi piticii : 19, 20, ... , 25, soluŃie gǎsitǎ şi de autoare.

P. 63 : Mihai şi Dan au primit împreună de la bunica 340 de lei. Dacă împărŃim jumǎtatea sumei lui Mihai la şesimea sumei lui Dan obŃinem câtul 5 şi restul 2. CâŃi lei are fiecare ?

Prof. Petru Asaftei, Şcoala Normalǎ “Vasile Lupu”, Iaşi SoluŃia 1 (aritmetic) : Din împǎrŃirea cu rest obŃinem cǎ jumǎtatea sumei lui Mihai reprezintǎ 5 şesimi din suma lui Dan şi încǎ 2 lei. Prin urmare întreaga sumǎ a lui Mihai reprezintǎ 10 şesimi din suma lui Dan şi încǎ 4 lei. Aşadar, suma primitǎ de cei doi copii reprezintǎ 16 şesimi din suma lui Dan şi încǎ 4 lei. Înseamnǎ cǎ 16 şesimi din suma lui Dan reprezintǎ 340 4 336− = , iar 2 şesimi 336 :8 42= lei. În concluzie, Dan are 42 3 126× = lei, iar Mihai 340 126 214− = lei.

Page 31: De ce să învăŃăm la matematică? Word - 2011 Revista Mici matematicieni...coordonate, de axe ortogonale sau nu. Ceea ce înseamnă…matematică. Reintrând într-un registru

Micii MATEMATICIENI

31

SoluŃia 2 (algebric) : Notând cu a , jumǎtatea sumei lui Mihai şi cu b , şeimea sumei lui Dan obŃinem relaŃiile : 2 6 340a b⋅ + ⋅ = şi 5 2a b= ⋅ + , de unde 3 170 5 2 3 170a b b b+ = ⇔ + + =

8 168 21b b⇔ = ⇔ = . Prin urmare, Dan are 6 21 126⋅ = lei, iar Mihai 340 126 214− = lei.

P. 64 : Un numǎr natural de trei cifre împǎrŃit la 3 dǎ drept cât rǎsturnatul sǎu, restul un numǎr impar. Care este numǎrul ?

Inst. Marian Ciuperceanu, Colegiul NaŃional ”FraŃii Buzeşti”, Craiova SoluŃie : Fie abc numǎrul cǎutat. Avem 3 1abc cba= ⋅ + cǎci restul împǎrŃirii este un numǎr

impar mai mic decât 3. Descompunând în baza zece obŃinem : ( )00 0 3 00 0 1a b c c b a+ + = ⋅ + + +

100 10 300 30 3 1a b c c b a⇔ ⋅ + ⋅ + = ⋅ + ⋅ + ⋅ + 97 299 20 1a c b⇔ ⋅ = ⋅ + ⋅ + ( )∗ . Se observǎ cǎ

299 97c a⋅ ≤ ⋅ şi 97 97 9a⋅ ≤ ⋅ rezultǎ cǎ 299 873c⋅ ≤ , de unde c poate fi 1 sau 2. Dacǎ 1c = atunci 97 300 20a b⋅ = + ⋅ . Cum cifra unitǎŃilor numǎrului 300 20 b+ ⋅ este 0 gǎsim cǎ şi 97 a⋅ are cifra unitǎŃilor 0, fapt imposibil cǎci nu existǎ cifra nenulǎ a care înmulŃitǎ cu 7 sǎ se termine în cifra 0 ( ci în 7,4,1,8,5,2,9, 6,3). Prin urmare 2.c = Rezultǎ din ( )∗ cǎ 97 599 20a b⋅ = + ⋅ .

Cum 599 20 b+ ⋅ are ultima cifrǎ 9 rezultǎ cǎ 97 a⋅ are ultima cifrǎ 9, de unde 7a = , iar 97 7 599 20 679 599 20 20 80b b b⋅ = + ⋅ ⇔ = + ⋅ ⇔ ⋅ = ne dǎ 4b = . Numǎrul cǎutat este 742 .

P. 65 : Într-o familie sunt 8 copii. Ei au economisit sume egale de bani. Fiind ziua de naştere a mamei, ei au pus câte un sfert din sumele ce le aveau, iar tatăl 100 de lei şi-i cumpără un cadou de 500 lei. La ziua de naştere a tatălui, ei, copiii contribuie cu o treime din sumele ce le rămăseseră, iar mama cu 100 de lei şi-i cumpără un cadou.

Cât a costat cadoul pentru tatăl copiilor? Înv. Maria Simionescu, Şcoala „Petru Rareş”, Hîrlǎu

SoluŃia 1 : Planul logic al rezolvǎrii : � Cu cât a contribuit împreunǎ cei 8 copii la cadoul mamei ? :Raspuns→ 500 100 400− = lei � Care este suma economisitǎ de cei 8 copii ? :Raspuns→ 400 4 1600× = lei � Cu ce sumǎ au mai rǎmas cei 8 copii dupǎ ziua mamei ? :Raspuns→ 1600 400 1200− = lei � Care este contribuŃia celor 8 copii la cadoul tatei ? :Raspuns→ 1200 : 3 400= lei � Cât a costat cadoul pentru tatăl copiilor? :Raspuns→ 400 100 500+ = lei. SoluŃia 2 : Vom nota cu S , suma strânsǎ împreunǎ de copii pentru cadoul mamei. Atunci suma totalǎ economisitǎ de ei este 4 S⋅ , de unde rezultǎ cǎ suma rǎmasǎ dupǎ ziua mamei este 4 3S S S⋅ − = ⋅ . ContribuŃia copiilor la cadoul tatei este 3 : 3S S⋅ = . Se observǎ cǎ copii contribuie cu aceeaşi sumǎ atât la cadoul mamei, cât şi la cadoul tatei. Cum şi pǎrinŃii au aceeaşi contribuŃie (de 100 lei) rezultǎ cǎ valoarea cadoului tatei este cât al mamei, adicǎ 500 lei. Nota redacŃiei : Dupǎ cum se observǎ din soluŃia 2, nu este necesar ca datele problemei sǎ conŃinǎ informaŃii despre numǎrul copiilor familiei (8), nici cǎ ei au economisit „sume egale de bani” şi chiar de valoarea contribuŃiei fiecǎrui pǎrinte la cadoul celuilalt (de 100 lei).

P. 66 : În anul 2010, Ana şi Mihai aveau vârstele 18 ani, respectiv 12 ani. În ce an vor avea împreună un secol ?

Înv. Maria Ilie şi Inst. Corneliu Constantin Ilie, Şcoala ,,V. Conta’’ Corp B, Iaşi SoluŃia 1 : Notǎm cu a , vârsta Anei şi cu m , vârsta lui Mihai atunci când împreunǎ vor avea un secol. Avem 18 12a m− = − , de unde 6a m= + . Dar 100 6 100a m m m+ = ⇒ + + = , de unde

2 94m⇒ ⋅ = . Deci Mihai va avea 47 ani, iar Ana 53 ani. Vârsta fiecǎruia s-a mǎrit cu 53 18 35− = ani ( sau 47 12 35− = ). Prin urmare vor avea împreunǎ un secol în anul 2045. SoluŃia 2 : În anul 2010, Ana şi Mihai au împreunǎ 18 12 30+ = ani. Pentru a avea împreunǎ un secol ar trebui sǎ mai trǎiascǎ amândoi încǎ 100 30 70− = , iar fiecare încǎ 70 : 2 35= ani.

Page 32: De ce să învăŃăm la matematică? Word - 2011 Revista Mici matematicieni...coordonate, de axe ortogonale sau nu. Ceea ce înseamnă…matematică. Reintrând într-un registru

Micii MATEMATICIENI

32

Prin urmare în anul 2045 ( )2010 35= + , cei doi vor avea împreunǎ un secol.

MATEMATICA GIMNAZIALĂ Clasa a V-a

5.38 : La o împǎrŃire, deîmpǎrŃitul este de 3 ori mai mare decât câtul, iar câtul este cu 142 mai mare decât împǎrŃitorul. ReconstituiŃi operaŃia !

Înv. Vioricǎ Ceauşescu, Şcoala Motru, Gorj SoluŃie : Notǎm cu d , cu i , cu c , cu r , deîmpǎrŃitul, împǎrŃitorul, câtul, respectiv restul împǎrŃirii. Avem : 3d c= ⋅ şi 142c i= + . Din teorema împǎrŃirii cu rest obŃinem d i c r= ⋅ + şi r i< . Eliminând deîmpǎrŃitul obŃinem cǎ ( )3 3 3c i c r c i c r c i r⋅ = ⋅ + ⇔ ⋅ − ⋅ = ⇔ ⋅ − = ( )∗ .

Rezultǎ cǎ { }3 0 3 0,1, 2,3i i i− ≥ ⇒ ≤ ⇒ ∈ . Din ( )∗ deducem cǎ 3 i r− ≤ , dar r i< , de unde

3 3 2 2i i i i− < ⇒ < ⋅ ⇒ ≥ . Deci { }2;3i∈ . Dacǎ 2i = atunci 142 144c i= + = şi din ( )∗ gǎsim

cǎ ( )144 3 2 144r r⋅ − = ⇒ = în contradicŃie cu r i< , adicǎ 144 2< , falsǎ.

Dacǎ 3i = atunci 142 145c i= + = şi din ( )∗ gǎsim cǎ ( )145 3 3 0r r⋅ − = ⇒ = , iar 435d = .

Astfel s-a reconstituit împǎrŃirea, adicǎ s-au determinat toŃi termenii operaŃiei. Nota redacŃiei : MulŃi dintre rezolvitori au ajuns la rezultatul corect, dar au considerat ( chiar şi autoarea) împǎrŃirea ca fiind exactǎ (rest nul), vǎduvind astfel rezolvarea de o parte importantǎ a sa.

5.39 : AflaŃi cubul perfect abc ştiind că diferenŃa dintre numărul format de primele două cifre şi numărul format din ultimele două cifre este 5.

Prof. Aurel Neicu, Liceul ” Ştefan cel Mare”, Hirlău SoluŃie : Cuburile de 3 cifre sunt : ( )3125 5= ; ( )3216 6= ; ( )3343 7= ; ( )3512 8= şi ( )3729 9= .

o Dacǎ 125abc = atunci 12 25ab bc− = − , nu este numǎr natural .

o Dacǎ 216abc = atunci 21 16 5ab bc− = − = , este soluŃie a problemei .

o Dacǎ 343abc = atunci 34 43ab bc− = − , nu este numǎr natural .

o Dacǎ 512abc = atunci 51 12 39 5ab bc− = − = ≠ , nu este soluŃie a problemei.

o Dacǎ 729abc = atunci 72 29 43 5ab bc− = − = ≠ , nu este soluŃie a problemei. Prin urmare singurul numǎr care verificǎ condiŃiile problemei este 216.

5.40 : Fie egalitatea : 284xy cy xc yx x+ + + + = . DemonstraŃi că : cifrele y şi c sunt

consecutive dacă şi numai dacă numerele 2 2x c+ şi 2y sunt consecutive. Prof. Ioan RăuŃu, Şcoala “Petru Rareş”, Hirlău

SoluŃie : Descompunerea în baza 10 ne dǎ 10 10 10 10 284x y c y x c y x x+ + + + + + + + = , adicǎ

22 12 11 284x y c+ + = 22 11 11 275 9x y c y⇔ + + − = − ( )11 2 25 9x y c y⇔ ⋅ + + − = − , de unde

deducem cǎ cifra 119 y− ∈Μ . ObŃinem 9y = . Rezultǎ cǎ 2 25 0 2 16x y c x c+ + − = ⇔ + = , de

unde gǎsim cǎ perechile ( ) ( ) ( ) ( ) ( ){ }, 8;4 , 6;5 , 4;6 , 2;7c x ∈ { }2 2 80;61;52;53x c⇒ + ∈ ( )∗ .

" ":⇒ Dacǎ cifrele y şi c sunt consecutive şi cum 9y = rezultǎ, din ( )∗ cǎ 8c = şi 2 2 80x c+ = Cum 2 81y = deducem cǎ numerele 2 2x c+ şi 2y sunt consecutive.

Page 33: De ce să învăŃăm la matematică? Word - 2011 Revista Mici matematicieni...coordonate, de axe ortogonale sau nu. Ceea ce înseamnă…matematică. Reintrând într-un registru

Micii MATEMATICIENI

33

" ":⇐ Dacǎ numerele 2 2x c+ şi 2y sunt consecutive şi cum 2 81y = rezultǎ cǎ

{ }2 2 80;82x c+ = . Din ( )∗ avem cǎ 2 2 80x c+ = pentru ( ) ( ), 8;4 8c x c= ⇒ = . Deci cifrele y

şi c sunt consecutive.

5.41 : a) CalculaŃi suma primilor 100 de termeni ai şirului : 1;9;61;369;2101;... .

b) DemonstraŃi cǎ numǎrul 2010 20113 5 4 1a = ⋅ − + este divizibil cu 12 . Leonard Vîrlan, elev, Liceul “Tudor Arghezi”, Craiova

SoluŃie :a) Se observǎ cǎ 1 11 5 4= − ; 2 29 5 4= − ; 3 361 5 4= − ; 4 4369 5 4= − ; .... . Notând cu

na al n-lea termen al şirului dat, gǎsim urmǎtoare regulǎ de formare a termenilor 5 4n nna = − .

Suma primilor 100 termeni ai şirului dat este ( ) ( )1 2 100 1 2 1005 5 ... 5 4 4 ... 4S = + + + − + + + .

Folosind formula 1

2 3 ...1

nn x x

x x x xx

+ −+ + + + =

− (demonstraŃia cred cǎ e bine cunoscutǎ) obŃinem

suma cerutǎ 101 1015 5 4 4

4 3S

− −= −

101 1013 5 15 4 4 16

12S

⋅ − − ⋅ +⇒ =

101 1023 5 4 1

12S

⋅ − +⇒ = .

b) Suma primilor 2009 termeni ai şirului de la a) este un numǎr natural. Pe de altǎ parte, aceastǎ

sumǎ este egalǎ cu 2010 2010 2010 20105 5 4 4 3 5 15 4 4 16

4 3 12

− − ⋅ − − ⋅ +− = . Deci

2010 20113 5 4 1

12

⋅ − +∈ℕ .

Deducem cǎ 2010 20113 5 4 1a = ⋅ − + este divizibil cu 12 . 5.42 : Să se determine cel mai mic număr natural m având proprietatea că între m şi 2m

există exact trei numere prime. Prof. Mircea Popa, Şcoala “Petru Rareş”, Hirlău

SoluŃia 1(a redacŃiei) :. Numerele prime sunt : 2, 3, 5, 7, 11, 13, 17, 19, 23, ... . Ideea rezolvǎrii este sǎ determinǎm m∈ℕ astfel încât 3 numere prime consecutive sǎ se afle între m şi 2m . � Dacǎ 2 3 5 2m m< < < < atunci 5 2 3m m< ⇒ ≥ , dar 2 3m < < , imposibil. Deci m∈∅ . � Dacǎ 3 5 7 2m m< < < < atunci 7 2 4m m< ⇒ ≥ , dar 3 4m < < , imposibil. Deci m∈∅ . � Dacǎ 5 7 11 2m m< < < < atunci 11 2 6m m< ⇒ ≥ , dar 5 6m < < , imposibil. Deci m∈∅ . � Dacǎ 7 11 13 2m m< < < < atunci 13 2 7m m< ⇒ ≥ , dar 7m < , imposibil. Deci m∈∅ . � Dacǎ 11 13 17 2m m< < < < atunci 17 2 9m m< ⇒ ≥ şi cum 11m < rezultǎ cǎ { }9,10m∈

Pentru 9m = avem 9 11 13 17 18< < > < , aşadar între 9 şi 18 existǎ 3 numere prime şi cum m∈ℕ trebuie sǎ fie minim rezultǎ cǎ 9m = este soluŃia problemei. SoluŃia 2 : Organizând rezolvarea problemei în tabelul :

m între m şi 2m avem numerele Câte numere prime sunt ?

0 Între 0 şi 0 : ----- 0 1 Între 1 şi 2 : ----- 0 2 Între 2 şi 4 : 3. 1 3 Între 3 şi 6 : 4; 5 1 4 Între 4 şi 8 : 5; 6; 7 2 5 Între 5 şi 10 : 6; 7; 8; 9 1 6 Între 6 şi 12 : 7; 8; 9; 10; 11 2 7 Între 7 şi 14 : 8; 9; 10; 11; 12; 13 2 8 Între 8 şi 16 : 9; 10; 11; 12; 13; 14; 15 2 9 Între 9 şi 18 : 10; 11; 12; 13; 14; 15; 16; 17 3

deducem cǎ 9m = este cel mai mic numǎr cu proprietatea din enunŃ

Page 34: De ce să învăŃăm la matematică? Word - 2011 Revista Mici matematicieni...coordonate, de axe ortogonale sau nu. Ceea ce înseamnă…matematică. Reintrând într-un registru

Micii MATEMATICIENI

34

5.43 : Suma dintre un număr prim şi un număr impar este 2011. AflaŃi numerele.

Prof. Nicolae Ivăşchescu, Colegiul NaŃional ” FraŃii Buzeşti”, Craiova SoluŃie : Numǎrul prim este egal cu diferenŃa dintre 2011 şi un numǎr impar, adicǎ dintre douǎ numere impare, deci un numǎr par. Cum singurul numǎr prim şi par este 2 rezultǎ soluŃia problemei numerele 2 şi 2009. 5.44 : Să se determine numărul natural de două cifre care prin împărŃirea la suma cifrelor sale dă câtul 7 şi restul 3 , iar prin împărŃirea la diferenŃa cifrelor sale dă câtul 18 şi restul 1 .

Prof. Gheorghe Spiridon, Şcoala ,, N. Iorga’’, Paşcani SoluŃie : Fie ab , numǎrul cǎutat. Avem ( )7 3ab a b= ⋅ + + şi ( )18 1ab a b= ⋅ − + . Din a doua

egalitate rezultǎ cǎ ab este numǎr impar, adicǎ { }1,3,5,7,9b∈ , iar din prima obŃinem cǎ

:310 7 7 3 10 7 7 3 3 6 3 2 1a b a b a a b b a b a b+ = + + ⇔ − = + − ⇔ = + ⇔ = + şi cum a este cifrǎ

rezultǎ cǎ { }1,3b∈ . Dintre { }31,73ab∈ numai 73 verificǎ şi a doua egalitate.

5.45 : Ziua şi luna de naştere a unui copil sunt douǎ numere rǎsturnate distincte, de douǎ cifre. Câtul împǎrŃirii anului de naştere la ziua de naştere este 95, iar restul este egal cu numǎrul reprezentând luna. AflaŃi data de naştere a copilului.

Prof. Ioan Sǎcǎleanu, Liceul ” Ştefan cel Mare”, Hirlău SoluŃie : Cum numerele sunt rǎsturnate distincte şi de douǎ cifre şi luna este un numǎr mai mic egal cu 12 rezultǎ cǎ luna este 12, iar ziua 21. Din teorema împǎrŃirii cu rest rezultǎ cǎ anul se obŃine din 21 95 12 2007⋅ + = . În concluzie, data de naştere a copilului este 21 decembrie 2007.

5.46 : Să se determine numărul prim ab ştiind că ( )3ab ba a b− + − este un pătrat perfect.

Prof. Gheorghe Oancea, Liceul ” Ştefan cel Mare”, Hirlău SoluŃia 1 : Avem : ( ) ( )23 10 10 3 3 12 12 2 3ab ba a b a b b a a b a b a b− + − = + − − + − = − = ⋅ ⋅ − .

Pentru a fi pǎtrat perfect trebuie ca 23a b k− = ⋅ , dar 9a b a− ≤ ≤ , de unde { }2 3 0,1k k≤ ⇒ ∈ .

Dacǎ 0k = atunci a b ab aa= ⇒ = este numǎr prim numai pentru 1a = . Dacǎ 1k = atunci

3a b− = . Deoarece ab este prim deducem cǎ el este impar cǎci 2 este singurul numǎr prim şi

par. Deci b este impar şi cum 6b ≤ , rezultǎ cǎ { }41,63,85ab∈ , dintre care prim este 41.

Prin urmare, numerele cǎutate sunt 11 şi 41. SoluŃia 2 (a redacŃiei) : Dintre cele 21 de numere prime de douǎ cifre numai 10 verificǎ condiŃia necesarǎ ca cifra zecilor sǎ fie mai mare sau egalǎ cu cifra unitǎŃilor. Din tabelul urmǎtor :

ab 11 31 41 43 53 61 71 73 83 97

( )3E ab ba a b= − + − 20 0= 24 236 6= 12 24 60 72 48 60 24

Este E pǎtrat perfect ? DA NU DA NU NU NU NU NU NU NU deducem cǎ numerele cǎutate sunt 11 şi 41. 5.47 : SuprafaŃa unui pǎtrat cu perimetrul de 100 cm se descompune în douǎ dreptunghiuri. Dacǎ aria unui dreptunghi este de 275cm , sǎ se afle perimetrul celuilalt dreptunghi.

Prof. Felicia Ozunu, Şcoala Vulcan, Hunedoara SoluŃie : SuprafaŃa unui pǎtrat se poate descompune numai în douǎ dreptunghiuri printr-o dreaptǎ paralelǎ la o laturǎ. Astfel deducem cǎ cele douǎ dreptunghiuri au lungimile egale cu

Page 35: De ce să învăŃăm la matematică? Word - 2011 Revista Mici matematicieni...coordonate, de axe ortogonale sau nu. Ceea ce înseamnă…matematică. Reintrând într-un registru

Micii MATEMATICIENI

35

latura pǎtratului, adicǎ un sfert din perimetru : 100 : 4 25= cm, iar suma lǎŃimilor egalǎ cu 25 cm. Cum aria unui dreptunghi este de 275cm atunci lǎŃimea sa este 75 : 25 3cm= , de unde deducem cǎ lǎŃimea celuilalt dreptunghi este 25 3 22cm− = şi cum are lungimea de 25 cm, gǎsim perimetrul cǎutat: 2 25 2 22 94 cm⋅ + ⋅ = .

5.48 : Sǎ se arate cǎ dacǎ ( ) ( )

1 1 1

0, 0,0x x x+ + ∈ℕ atunci 2 5x − este divizor al lui 2010.

Prof. Hermina Romaşcu, Şcoala Picioru Lupului, Ciurea, Iaşi

SoluŃie : Transformând în fracŃii ordinare obŃinem : 1 9 90 100

x x x x+ + ∈ ⇔ ∈ℕ ℕ , de unde rezultǎ

cǎ cifra x este divizor al lui 100. Deci { }1,2,4,5x∈ , de unde { }2 5 3, 1,3,5x − ∈ − − . Cum avem

descompunerea lui 2010 1 2 3 5 67= ⋅ ⋅ ⋅ ⋅ rezultǎ cǎ 2 5x − este divizor întreg al lui 2010.

5.49 : La concursul ” EUCLID” organizat I de Liceul ”Vasile Alecsandri” Iaşi o treime dintre candidaŃi au fost elevi ai liceului, 250 elevi de la alte unităŃi şcolare din municipiul Iaşi, iar restul, care reprezintă un sfert din numărul de candidaŃi, din alte localităŃi. AflaŃi numărul total de candidaŃi.

Prof. Nelu Tudorache, Liceul ”Vasile Alexandri”, Iaşi SoluŃia 1 : Din schema graficǎ a problemei se deduce cǎ 12 pǎrŃi reprezintǎ numǎrul total de concurenŃi, 4 pǎrŃi nr. De elevi ai liceului, 5 părŃi nr. De elevi din municipiu, 3 părŃi nr. De elevi din alte localităŃi. Avem 5 pǎrŃi echivalent cu 250 elevi, de unde o parte reprezintǎ 50 elevi. Prin urmare, numǎrul total de candidaŃi este de 12 50 600⋅ = candidaŃi.

SoluŃia 2 : Notând cu x , numǎrul total de candidaŃi obŃinem ecuaŃia 2503 4

x xx+ + = , de unde

prin eliminarea numitorilor 4 3000 3 12 7 3000 12 5 3000 600x x x x x x x+ + = ⇔ + = ⇔ = ⇔ = . 5.50 : Sǎ se arate cǎ existǎ 10 numere naturale diferite cu suma 2009, care satisfac simultan condiŃiile: a) împǎrŃind fiecare dintre aceste numere la 7 , obŃinem resturi egale cu 2 sau cu 3;

b) suma tuturor acestor resturi este 28.

Prof. Petru Asaftei, Şcoala Normalǎ “Vasile Lupu”, Iaşi

SoluŃia autorului: Numerele cǎutate sunt de forma 7 i ic r⋅ + cu { }1,2,...,10i∈ şi { }2,3ir ∈ .

Fǎcând suma acestor numere obŃinem ( )1 2 102009 7 ... 28c c c= ⋅ + + + + , de unde

1 2 10... 283c c c+ + + = .

� Notând cu x , numǎrul resturilor egale cu 2 obŃinem ecuaŃia ( )2 3 10 28x x⋅ + ⋅ − = având

soluŃia 2x = , ceea ce înseamnǎ cǎ 2 resturi sunt egale cu 2, iar celelalte 8 egale cu 3. � Rǎmâne sǎ arǎtǎm cǎ existǎ 10 numere diferite cu suma egalǎ cu 283. Pentru aceasta,

observǎm cǎ 10 22 220⋅ = şi cǎ ( ) ( ) ( ) ( ) ( )22 1 22 2 22 3 ... 22 9 22 18 283+ + + + + + + + + + = , ceea ce

finalizeazǎ soluŃia. Nota redacŃiei : O soluŃie corectǎ a dat elevul Aştefǎnesei Daniel (cl. A V-a), gǎsind efectiv numerele şi anume: 226; 241; 254; 276; 248; 101; 94; 199; 206 şi 164 (fǎrǎ a preciza modalitatea de cǎutare a numerelor).

Nr. elevi ai liceului

Nr. elevi din alte localităŃi

Nr. elevi din municipiu

Page 36: De ce să învăŃăm la matematică? Word - 2011 Revista Mici matematicieni...coordonate, de axe ortogonale sau nu. Ceea ce înseamnă…matematică. Reintrând într-un registru

Micii MATEMATICIENI

36

5.51 : Se consideră numărul 201010 2008A = − . ArătaŃi că suma cifrelor acestui număr este divizibilă cu 2009.

Prof. Ana Marioara Spiridon, Prof. Gheorghe Spiridon , Paşcani SoluŃie : Avem 2010

2010 2006

10 2008 100...00000 2008 99...997992zerouri

A = − = − =�� �� . Atunci suma cifrelor

numǎrului A este egalǎ cu ( )2006 9 7 9 9 2 2006 9 3 9 9 2006 3 9 2009⋅ + + + + = ⋅ + ⋅ = ⋅ + = ⋅ . Prin

urmare suma cifrelor numǎrului A este divizibilǎ cu 2009.

5.52 : Sǎ se determine cifra a astfel ca numǎrul natural 148

aaaa ⋅ sǎ fie un cub perfect.

Prof. Costache RaŃǎ, Şcoala Preventoriu-Deleni, Iaşi

SoluŃie : Din 2

2111 3 37 3

148 148 4 37 4

aaa a aa a a

⋅ ⋅⋅ ∈ ⇒ ⋅ ∈ ⇒ ⋅ ∈ ⇒ ∈

⋅ℕ ℕ ℕ ℕ şi cum ( )4,3 1= rezultǎ

cǎ 24 2a a⇒ { }2,4,6,8a⇒ ∈ , cǎci a este cifrǎ. Cum 148

aaaa ⋅ este un cub perfect rezultǎ cǎ

23

4

a este cub perfect, deci 2 23 3a a⇒ şi cum { }2,4,6,8a∈ gǎsim cǎ 6a = .

Clasa a VI-a

6. 30 : Media aritmeticǎ a trei numere este 2

3

2−

. AflaŃi numerele ştiind cǎ sunt direct

proporŃionale cu 3,0

1,

)3(,0

1 ,

)3(0,0

1.

Prof. Nicu Goşman, Şcoala ,,G. Ibrăileanu” ,Tg.Frumos

SoluŃie : Notǎm cu a , b , c numerele cǎutate . Avem 2

2 9

3 3 4

a b c−

+ + = =

27

4a b c⇒ + + = .

Din a , b , c direct proporŃionale cu 3,0

1,

)3(,0

1 ,

)3(0,0

1 rezultǎ cǎ ele sunt invers proporŃionale

cu 0,3 , 0, (3) , 0, 0(3) , adicǎ are loc: ( ) ( )0,3 0, 3 0,0 3a b c⋅ = ⋅ = ⋅ , de unde dupǎ împǎrŃirea cu 3

avem: ( ) 27

10 9 90 109 436

rapoarte

egale

a b c a b c ∗+ += = = = . Deci

270

436a = ,

243

436b = şi

2430

436c = .

6. 31 : DeterminaŃi abc ştiind cǎ ;a b+ ;b c+ c a+ sunt direct proporŃionale cu numerele

2 ;a b− 2 ;b c− 2c a− şi cǎ 81b a ba b c+ + = . Prof. Ioan RăuŃu, Şcoala “Petru Rareş”, Hirlău

SoluŃie : Avem : 2 2 2

22 2 2

rapoarte

egale

a b b c c a a b c

a b b c c a a b c

+ + + + += = =

− − − + += . Din 22

a b

a b

+=

− rezultǎ cǎ

4 2a b a b+ = − 3 3b a⇒ = b a⇒ = . Din 22

b c

b c

+= ⇒

− 4 2b c b c+ = − 3 3c b⇒ = c b⇒ = .

Folosind 81b a ba b c+ + = obŃinem cǎ 3 81bb⋅ = , de unde 33 3bb b= ⇒ = . Deci 333abc = .

Page 37: De ce să învăŃăm la matematică? Word - 2011 Revista Mici matematicieni...coordonate, de axe ortogonale sau nu. Ceea ce înseamnă…matematică. Reintrând într-un registru

Micii MATEMATICIENI

37

6. 32 : Trei elevi trebuiau să-şi plătească o excursie cu sume direct proporționale cu numerele 3, 4, 5. Dacă taxa fixată fiecăruia se majorează cu 10 % , 15 % , respectiv 20 % atunci ei ar plăti cu 120 lei mai puŃin decât dacă taxele se majorează cu 20 % , 25 % , respectiv 30 % . Ce sumă trebuia să plătească fiecare elev ?

Prof. Turnea Mihaela, Liceul “I. Neculce” Târgu Frumos

SoluŃie : Sumele a , b , c fiind direct proporŃionale cu 3, 4, 5 rezultǎ cǎ .

3 4 5

nota b ck= = = , de unde

3a k= , 4b k= şi 5c k= . Avem10 15 20 20 25 30

120100 100 100 100 100 100

a b c a b c⋅ + ⋅ + ⋅ + = ⋅ + ⋅ + ⋅ , relaŃia

dintre majorǎri. Atunci 10 3 15 4 20 5 12000 20 3 25 4 30 5k k k k k k⋅ + ⋅ + ⋅ + = ⋅ + ⋅ + ⋅ de unde gǎsim cǎ 190 12000 310k k⇒ + = 12000 120k⇒ = 100k⇒ = . Deci 300a = , 400b = şi 500c = . Nota redacŃiei: Problema se regǎseşte printre subiectele Olimpiadei judeŃeane de matematicǎ date în anul 2002 în judeŃul Cluj.

6. 33 : Cu 100 euro s-au cumpǎrat 100 de obiecte de trei tipuri având preŃuri de 5 eurocenŃi, de 1 euro, respectiv de 5 euro. DeterminaŃi câte obiecte de fiecare fel s-au cumpǎrat.

Prof. Mina Merlǎ, Şcoala łuŃora, Iaşi SoluŃie : Notǎm cu , ,a b c numǎrul celor trei obiecte cumpǎrate. Atunci 100a b c+ + = ( )1 .

łinând cont de preŃul fiecǎruia se obŃine 5 100 500 10000a b c+ + = ( )2 (în eurocenŃi). ÎnmulŃind

relaŃia (1) cu 100 şi scǎzând-o din (2) gǎsim 95 400 0 19 80a c a c− + = ⇒ = . Cum 19 este prim rezultǎ cǎ 19 divide c , aşadar 19c x= , iar 80a x= cu x∈ℕ . Înlocuind în egalitatea (1) obŃinem 19 80 100 99 100x b x x b+ + = ⇔ + = . Cum 0x ≠ şi 99 100x ≤ rezultǎ 1x = şi 1b = . Prin urmare numǎrul obiectelor sunt : 80 obiecte de 5 eurocenŃi, 1 obiect de 1 euro şi 19 de 5 euro.

6. 34 : CalculaŃi 120x y⋅ − ştiind că :

5 10 15 ... 125x = + + + + şi 1 1 1 1

...1 26 2 39 3 52 25 312

y = + + + +⋅ ⋅ ⋅ ⋅

.

Prof. Spiridon Ana Marioara, Şcoala ,, Iordachi Cantacuzino’’, Paşcani

SoluŃia autorului Avem ( ) 26 255 1 5 2 5 3 ... 5 25 5 1 2 .. 25 5 5 13 25

2x

⋅= ⋅ + ⋅ + ⋅ + + ⋅ = ⋅ + + + = ⋅ = ⋅ ⋅

Calculul lui 1 1 1 1 1 1 1 1 1

... ...1 2 13 2 3 13 3 4 13 25 24 13 13 1 2 2 3 3 4 25 24

y = + + + + = ⋅ + + + + ⋅ ⋅ ⋅ ⋅ ⋅ ⋅ ⋅ ⋅ ⋅ ⋅ ⋅ ⋅

.

Cum ( )1 1 1

1 1n n n n= −

+ + rezultǎ

1 1 1 1 1 1 1 1 1 1 24...

13 1 2 2 3 24 25 13 1 25 13 25y

= ⋅ − + − + + − = ⋅ − = ⋅ .

Avem 24

120 5 13 25 120 5 24 120 120 120 013 25

x y⋅ − = ⋅ / ⋅ ⋅ − = ⋅ − = − =// ⋅ /

.

Nota redacŃiei : În sensul soluŃiei autorului, ultimul termen al sumei care defineşte pe y şi

anume 1

25 312⋅ ar fi trebuit sǎ fie

1

24 325⋅.

6. 35 : a) GăsiŃi divizorii numărului 2010.

b) AflaŃi valorile naturale ale lui x pentru care 41 2011

41 1

x

x

⋅ +∈

⋅ +ℕ .

Prof. Nicolae Ivăşchescu, Colegiul NaŃional ” FraŃii Buzeşti”, Craiova SoluŃie : a) Descompunem pe 2010 în factori primi şi gǎsim : 2010 2 3 5 67= ⋅ ⋅ ⋅ .

Page 38: De ce să învăŃăm la matematică? Word - 2011 Revista Mici matematicieni...coordonate, de axe ortogonale sau nu. Ceea ce înseamnă…matematică. Reintrând într-un registru

Micii MATEMATICIENI

38

Numǎrul divizorilor este ( ) ( ) ( ) ( ) 41 1 1 1 1 1 1 1 2 16+ ⋅ + ⋅ + ⋅ + = = . Aceşti divizori sunt : 1, 2, 3, 5, 6,

10, 15, 30, 67, 134, 201, 335, 402, 670, 1005 şi 2010.

b) Din 41 2011

41 1

x

x

⋅ +∈

⋅ +ℕ rezultǎ cǎ 41 1 41 2011x x+ + şi cum 41 1 41 1x x+ + , deducem cǎ

( ) ( )41 1 41 2011 41 1 41 1 2010x x x x+ + − + ⇒ + , de unde 201041 1x D+ ∈ . Egalǎm pe rând 41 1x +

cu divizorii lui 2010, obŃinuŃi la a). Singura valoare pentru care x∈ℕ se obŃine din 41 1 2010x + = şi anume, 49x = ; în rest se verificǎ prin calcul cǎ nu se obŃine x numǎr natural. 6. 36 : În triunghiul dreptunghic ABC ( ( ) 090m B =∢ , ( )M BC∈ astfel încât 2AM BM= .

Dacă 2AC AB= , demonstraŃi că M se află pe mediatoarea laturii AC. Prof.ConstanŃa Tudorache, Şcoala ”St. Bârsănescu”, Iaşi

SoluŃie : Reciproca teoremei unghiului de 030 spune cǎ, dacǎ într-un triunghi dreptunghic o catetǎ este jumǎtate din ipotenuzǎ atunci unghiul opus acelei catete are mǎsura de 030 . Deoarece

cateta 2

ACAB = şi ABC△ este dreptunghic în B rezultǎ cǎ ( ) 030m ACB =∢ ( )1 , dar şi în

( )090ABM B =△ ∢ avem cǎ 2

AMBM = , deci ( ) 030m BAM =∢ . Cum unghiurilor ascuŃite

( )090ABC B =△ ∢ sunt complementare rezultǎ cǎ ( ) 060m BAC =∢ . Rezultǎ cǎ

( )030CAM MCA≡ =∢ ∢ , cǎci ( ) ( ) ( ) 0 0 060 30 30m CAM m BAC m BAM= − = − =∢ ∢ ∢ .

Deducem cǎ ACM△ este isoscel de bazǎ ( )AC , de unde M se află pe mediatoarea laturii AC.

6. 37 : Suma a 999 numere naturale nenule distincte este 990000. Care poate fi numǎrul maxim

de numere impare din aceastǎ sumǎ? Prof. Petru Asaftei, Şcoala Normalǎ “Vasile Lupu”, Iaşi

SoluŃie : Cele 999 numere nu pot fi toate pare cǎci altfel suma din enunŃ ar fi ≥ decât suma celor mai mici numere pare distincte, adicǎ 99000 2 4 6 ... 1998 99000 999000≥ + + + + ⇔ ≥ , absurd. Mai mult, numǎrul numerelor pare trebuie sǎ fie par cǎci altfel suma lor ar trebui sǎ fie imparǎ, în contradicŃie cu 99000, numǎr par. Deci printre cele 999 numere sunt cel puŃin douǎ numere impare. Calculǎm suma primelor 999 numere impare consecutive: 21 3 5 1997 999 998001+ + +…+ = = ; ea este mai mare faŃǎ de suma datǎ ( 990000 ) cu 8001. Pentru a « echilibra » suma calculatǎ cu diferenŃa obŃinutǎ astfel încât sǎ obŃinem un numǎr maxim de numere impare ar trebui sǎ înlǎturǎm un numǎr minim de numere impare dintre cele 999 numere impare consecutive. Evident, ar trebui eliminate pe cele mai mari. Vom testa aceastǎ « posibilǎ » eliminare pânǎ obŃinem suma acestora mai mare decât diferenŃa obŃinutǎ (8001). Deoarece 1997 1995 3992 8001+ = < ; 1997 1995 1993 5985 8001+ + = < ; 1997 1995 1993 1991 7976 8001+ + + = > şi întrucât suma : 1997 1995 1993 1991 1989 9965 8001+ + + + = > , a cǎrui diferenŃǎ este de 1964 trebuie sǎ înlocuim aceastǎ diferenŃǎ (1964 ) cu 5 numere pare, pe care sǎ le intercalǎm în şirul 1, 3, 5, , 1987… . Acest lucru este posibil, de exemplu 1964 2 4 6 8 1944= + + + + . Concluzionǎm cǎ numǎrul maxim de numere impare este 994.

6. 38 : ArǎtaŃi cǎ ( ) ( ) ( )2

4 4 12 2 21 1 1 3 3 3

1 1 1 1,2 3 6 4 4 4

n nn n n n

n+

+ + − ⋅ + − ⋅ + − ⋅ + ⋅ − + − = ∀ ∈

ℕ .

Prof. Victor Sǎceanu, Drobeta Turnu Severin

Page 39: De ce să învăŃăm la matematică? Word - 2011 Revista Mici matematicieni...coordonate, de axe ortogonale sau nu. Ceea ce înseamnă…matematică. Reintrând într-un registru

Micii MATEMATICIENI

39

SoluŃie : Numerele 2n , ( )2 1n n n n+ = + şi ( )2 2 2 1n n+ = + fiind pare, avem urmǎtoarea

desfǎşurare succesivǎ a membrului drept al egalitǎŃii din enunŃ : 4 4 1 4 1 4 1

1 1 1 3 3 3 1 1 1 3 3 1 1 1 3 2 11 1 1 1

2 3 6 4 4 4 2 3 6 4 4 2 3 6 6

n n n n+ + ++ + ⋅ + ⋅ + ⋅ + ⋅ − = + + + − = + + = =

.

Prin urmare, are loc egalitatea din enunŃ pentru orice n∈ℕ .

6. 39 : Fie suma 100

3 33 333 ... 333...3S = + + + +�� .

a) Să se determine numǎrul S. b) Să se determine suma cifrelor numǎrului S.

Prof. Popa Gabriela, Şcoalan nr. 43 “D.A.Sturza”, Iaşi SoluŃie a)Fie � ( ) ( ) ( ) �2 100

100 101 101

9 99 ... 99...9 10 1 10 1 ... 10 1 1...10 100 1...1010A = + + + = − + − + + − = − =�� .

Atunci ( )100 100

3 33 333 ... 333...3 3 : 9 : 3 370370...3703670S A A= + + + + = ⋅ = =�� �� . Vom demonstra

ultima egalitate. Avem ( )97 94 91 4: 3 1110 10 1110 10 1110 10 ... 1110 10 11010 :3S A= = ⋅ + ⋅ + ⋅ + + ⋅ +

deci ( ) ( ) ( ) ( ) ( )97 94 91 4: 3 1110 :3 10 1110 :3 10 1110 :3 10 ... 1110 :3 10 11010 :3S A= = ⋅ + ⋅ + ⋅ + + ⋅ + ,

adicǎ ( ) ( ) ( ) ( ) ( )97 94 91 4: 3 1110 :3 10 1110 :3 10 1110 :3 10 ... 1110 :3 10 11010 :3S A= = ⋅ + ⋅ + ⋅ + + ⋅ +

de unde ⇔100

: 3 370370...3703670S A= =�� unde avem 32 grupe consecutive cu numǎrul 370.

b)Suma cifrelor lui S este: ( )32 3 7 0 3 6 7 0 32 10 16 320 16 336⋅ + + + + + + = ⋅ + = + = .

6. 40 : La un spectacol sunt 500 spectatori care au plǎtit câte 40 lei pentru biletul de intrare. Dacǎ la spectacol ar veni cu 20 % mai mulŃi spectatori, cu ce procent ar trebui sǎ scadǎ preŃul biletului pentru a încasa o sumǎ cu 5 % mai mare faŃǎ de primul spectacol ?

Prof. Mirela Popescu, Şcoala Leşile, Dolj SoluŃie : Suma încasatǎ la primul spectacol este de 500 40 20000⋅ = lei. La al doilea spectacol,

numǎrul spectatorilor este 20

500 500 500 100 600100

+ ⋅ = + = spectatori. Suma mǎritǎ cu 5% faŃǎ

de primul spectacol este 5

20000 20000 20000 1000 21000100

+ ⋅ = + = lei. Noul preŃ al biletului

este 21000 : 600 35= lei . Procentul p% cu care s-a micşorat biletul se calculeazǎ în acest mod : 4 50

40 40 35 5 50 4 12,5100 10 4

p pp p p− ⋅ = ⇔ = ⇔ = ⇔ = ⇔ = %.

6. 41 : Fie ABC△ cu [ ] [ ]AB AC≡ . Perpendicularele în B pe AB şi în C pe AC se intersectează

în punctul I. Fie [ ]M AB∈ , N AC∈ astfel încât [ ]C AN∈ şi AC AM CN= + .

a) DemonstraŃi că mediatoarea segmentului MN trece prin I. b) PrecizaŃi poziŃia punctului P astfel încât patrulaterul MINP să fie romb.

Prof. Nelu Tudorache, Liceul ”Vasile Alexandri”, Iaşi

SoluŃie : a) Din [ ] [ ]AB AC≡ , [ ] [ ]AI AI≡ şi ( )090ABI ACI≡ =∢ ∢ IC

ABI ACI⇒ ≡△ △ de unde

BAI CAI⇒ ≡∢ ∢ [AI⇒ este bisectoarea BAC∢ [AI⇒ este bisectoarea MAN△ ( )1 . Deoarece

Page 40: De ce să învăŃăm la matematică? Word - 2011 Revista Mici matematicieni...coordonate, de axe ortogonale sau nu. Ceea ce înseamnă…matematică. Reintrând într-un registru

Micii MATEMATICIENI

40

AC AM CN= + AN CN AM CN AN AM AMN⇒ + = + ⇒ = ⇒△ este isoscel de bazǎ MN . łinând cont şi de ( )1 , rezultǎ cǎ AI este mediatoarea segmentului MN.

b) Pentru ca patrulaterul MINP să fie romb trebuie ca fiecare diagonalǎ sǎ fie mediatoarea celeilalte. Cum AI este mediatoarea diagonalei MN rezultǎ cǎ P AI∈ . Mai este necesar ca MN sǎ fie mediatoarea diagonalei PI , de unde avem cǎ P este simetricul punctului I faŃǎ de MN.

6. 42 : DemonstraŃi cǎ suma a 30 numere întregi consecutive este 75− dacǎ şi numai dacǎ numerele negative sunt cu 5 mai multe decât cele pozitive.

Prof. Mihai Crăciun, Colegiul NaŃional “M. Sadoveanu”, Paşcani SoluŃie : Notǎm cu p ∗∈ℕ câte numere negative sunt în plus faŃǎ de cele pozitive. Considerǎm

urmǎtoarele numere consecutive : ( ); 1 ;...; 1; ; 1;...; 1;0;1;2;...;n p n p n n n n− − − − − − − − − + − .

Avem 1 2 1n n p n p+ + + = + + numere consecutive în şir, de unde 2 1 30n p+ + = ( )1 . Suma

lor este ( ) ( )( ) ( ) ( ) ( )1 ... 1 1 ... 1 0 1 2... 1S n p n p n n n n n/ / /= − − + − − − + − − − − − − − + + + + − +/ / , de

unde ( ) ( ) ( ) ( )1 2 1 2 11 2 ...

2 2 2

p p n p p p p n pS n p p n p

⋅ + ⋅ + ⋅ + − ⋅ + += − ⋅ − + + + = − ⋅ − = − = .

Folosind egalitatea ( )1 gǎsim cǎ 30

152

pS p

− ⋅= = − ( )2 .

" "⇒ Presupunem cǎ suma este 75− . Atunci din ( )2 avem: 15 75p− = − , de unde 5p = , ceea

ce înseamnǎ cǎ numerele negative sunt cu 5 mai multe decât cele pozitive. " "⇐ Presupunem cǎ numerele negative sunt cu 5 mai multe decât cele pozitive. Atunci

5p = . Folosind ( )2 obŃinem cǎ suma este 15 15 5 75S p= − = − ⋅ = − .

ObservaŃie : Aceste numere sunt 17, 16,..., 1,0,1,...,12− − − . Clasa a VII-a

7. 24 : Rădăcina pătrată din produsul numerelor 316

a şi 3

25

b este egală cu

3320 .

Să se determine numerele a şi b. Prof. Nicu Goşman, Şcoala ,,G.Ibrăileanu” ,Tg.Frumos

SoluŃie : Avem3

3 3 316 25 20

a b⋅ = cu 16a < şi 25b < . Ridicând la pǎtrat şi introducând întregii

în fracŃie obŃinem : ( ) ( ) ( ) ( )

22

2

48 75 6348 75 63

16 25 20

a ba b

+ ⋅ += ⇔ + ⋅ + =

⋅. Cum 75 75 100b< + <

şi 26375b D+ ∈ rezultǎ singura posibilitate 75 81 6b b+ = ⇒ = , iar 48 49 1a a+ = ⇒ = .

7. 25 : Cifrele a, b, c, d, x, y şi z verifică egalitatea: ab

xy z abcd⋅ + = cu ab şi xy numere prime.

a) Să se demonstreze că { },cd xy ab∈ .

b) ArătaŃi că numărul 0cdxy nu poate fi pătrat perfect. Prof. Ioan Sǎcǎleanu, Liceul ” Ştefan cel Mare”, Hirlău

SoluŃie : a) Dacǎ 0z = atunci egalitatea din enunŃ devine ab

xy ab xy cdcd⋅ = ⇒ = .

Page 41: De ce să învăŃăm la matematică? Word - 2011 Revista Mici matematicieni...coordonate, de axe ortogonale sau nu. Ceea ce înseamnă…matematică. Reintrând într-un registru

Micii MATEMATICIENI

41

Dacǎ 0z ≠ atunci din ab

xy z abcd⋅ + = ( )1 xy ab z cd ab cd⇒ ⋅ + ⋅ = ⋅ z cd ab cd xy ab⇒ ⋅ = ⋅ − ⋅ ,

de unde deducem cǎ ( )z cd ab cd xy⋅ = ⋅ − . Rezultǎ cǎ numǎrul prim ab divide z cd⋅ . Cum ab

nu divide cifra z rezultǎ cǎ ab divide cd , adicǎ existǎ :k cd k ab∈ = ⋅ℕ ( )2 . Înlocuim în ( )1 :

abxy z ab

k ab⋅ + =⋅

şi obŃinem: xy

ab zk= − ∈ℕ , de unde k este divizor al numǎrului prim xy .

Avem k xy≠ , cǎci altfel membrul drept al egalitǎŃii ( )2 ar fi numǎr de 3 cifre, iar cel stâng are

2 cifre. Deci 1k = şi din ( )2 rezultǎ cǎ cd ab= . Prin urmare { },cd xy ab∈ .

b) Deoarece numărul 0cdxy este divizibil cu 10 rezultǎ cǎ el este pătrat perfect numai dacǎ se

divide cu 100, de unde 0y = , adicǎ numǎrul prim xy este par, absurd cǎci singurul numǎr prim

şi par este 2. Prin urmare numărul 0cdxy nu poate fi pătrat perfect.

7. 26 : Să se determine numărul prim ab ştiind că partea întreagă a numărului ab este 6 şi că suma cifrelor numărului este 5.

Prof. Bogdan Dorneanu, Şcoala “Petru Rareş”, Hîrlău

SoluŃia 1 : Partea întreagă a numărului ab este 6. Atunci 6 7ab≤ < . Prin ridicare la pǎtrat

obŃinem 36 49ab≤ < . Cum ab este numǎr prim gǎsim cǎ singurul 41ab = şi mai mult, are suma cifrelor egalǎ cu 5.

SoluŃia 2 : Din 5a b+ = rezultǎ { }14;23;32;41;50ab∈ , de unde singurul numǎr prim este 41 şi

mai mult, partea întreagǎ a lui 41 6,4≃ este 6.

SoluŃia 3 : Din 5a b+ = rezultǎ { }14;23;32;41;50ab∈ . Cum 14 3,7≃ , 23 4,7≃ ,

32 5,6≃ , 41 6,4≃ şi 50 7,07≃ rezultǎ cǎ singurul numǎr cu partea întreagǎ egalǎ cu 6

este 41 , adicǎ 41ab = şi mai mult, este umǎr prim. Nota redacŃiei : Dupǎ cum se observǎ din soluŃiile date, problema are un surplus de informaŃii. Era suficient sǎ ştim numai douǎ din cele 3 informaŃii ca sǎ rezolvǎm problema. Mai mult,

oricare douǎ din : ( )1 ab este numǎr prim, ( )2 ab suma cifrelor egalǎ cu 5, ( )3 partea întreagǎ

a numǎrului ab este 6, implicǎ pe a treia. 7. 27 : Sǎ se determine n∈ℤ astfel încât 1n + sǎ fie cel mai mare numǎr natural, pǎtrat

perfect mai mic ca 2010. Prof. Aurel Neicu, Liceul ” Ştefan cel Mare”, Hîrlău

SoluŃie : Deoarece 2 244 1936 2010 2025 45= < < = rezultǎ cǎ cel mai mare pǎtrat mai mic decât

2010 este 1936. Deci 1 1936n + = , de unde 21936 1n = − , adicǎ 3748095n = .

7. 28 : Se dă triunghiul ABC în care ( )M AB∈ cu 7

9

MA

AB= , ( )N BC∈ cu

3

10

BN

BC= şi

( )P AC∈ cu 5

3

AC

PA= . DemonstraŃi că dreptele AN, BP şi CM sunt concurente .

Prof. Nicolae Ivăşchescu, Colegiul NaŃional ” FraŃii Buzeşti”, Craiova

Page 42: De ce să învăŃăm la matematică? Word - 2011 Revista Mici matematicieni...coordonate, de axe ortogonale sau nu. Ceea ce înseamnă…matematică. Reintrând într-un registru

Micii MATEMATICIENI

42

SoluŃie : Folosind proporŃii derivate, obŃinem din 7 7 7

9 9 7 2

MA MA AM

AB AB MA MB= ⇒ = ⇒ =

− − ( )1 ,

din 3 3 3

10 10 3 7

BN BN BN

BC BC BN NC= ⇒ = ⇒ =

− −( )2 , iar 5 5 3 2

3 3 3

AC AC PA PC

PA PA PA

− −= ⇒ = ⇒ = . ( )3

ÎnmulŃind ( )1 , ( )2 şi ( )3 obŃinem 7 3 2

12 7 3

AM BN CP

MB NC PA

///⋅ ⋅ = ⋅ ⋅ =

/ //, de unde conform reciprocei

teoremei lui Ceva obŃinem cǎ AN , BP şi CM sunt ceviene concurente. 7. 29 : Fie paralelogramul ABCD în care AD DB⊥ . Fie E şi N pe dreapta BC aşa încât

( )B CE∈ , ( )E BN∈ şi [ ] [ ] [ ]CB BE EN≡ ≡ .

a) DemonstraŃi cǎ triunghiul ABN este isoscel. b) Dacǎ M este simetricul lui B faŃǎ de D, arǎtaŃi cǎ punctele N, A, M sunt coliniare.

Prof. Nelu Tudorache, Liceul ”Vasile Alexandri”, Iaşi SoluŃie : Notǎm cu O, intersecŃia diagonalelor [ ]AC şi [ ]BD , T simetricul lui A faŃǎ de D, iar

cu S mijlocul lui [ ]AB .

a) Din [ ] [ ]CB BE≡ ⇒ cǎ B este mijlocul lui [ ]CE . Din faptul cǎ ABCD este paralelogram,

aflǎm cǎ O este mijlocul lui [ ]AC . Deducem cǎ OB este linie mijlocie în ACE△ , de unde

OB AE� şi cum OB BC⊥ (ip.) rezultǎ AE înǎlŃime în ABN△ . Dar, [ ] [ ]BE EN≡ atunci

AE este şi medianǎ în ABN△ . Prin urmare ABN△ este triunghi isoscel de bazǎ [ ]BN .

b) SoluŃia 1 : Din M, simetricul lui B faŃǎ de D ⇒ D mijlocul lui [ ]MB şi cum S mijlocul lui

[ ]AB , deducem cǎ DS este linie mijlocie în MAB△ .

Conform teoremei liniei rezultǎ cǎ DE AM� ( )1 .

Analog, SE este linie mijlocie în NAB DE AN⇒△ � Atunci N, A, M sunt coliniare cǎci altfel prin A s-ar duce douǎ paralele la DE, astfel contrazicându-se axioma paralelelor. SoluŃia 2 : Din ABCD paralelogram AD BC� , de

unde AD BE� , dar şi ip

AD BC BE= = . Deducem cǎ AEBD este paralelogram şi cum AD DB⊥ (ip.) rezultǎ AEBD este dreptunghi AB DE⇒ = . Avem DE, DS şi SE linii mijlocii în BNM△ , ABM∆ , respectiv ABN△

Atunci 2

MNDE⇒ = ,

2

AMDS = şi

2

ANSE = .

Cum D S E− − coliniare atunci DE DS SE= + , de unde MN MA AN= + . Conform criteriului metric de coliniaritate rezultǎ cǎ M A N− − coliniare. SoluŃia 3 : Deoarece D mijlocul lui [ ]MB şi a lui [ ]AT , rezultǎ cǎ ABTM paralelogram

AM BT⇒ � . Avem 2 2 2AT AD BC BE BN= = = = şi cum AT BN� cǎci AD BC� rezultǎ cǎ ATBN este paralelogram AN BT⇒ � . Atunci N, A, M sunt coliniare cǎci altfel prin A s-ar duce douǎ paralele la DE, contrazicându-se astfel axioma paralelelor.

D

A B

C

E

M

N

T

O

Sx xy

y

Page 43: De ce să învăŃăm la matematică? Word - 2011 Revista Mici matematicieni...coordonate, de axe ortogonale sau nu. Ceea ce înseamnă…matematică. Reintrând într-un registru

Micii MATEMATICIENI

43

SoluŃia 4 : Deoarece AD este înǎlŃime şi medianǎ în ABM rezultǎ cǎ AD bisectoarea BAM∢ ,

de unde not

MAD DAB y= =∢ ∢ . Din ABN△ este triunghi isoscel de bazǎ [ ]BN şi AE medianǎ

rezultǎ cǎ AE bisectoarea BAN∢ , de unde .not

BAE NAE x= =∢ ∢ . La fel ca în soluŃia 2 se aratǎ cǎ AEBD este dreptunghi, de unde ( ) 090m DAE =∢ 090x y⇔ + = . Dar, ( ) 2 2m MAN x y= +∢ .

Prin urmare ( ) ( ) 02 180m MAN x y⇒ = + =∢ , adicǎ MAN∢ este unghi alungit. Deci laturile

sale sunt semidrepte opuse, de unde M, A, N sunt puncte coliniare. 7. 30 : Fie patrulaterul oarecare ABCD. Fie E mijlocul diagonalei AC , F mijlocul lui BE, M mijlocul lui CF şi P mijlocul lui BM.

a) Sǎ se arate cǎ BAF∆ si EFC∆ au aceeaşi arie. b) Sǎ se arate cǎ A, F, P sunt puncte coliniare.

Prof. Popa Gabriela, Şcoala nr. 43 “D.A.Sturza”, Iaşi SoluŃie : a) Se ştie cǎ mediana unui triunghi descompune triunghiul în douǎ triunghiuri echivalente (de arii egale). Din F , mijlocul lui [ ]BE rezultǎ AF medianǎ a ABE∆ , de unde

gǎsim BAF AFES S∆ ∆= ( )1 . Din E mijlocul lui [ ]AC rezultǎ FE

medianǎ în AFC∆ , de unde EFC AFES S∆ ∆= ( )2 .

Din ( )1 şi ( )2 obŃinem cǎ BAF∆ si EFC∆ au aceeaşi arie.

b) Se observǎ cǎ FP este linie mijlocie în BME∆ şi ca urmare avem FP ME� ( )3 , conform teoremei liniei mijlocii în triunghi.

Cum ME este linie mijlocie în AFC∆ rezultǎ cǎ ME FA� ( )4 .

Din ( )3 şi ( )4 rezultǎ cǎ prin F trec douǎ paralele la dreapta ME .

Conform axiomei paralelelor prin F poate trece o singurǎ paralelǎ la dreapta ME . Prin urmare dreptele FP şi FA coincid, adicǎ punctele A, F, P sunt puncte coliniare. 7. 31 : În dreptunghiul ABCD, E este simetricul lui A faŃă de diagonala BD, M este punctul de intersecŃie al dreptelor BE şi DC, iar N este piciorul perpendicularei din M pe BD.ArătaŃi că:

a) triunghiul MEC este isoscel; b) patrulaterul BCED este trapez isoscel;

c) dreptele AE şi EC sunt perpendiculare;

d) punctele A, N, C sunt coliniare. Prof.ConstanŃa Tudorache, Şcoala ”St. Bârsănescu”, Iaşi

SoluŃie : a) Din E, simetricul lui A faŃă de BD⇒BD mediatoarea lui [ ]AE ⇒ AE BD⊥ ( )1

AD DE≡ şi AB BE≡ (proprietatea mediatoarei). Cum AB DC≡ şi AD BC≡ cǎci ABCD dreptunghi, rezultǎ cǎ DE BC≡ , DC BE≡ ( )2

şi cum EC CE≡ LLL

DEC BCE⇒ ≡△ △ DCE BEC⇒ ≡∢ ∢ , adicǎ avem cǎ MCE MEC MEC≡ ⇒∢ ∢ △ isoscel MC ME⇒ ≡ ( )3 .

b) Din ( )2 şi ( )3 prin diferenŃǎ gǎsim MD MB≡ ( )4 MBD⇒△ â

isoscel, de unde ( ) ( )0180

2

m DMBm DBM

−=

∢∢ ,

A B

CD

E

M

N

Page 44: De ce să învăŃăm la matematică? Word - 2011 Revista Mici matematicieni...coordonate, de axe ortogonale sau nu. Ceea ce înseamnă…matematică. Reintrând într-un registru

Micii MATEMATICIENI

44

iar din MEC△ isoscel ( ) ( )0180

2

m CMEm MEC

−⇒ =

∢∢ . Cum DMB CME≡∢ ∢ (opuse la

vârf) rezultǎ cǎ DBM CEM DBE CEB≡ ⇔ ≡∢ ∢ ∢ ∢ şi cum sunt alterne interne⇒ BD CE BCED⇒� trapez cu diagonaleleDC BE≡ ( )2 ⇒ BCED trapez isoscel.

c) Din MN BD⊥ (ip.) şi CE BD� (b) rezultǎ cǎ MN CE⊥ ( )5 . Din BD mediatoarea lui

[ ]AE ⇒ AE BD⊥ şi cum MN BD⊥ rezultǎ MN AE� , dar MN CE⊥ ( )5 , de unde AE CE⊥ .

d) Din ( )4 rezultǎ cǎ MD MB≡ MBD⇒△ isocel şi cum MN înǎlŃime ⇒ N mijlocul lui [ ]BD .

Dar ABCD este dreptunghi, deci N este şi mijlocul lui [ ]AC . Prin urmare A, N, C sunt coliniare.

7. 32 : Fie triunghiul ABC, nedreptunghic şi O centrul cercului circumscris triunghiului , F pe latura BC, D pe latura AC a triunghiului ABC astfel incat AF∩BD={O}. Notăm cu G, respectiv cu H simetricele punctului O faŃă de F şi faŃă de D.Să se arate că sunt echivalente afirmaŃiile:

a) Punctele G şi H aparŃin cercului circumscris triunghiului ABC; b) AF si BD sunt înǎlŃimi ale triunghiului ABC.

Prof. Rodica Leontieş, Lic.„V. Alexandri”; Constantin Chirilǎ, Lic.„G. Ibrǎilenu”, Iaşi SoluŃie : Din F pe latura BC⇒ [ ] ( )AF Int ABC ABC⊂ ∆ ∪∆ , iar din D pe latura AC ⇒

[ ] ( )AF Int ABC ABC⊂ ∆ ∪∆ ⇒ ( )O AF BD Int ABC ABC∈ ∩ ⊂ ∆ ∪∆ . Deoarece ABC∆ nu

este dreptunghic O ABC⇒ ∉∆ , de unde O Int ABC∈ ∆ , adicǎ ABC∆ este ascuŃitunghic. ) )b a⇒ Dacǎ AF si BD sunt înǎlŃimi ale triunghiului ABC atunci intersecŃia lor O este

ortocentrul ABC∆ şi cum O centrul cercului circumscris triunghiului rezultǎ cǎ ABC∆ este echilateral. Deducem cǎ O este şi centrul de greutate al triunghiului. Atunci 2OA OF= , dar

2OG OF= (din simetrie), deci OG R= ⇒ ( );G C O R∈ , adicǎ G se aflǎ pe cercul circumscris

ABC∆ . Analog, se demonstreazǎ cǎ H aparŃine cercului circumscris triunghiului ABC. ) )a b⇒ Dacǎ Punctele G şi H aparŃin cercului circumscris triunghiului ABC atunci OG OH R= = . Din G, respectiv H simetricele punctului O faŃă de F şi faŃă de D rezultǎ cǎ

2

ROF OD= = . Gǎsim cǎ

1

2

OF OD

OA OB = =

. Rezultǎ din reciproca teoremei lui Thales cǎ

DF AB� . Din teorema fundamentalǎ a asemǎnǎrii obŃinem cǎ DOF BOA∆ ∆∼ , de unde

1

2

DF DO

AB OB= = şi cǎ CDF CAB∆ ∆∼

1

2

CF CD DF

CB CA AB⇒ = = = , de unde AF şi BD sunt mediane

în ABC∆ . Rezultǎ cǎ O este centrul de greutate al ABC∆ şi fiind O centrul cercului circumscris triunghiului rezultǎ cǎ ABC∆ este echilateral. Prin urmare O este ortocentrul ABC∆ , adicǎ AF si BD sunt înǎlŃimi ale triunghiului ABC.

7. 33 : Un patrulater convex are exact n unghiuri congruente şi exact n laturi congruente. Sǎ se determine cel mai mic numǎr n astfel încât patrulaterul sǎ devinǎ pǎtrat.

Prof. Mina Merlǎ, Şcoala łuŃora; Prof. Ioan Sǎcǎleanu, Hîrlău SoluŃie : Evident, avem { }2;3;4n∈ . Un trapez isoscel are 2 laturi congruente şi 2 unghiuri

congruente, dar nu este suficient ca el sǎ devinǎ pǎtrat. Prin urmare 2n ≠ . Pentru cazul 3n = , vom considera patrulaterul convex ABCD având fixate congruenŃele DA AB BC≡ ≡ . Se disting douǎ situaŃii în funcŃie de cele trei unghiuri congruente şi anume : � Dacǎ A B D≡ ≡∢ ∢ ∢ sau A B C≡ ≡∢ ∢ ∢ ( 2 unghiuri sunt formate din laturile congruente

fixate)atunci, din AD BC≡ , AB BA≡ şi BAD ABC≡∢ ∢LUL

BAD ABC≡⇒△ △ , de unde â

Page 45: De ce să învăŃăm la matematică? Word - 2011 Revista Mici matematicieni...coordonate, de axe ortogonale sau nu. Ceea ce înseamnă…matematică. Reintrând într-un registru

Micii MATEMATICIENI

45

BD AC≡ , la care adǎugând DC CD≡ şi BC AD≡ obŃinem LLL

BCD ADC≡△ △ . Rezultǎ cǎ

BCD ADC≡∢ ∢ . Astfel, patrulaterul ABCD are toate unghiurile congruente⇒ ABCD este dreptunghi, având şi douǎ laturi alǎturate congruente el devine pǎtrat. � Dacǎ A C D≡ ≡∢ ∢ ∢ sau B C D≡ ≡∢ ∢ ∢ ( un unghi format din laturile congruente fixate)

atunci, din AD BC≡ , DC CD≡ şi ADC BCD≡∢ ∢LUL

ADC BCD≡⇒△ △ , de unde AC BD≡ ,

la care adǎugând AB BA≡ şi BC AD≡ obŃinem LLL

ABC BAD≡△ △ . Rezultǎ cǎ

ABC BAD≡∢ ∢ . Astfel, patrulaterul ABCD are toate unghiurile congruente⇒ ABCD este dreptunghi, având şi douǎ laturi alǎturate congruente el devine pǎtrat.

Prin urmare cel mai mic numǎr cu proprietatea cerutǎ este 3n = . ObservaŃie : Cazul 4n = este valabil în mod evident (definiŃia pǎtratului).

7. 34 : Fie ABCD un patrulater convex cu diagonalele congruente. Sǎ se demonstreze cǎ ABCD

este trapez isoscel dacǎ şi numai dacǎ ( )( )

( )( )

m CAD m CBD

m ACB m BDA=

∢ ∢

∢ .

Prof. Ioan Sǎcǎleanu, Liceul ” Ştefan cel Mare”, Hirlău SoluŃie : Cu notaŃiile din figurǎ obŃinem : ( )1a c b d+ = + cǎci

[ ] [ ]AC BD≡ , iar x z y t+ = +∡ ∡ ∡ ∡ ( )2 cǎci

unghiurile opuse la vârf sunt AOD BOC≡∢ ∢ . " ":⇒ Presupunem cǎ ABCD este trapez isoscel. Atunci se disting cazurile: � Dacǎ AD BC� atunci x y=∡ ∡ şi t z=∡ ∡ (alt.

int.) Deci ( )( )

( )( )

1m CAD m CBDx t

m ACB y z m BDA= = = =

∢ ∢∡ ∡

∢ ∡ ∡

� Dacǎ AB CD� atunci OA OB

OC OD= (Thales în

ODC△ ), de unde ( )1

;OA OB OD OC⇒ ≡ ≡ şi cum AOD BOC≡∢ ∢ (opuse la vârf) . . .L L L

AOD BOC⇒ ≡△ △ . ObŃinem cǎ x t=∡ ∡ şi z y=∡ ∡ . Deci

( )( )

( )( )

m CAD m CBDx t

m ACB y z m BDA= = =

∢ ∢∡ ∡

∢ ∡ ∡.

" ":⇐ Presupunem cǎ ( )( )

( )( )

.notm CAD m CBD x tk

m ACB m BDA y z= ⇒ = =

∢ ∢ ∡ ∡

∢ ∡ ∡ x k y⇒ = ⋅∡ ∡ şi t k z= ⋅∡ ∡ .

Înlocuim în ( )2 ( ) ( )1 1 0k y z y k z k y k z⋅ + = + ⋅ ⇔ − ⋅ − − ⋅ =∡ ∡ ∡ ∡ ∡ ∡ ( ) ( )1 0k y z⇔ − ⋅ − =∡ ∡

� Dacǎ 1k = atunci x y=∡ ∡ DAC ACB⇒ ≡∢ ∢ şi cum sunt unghiuri alterne interne formate de AD şi BC cu secanta AC deducem cǎ AD BC� . Rezultǎ cǎ ABCD este trapez cu diagonalele congruente. Deci ABCD este trapez isoscel.

� Dacǎ ( )2

z y x t= ⇒ =∡ ∡ ∡ ∡ atunci AOD BOC△ ∼△ (cazul 2 de asemǎnare) ⇒OA OD

OB OC= , de

unde .nota dx a b x

b c= = ⇒ = ⋅ şi d c x= ⋅ . Înlocuim în ( )1 : b x c b c x⋅ + = + ⋅ ( )( )1 0x b c⇔ − − = .

A

B

CD

Oa

b

cd

x∡

y∡z∡

t∡

Page 46: De ce să învăŃăm la matematică? Word - 2011 Revista Mici matematicieni...coordonate, de axe ortogonale sau nu. Ceea ce înseamnă…matematică. Reintrând într-un registru

Micii MATEMATICIENI

46

� Dacǎ 1x = atunci a b AOB= ⇒ △ isoscel şi d c DOC= ⇒△ isoscel. Avem

( ) ( ) ( ) ( )0 0. .180 180

2 2

op vfm AOB m DOCm CAB m ACD

− −= = =

∢ ∢∢ ∢ CAB∢ şi cum ACD∢ ,

alterne interne rezultǎ cǎ AB CD� şi cum [ ] [ ]AC BD≡ obŃinem cǎ ABCD este trapez isoscel.

� Dacǎ b c COB= ⇒ △ isoscel şi , din ( )1 rezultǎ d a DOA= ⇒△ isoscel. Avem

( ) ( ) ( ) ( )0 0. .180 180

2 2

op vfm AOD m BOCm CAD m ACB

− −= = =

∢ ∢∢ ∢ şi cum CAD∢ şi ACB∢ ,

alterne interne rezultǎ cǎ AD CB� şi cum [ ] [ ]AC BD≡ obŃinem cǎ ABCD este trapez isoscel.

Clasa a VIII-a

8. 22 : Se dă mulŃimea { }3 5 4A x x= ∈ − − =ℤ şi funcŃia :f A B→ , ( ) 13

2f x x= −

a) DeterminaŃi mulŃimea B ştiind că ea are un număr minim de elemente. b) ReprezentaŃi grafic funcŃia f , determinată mai sus.

Prof. Spiridon Ana Marioara , Şcoala ,, Iordachi Cantacuzino’’, Paşcani SoluŃie : a) Din { } { }3 5 4 3 5 4 3 9;1 3 9; 1x x x x⇒ − − = ± ⇒ − = ± ⇒ − ∈ ⇒ − ∈ ± ± . Se obŃine

cǎ { }12; 6;4;2x∈ − ⊂ ℤ . Prin urmare domeniul de definiŃie al funcŃiei f este { }12; 6;4;2A = − .

Cum codomeniul funcŃiei are un numǎr minim de elemente şi cum mulŃimea valorilor funcŃiei

( )f A B⊆ rezultǎ cǎ ( )B f A= , adicǎ ( ) ( ) ( ) ( ){ }6 ; 2 ; 4 ; 12B f f f f= − . Deci

{ }6; 2; 1;3B = − − − .

b) Cum A este finitǎ rezultǎ cǎ graficul funcŃiei este ( ) ( ) ( ) ( ){ }6; 6 , 2; 2 , 4; 1 , 12;3fG = − − − − a

cǎrei reprezentare graficǎ reprezintǎ 4 puncte coliniare. Reprezentarea graficǎ efectivǎ se realizeazǎ uşor (exerciŃiu!).

8. 23 : DeterminaŃi n∈ℤ pentru care A să fie cel mai mare număr natural, pătrat perfect

mai mic ca 2010 , unde 2 22 1 1 8 1

: 24 3 12 6 2

n n n n n nA

− + + + += − + + ⋅

.

Prof. Nicolae Ivăşchescu, Colegiul NaŃional ” FraŃii Buzeşti”, Craiova SoluŃie : Cel mai mare pǎtrat perfect mai mic ca 2010 este 21936 44= . Avem 1936A = .

Aducem la forma mai simplǎ pe 2 26 3 4 4 8 6

212 1 2

n n n n nA

n

− − − + + / += ⋅ + ⋅ / +

, de unde gǎsim

( )( )2 12 1

22 1 2

n nn nA

n

⋅ ++ += + ⋅

+

( )( )

( ) ( ) ( )2

1 12 1 1

2 1 2

n n nA A n n n

n

/ + ⋅ +⇔ = + ⋅ ⇔ = + + +

+ .

Prin urmare ( )21 1 1936 1 1935A n A n n n= + ⇔ = + ⇔ = + ⇔ = .

8. 24 : Un elev coloreazǎ complet cu trei culori suprafaŃa unei planşe de desen având o formǎ de dreptunghi cu dimensiunile mai mari de 6 cm. Sǎ se demonstreze cǎ existǎ douǎ vârfuri ale planşei colorate cu aceeaşi culoare situate la o distanŃǎ mai micǎ decât 8,5 cm. Prof. Ion Pǎtraşcu, prof. Nicolae Ivǎşchescu, Colegiul NaŃional ” FraŃii Buzeşti”, Craiova

Page 47: De ce să învăŃăm la matematică? Word - 2011 Revista Mici matematicieni...coordonate, de axe ortogonale sau nu. Ceea ce înseamnă…matematică. Reintrând într-un registru

Micii MATEMATICIENI

47

SoluŃie : Deoarece planşa are dimensiunile mai mari de 6 cm, se poate construi un pǎtrat cu latura egalǎ cu 6 cm. Cum penru colorarea celor 4 vârfuri ale pǎtratului s-au folosit numai 3 culori rezultǎ, conform principiului lui Dirichlet (a cutiei) cǎ existǎ cel puŃin 2 vârfuri colorate cu aceeaşi culoare. Dacǎ aceste 2 vârfuri sunt alǎturate atunci distanŃa dintre ele este de 6 cm, deci mai micǎ decât 8,5 cm. Dacǎ aceste vârfuri, colorate cu aceeaşi culoare sunt opuse atunci

distanŃa dintre ele este 6 2 8,46≃ , care este mai micǎ decât 8,5 cm.

8. 25 : Numerele întregi a şi b validează egalitatea: 2 2

2 2

2

1 1 1

a b ab

a b ab+ =

+ + + . DemonstraŃi că

produsul numerelor a şi b este un pătrat perfect. Prof. Ioan Sǎcǎleanu, Liceul ” Ştefan cel Mare”, Hirlău

SoluŃie :Aducem la acelaşi numitor: ( )( ) ( )( ) ( )( )2 2 2 2 2 21 1 1 1 2 1 1a b ab b a ab ab a b+ + + + + = + +

ÎnmulŃim parantezele: ( )( ) ( ) ( ) ( )2 2 2 2 2 2 2 2 2 21 1 2 1a b a ab a b b ab ab a b b a+ + + + + = + + + 2 2 2 3 3 3 2 2 2 3 3 3 3 3 3 32 2 2 2a b a a b a b a b b a b ab a b ab a b ab⇔ + + + + + + + = + + + . Reducem termenii

asemenea : 2 2 2 2 3 32 2 0a b a b ab a b ab+ + − − − = . Descompunem în factori membrul stâng :

( ) ( ) ( )2 21 1 2 1 0a ab b ab ab ab− + − − − = ( )( )2 21 2 0ab a b ab⇔ − + − = .

� Dacǎ ( )22 2 2 0 0a b ab a b a b+ − = ⇔ − = ⇔ = atunci 2a b a⋅ = este pǎtrat perfect.

� Dacǎ 1 0a b− ⋅ = atunci 21a b⋅ = este pǎtrat perfect. 8. 26 : Pe planul trapezului dreptunghic ABCD cu bazele AB = 2dm, CD = 6dm şi înălŃimea AD = 4 3 dm se ridică perpendiculara DE, DE = 8dm. Fie M ( )BC∈ astfel încât BM = 2dm.

a) DeterminaŃi distanŃa de la E la AB b) GăsiŃi lungimea segmentului [EM] c) ArătaŃi că ( )AM EDM⊥

Prof. Bogdan Dorneanu, Şcoala “Petru Rareş”, Hîrlău SoluŃie : Ducem înǎlŃimea BS în trapezul dreptunghic ABCD. Rezultǎ cǎ ABSD este

dreptunghi, de unde 2DS AB= = 6 2 4SC DC DS⇒ = − = − = şi 4 3BS AD= = . Avem

( )090BSC S =△ ∢. .

2 2 2 8T P

BC BS SC BC⇒ = + ⇒ = .Deoarece ( ) ( ) 03sin 60

2

BSBCS m BCS

BC= = ⇒ =∢ ∢

Dar, DCM△ este isoscel cǎci 6DC CM= = , deci el este echilateral 6DM⇒ = ( )1 şi mai

mult ( ) ( ) 060m C m DMC= =∢ ∢ . Dar, ( ) ( )0 0180 120m B m C= − =∢ ∢ şi cum ABM△ este

isoscel ( )2AB BM= = obŃinem cǎ ( ) ( )00180

302

m Bm AMB

−= =

∢∢

Din BMC∢ unghi alungit se determinǎ ( ) ( ) ( )0 0180 90m AMD m AMB m DMC= − − =∢ ∢ ∢ ( )2 .

a) Din ( )ED ABCD⊥ şi ( )3

;T

DA AB EA AB d E AB EA⊥

⊥ ⇒ ⊥ ⇒ = ,

iar din ( )ED ABCD⊥ şi ( )DA ABCD⊂ rezultǎ cǎ ED DA⊥ ⇒

EDA△ dreptunghic în D . .

2 2 2 24 7T P

EA ED DA⇒ = + = ⋅ .

Prin urmare ( ); 4 7d E AB EA= = .

b) Din ( )ED ABCD⊥ şi ( )DM ABCD⊂ rezultǎ cǎ ( )3ED DM⊥ M

A B

CD

E

S

Page 48: De ce să învăŃăm la matematică? Word - 2011 Revista Mici matematicieni...coordonate, de axe ortogonale sau nu. Ceea ce înseamnă…matematică. Reintrând într-un registru

Micii MATEMATICIENI

48

⇒ EDM△ dreptunghic în D . .

2 2 2T P

EM ED DM⇒ = +

( )1

2 2 28 6 10EM EM⇒ = + ⇒ = . c) Din ( )ED ABCD⊥ şi ( )AM ABCD⊂ rezultǎ cǎ ED AM⊥ şi cum AM DM⊥ din ( )2

rezultǎ ( )AM EDM⊥ .

8. 27 : Să se rezolve în mulŃime ℝ sistemul de ecuaŃii: 2 2

6

13

xy

x y

=

+ = .

Prof. LaurenŃa Doca, Liceul „Ion Neculce”, Tg. Frumos

SoluŃie :Avem 2 2

2 12

13

xy

x y

=

+ = . Adunând şi scăzând ecuaŃiile între ele se obŃine sistemul

( )( )

22 2

2 2 2

252 25

2 1 1

x yx y xy

x y xy x y

+ = + + = ⇔

+ − = − =

5

1

x y

x y

+ = ±⇔

− = ±. Fiecare dintre aceste 4 sisteme are câte o

soluŃie. Fie ( );a b una dintre ele. Se observǎ cǎ şi ( );a b− − este soluŃie a sistemului iniŃial şi

chiar mai mult şi ( );b a şi ( );b a− − cǎci înlocuind pe x cu y şi invers se obŃine acelaşi sistem

(sistem simetric). Rǎmâne sǎ rezolvǎm un singur sistem dintre cele 4 şi fie acesta 5

1

x y

x y

+ =

− =.

Adunând cele douǎ ecuaŃii ale sistemului obŃinem 2 6 3x x= ⇒ = , iar din prima ecuaŃie 3 5 2y y+ = ⇒ = .Prin urmare mulŃimea soluŃiilor ( ) ( ) ( ) ( ){ }3;2 , 3; 2 , 2;3 , 2; 3S = − − − − .

8. 28 : Să se arate că tetraedrul care are lungimile muchiilor laterale egale cu 3 şi care fac

între ele unghiuri de 060 , 090 , respectiv 0120 au două înălŃimi egale. Prof. Turnea Mihaela, Liceul “I.Neculce”, Tg.Frumos

SoluŃie : Pentru a arǎta cǎ tetraedrul are 2 înǎlŃimi egale este suficient ca sǎ gǎsim 2 feŃe de arii

egale (din formula volumului). Fie VABC un tetraedru cu muchiile laterale 3VA VB VC= = = ,

( ) 060m AVB =∢ , ( ) 0120m CVB =∢ şi ( ) 090m AVC =∢ . Cum VAB△ este isoscel cu un unghi

de 060 rezultǎ cǎ el este echilateral, de unde 2 3 3 3

4 4VAB

lA = =△ . Ducem ( ),BO VC V OC⊥ ∈ .

În triunghiul dreptunghic VOB△ avem : ( ) 0 3sin sin 60

23

BO BOBVO BO

VB= ⇒ = ⇒ =∢ . Atunci

aria 1 1 3 3 3

32 2 2 4VBCA BO VC= ⋅ ⋅ = ⋅ ⋅ =△ . Prin urmare înǎlŃimea din C este egalǎ cu cea din A .

Nota redacŃiei : Pentru a fi îndeplinitǎ cerinŃa problemei nu era necesar sǎ fie precizate în ipotezǎ nici unghiul de 090 şi nici valoarea muchiilor laterale.

8. 29 : Fie cubul ABCDEFGH de muchie a, M simetricul punctului A faŃă de B, N piciorul perpendicularei din C pe [BH], iar P centrul pătratului ADHE. ArătaŃi că punctele M, N, P sunt coliniare.

Prof. Bogdan Dorneanu, Şcoala “Petru Rareş”, Hîrlău SoluŃie : Din P centrul pătratului ADHE rezultǎ cǎ P este mijlocul lui [ ]AH , iar din M

simetricul punctului A faŃă de B avem cǎ B este mijlocul lui [ ]AM , de unde PB este linie

mijlocie în HAM△ PB HM⇒ � . Notǎm cu S , intersecŃia dintre [ ]BH şi [ ]PM . Aplicǎm

Page 49: De ce să învăŃăm la matematică? Word - 2011 Revista Mici matematicieni...coordonate, de axe ortogonale sau nu. Ceea ce înseamnă…matematică. Reintrând într-un registru

Micii MATEMATICIENI

49

teorema fundamentalǎ în HSM△ cu PB HM ⇒� 1

2

BS PB

SH MH= =

3

BHBS⇒ = ( )1 . Cum

BCH△ este dreptunghic în C şi N, proiecŃia lui C pe BH , rezultǎ cu teorema catetei cǎ

( )2 2

22

2

BC BN BH l BN

CH HN BH HNl

⋅= ⇒ =

1

2

BN

HN⇒ =

3

BHBN⇒ = ( )2 . Din ( )1 şi ( )2 rezultǎ cǎ punctele

N şi S coincid. Cum M, S, P sunt coliniare rezultǎ cǎ şi punctele M, N, P sunt coliniare. ObservaŃie: Se poate rezolva problema şi aplicând reciproca teoremei lui Menelaos sau arǎtând cǎ N este centrul de greutate al HAM△ , dar în esenŃǎ se reduce

tot la determinarea raportului BN

HN.

8. 30 : DeterminaŃi x din proporŃia : 2

2

a x b

b a x

−=

+ ştiind cǎ

4 220082011 2 20082010 4 20082011 1a = + ⋅ + ⋅ − şi 4 220082011 2 20082012 4 20082011 3b = − ⋅ + ⋅ +

Prof. Mina Merlǎ, Şcoala łuŃora, Iaşi

Din proporŃia 2

2

a x b

b a x

−=

+ rezultǎ ( )( )2 2a x a x b− + = 24a x b⇔ − = 2

4

a bx

−⇔ = .

SoluŃia 1 : Pentru simplificare vom nota pe 20082011 cu t . Atunci ( )24 2 1 4 1a t t t= + ⋅ − + − ,

de unde ( ) ( ) ( )2 24 2 4 2 2 2 2 22 2 1 4 1 2 4 2 4 1 2 1 1 1a t t t t t t t t t t t/ /= + ⋅ − + + − = + − + + − = + ⋅ ⋅ + = +/ / ,

iar ( )24 2 1 4 3b t t t= − ⋅ + + + ( ) ( )2 24 2 2 2 2 22 4 2 4 3 2 1 1 1t t t t t t t/ /= − − − + + = − ⋅ ⋅ + = −/ / . Se obŃine

( ) ( )4 2 4 2 22 1 2 1 4a b t t t t t− = + + − − + = ObŃinem cǎ 2

2 2 4

4 4

a b tx x

/−= ⇔ =

/2 2x t x t⇔ = ⇔ = ± .

SoluŃiile ecuaŃiei sunt 20082011x = ± . SoluŃia 2 (Buzilǎ Bianca, cl. a VIII-a): Avem 2 22 20082010 2 20082012 4a b− = ⋅ + ⋅ − . Scriem pe 20082010 20082011 1= − şi 20082012 20082011 1= + şi dezvoltând cu pǎtratul binomului,

obŃinem ( )2 22 20082011 2 20082011 1 20082011 2 20082011 1 2a b / / /− = ⋅ − ⋅ + + + ⋅ + − , de unde 24 20082011a b− = ⋅ . Deci 2 220082011 20082011x x= ⇔ = ± .

8. 31 : Sǎ se arate cǎ 22 2

2 2

x y x yx y

+ + ⋅ ⋅ ≤

pentru orice , 0.x y ≥

Cristina Ene, elevǎ, Colegiul NaŃional “FraŃii Buzeşti”, Craiova SoluŃie : Notǎm x y b⋅ = şi x y a+ = . Rezultǎ cǎ 2 2 22a x xy y= + + , de unde 2 2 2 2x y a b+ = − .

Avem:22 2

2 2

x y x yx y

+ + ⋅ ⋅ ≤

22 2

2 2

a b ab

− ⇔ ⋅ ≤

(ridicǎm la pǎtrat)( )2 42

2 16

a b b a− ⋅⇔ ≤

⇔ ( ) ( ) ( )2 222 2 4 2 2 28 16 2 4 4 0 4 0a b b a a a b b a b− ≤ ⇔ − ⋅ ⋅ + ≥ ⇔ − ≥ , evident adevǎratǎ.

A B

H

E

G

F

DC

NM

P

Page 50: De ce să învăŃăm la matematică? Word - 2011 Revista Mici matematicieni...coordonate, de axe ortogonale sau nu. Ceea ce înseamnă…matematică. Reintrând într-un registru

Micii MATEMATICIENI

50

8. 32 : Fie numerele pozitive x , y şi z . DemonstraŃi cǎ , ,x y z sunt lungimile laturilor unui

triunghi ascuŃitunghic dacǎ şi numai dacǎ existǎ , ,a b c ∗+∈ℝ astfel încât

2 2 2

2 2 2

2 2 2

x a b

y a c

z c b

= +

= + = +

.

Prof. Ioan Sǎcǎleanu, Liceul ” Ştefan cel Mare”, Hirlău SoluŃie : " ":⇒ Presupunem cǎ , ,x y z sunt lungimile laturilor unui triunghi ascuŃitunghic, pe care

îl notǎm ABC△ cu AB x= , BC z= şi AC y= . Ortocentrul

1 1H AA BB∈ ∩ se aflǎ în interiorul ABC△ . Avem 1AA BC⊥

1BB AC⊥ . Notǎm cu V, intersecŃia dintre perpendiculara în H

planul ( )ABC cu sfera de diametru de diametru [ ]1AA . Avem

( )VH ABC⊥ . Am obŃinut existenŃa lui VABC şi implicit

existenŃa numerelor , ,a b c ∗+∈ℝ prin , ,a VA b VB c VC= = = .

Vom demonstra cǎ aceste numere verificǎ relaŃiile din enunŃ. Pentru aceasta vom demonstra cǎ tetraedrul construit este tridreptunghic în V, adicǎ VA VB VC VA⊥ ⊥ ⊥ . Din ( )VH ABC VH BC⊥ ⇒ ⊥ cǎci ( )BC ABC⊂ , dar cum şi

1AA BC⊥ deducem cǎ ( )1BC AVA⊥ , de unde AV BC⊥ . Din 1AVA∢ unghi înscris într-un

semicerc de diametru [ ]1AA (din construcŃia fǎcutǎ) 1AV VA⊥ şi cum AV BC⊥

( )AV VBC⇒ ⊥ , de unde VA VB⊥ şi VA VC⊥ . Mai rǎmâne sǎ arǎtǎm cǎ VB BC⊥ . Într-

adevǎr, din ( )VH ABC VH AC⊥ ⇒ ⊥ şi cum 1BB AC⊥ ( )1AC VBB AC VB⇒ ⊥ ⇒ ⊥ . Avem

VB AC⊥ şi VB VA⊥ ( )VB VAC⇒ ⊥ şi cum ( )VC VAC VB VC⊂ ⇒ ⊥ . Prin urmare tetraedrul

VABC este tridreptunghic în V. Din VA VB⊥ VAB⇒△ este dreptunghic în V . .

2 2 2T P

AB VA VB⇒ = + 2 2 2x a b⇒ = + . Din VA VC⊥ VAC⇒△ este dreptunghic în V . .

2 2 2T P

AC VA VC⇒ = + 2 2 2y a c⇒ = + . Din VC VB⊥ VCB⇒△ este dreptunghic în V . .

2 2 2T P

BC VB VC⇒ = + 2 2 2z b c⇒ = + ( ). . .q e d .

" ":⇐ Presupunem cǎ existǎ , ,a b c ∗+∈ℝ astfel încât 2 2 2x a b= + , 2 2 2y a c= + şi 2 2 2z c b= + .

Construim tetraedrul VABC tridreptunghic în V astfel încât , ,VA a VB b VC c= = = . Din

VA VB⊥ VAB⇒△ este dreptunghic în V . .

2 2 2T P

AB VA VB⇒ = + 2 2 2 2AB a b x AB x⇒ = + = ⇒ = .

Din VA VC⊥ VAC⇒△ este dreptunghic în V . .

2 2 2T P

AC VA VC⇒ = + 2 2 2 2AC a c y AC y⇒ = + = ⇒ = . Din VC VB⊥ VCB⇒△ , dreptunghic în V

. .2 2 2

T P

BC VB VC⇒ = + 2 2 2 2BC b c z BC z⇒ = + = ⇒ = . Vom demonstra cǎ ABC△ este ascuŃitunghic. Fie 1VA înǎlŃime în VBC△ , dreptunghic în V.

Rezultǎ cǎ ( )1A BC∈ ( ) ( )1VA Int ABC⇒ ⊂ △ . Din VB VA VC⊥ ⊥ ( )AV VBC⇒ ⊥ şi cum

1VA BC⊥ obŃinem cu teorema celor 3 perpendiculare cǎ 1AA BC⊥ , adicǎ 1AA înǎlŃime în

ABC△ . Fie 1VB înǎlŃime în VAC△ . Analog, se aratǎ cǎ 1BB înǎlŃime în ABC△ şi

V

A

1A

1B

B

C

H

a

b

c

x y

z

Page 51: De ce să învăŃăm la matematică? Word - 2011 Revista Mici matematicieni...coordonate, de axe ortogonale sau nu. Ceea ce înseamnă…matematică. Reintrând într-un registru

Micii MATEMATICIENI

51

( ) ( )1VB Int ABC⊂ △ . Din teorema transversalei rezultǎ cǎ

( ) ( ) ( )1 1H AA BB H Int ABC∈ ∩ ⇒ ∈ △ şi cum H este ortocentru deducem cǎ ABC△ este

ascuŃitunghic.

MATEMATICA LICEALĂ Clasa a IX-a

9. 24 : Sǎ se determine x∈ℝ pentru care valoarea raportului 2

2

30 10

30 10

x x

x x

− −+ +

sǎ fie maximǎ.

Prof. Aurel Neicu, Liceul ” Ştefan cel Mare”, Hîrlău

SoluŃia 1 : Avem ( )22 2 210 30 2 5 5 5 5 5x x x x x+ + = + ⋅ ⋅ + + = + + . Rezultǎ cǎ cea mai micǎ

valoare a numitorului 2 10 30x x+ + este 5 şi se realizeazǎ dacǎ ( )25 0x + = , adicǎ 5x = − .

Avem ( ) ( )22 2 210 30 2 5 5 55 5 55x x x x x− − + = − + ⋅ ⋅ + − = − + + . Rezultǎ cǎ cea mai mare

valoare a numǎrǎtorului 230 10x x− − este 55 şi se realizeazǎ dacǎ ( )25 0x + = , adicǎ 5x = − .

Prin urmare, pentru 5x = − se obŃine valoarea maximǎ a raportului 2

2

30 10

30 10

x x

x x

− −+ +

, adicǎ

5511

5= .

SoluŃia 2 : Pentru orice x∈ℝ avem 230 10 0x x+ + ≠ cǎci are discriminantul negativ. Putem

defini funcŃia :f →ℝ ℝ , ( )2

2

30 10

30 10

x xf x

x x

− −=

+ + . Problema se reduce la aflarea lui maxx . Pentru

aceasta vom determina imaginea funcŃiei. Fie Imy f∈ . Rezultǎ cǎ existǎ x∈ℝ astfel încât 2

2

30 10

30 10

x xy

x x

− −=

+ +, de unde ecuaŃia ( ) ( ) ( )21 10 1 30 1 0y x y x y+ ⋅ + + ⋅ + − = are soluŃii reale în x

cu parametru y ( ) ( )22 20 10 1 4 30 1 0y y y⇔∆ ≥ ⇔ + − ⋅ − ≥ 2 2100 200 100 120 120 0y y y⇔ + + − + ≥ 2 220 200 220 0 10 11 0y y y y⇔ − + + ≥ ⇔ − + + ≥ [ ]1 2;y y y⇔ ∈ [ ]1;11y⇔ ∈ − .Deci [ ]Im 1;11f = −

Valoarea maximǎ a raportului este max 11f = . Pentru 11y = ⇒ 0y∆ = , iar

( )( )max

10 15

2 1

yx

y

− += = −

+.

9. 25 : Fie numerele reale pozitive x, y ,z astfel încât 1x y z+ + = .Să se arate că:

4 4 43

(1 )(1 ) (1 )(1 ) (1 )(1 )

xy xz yz

x y x z y z+ + ≥

− − − − − −.

Prof. Constantin Chirilă, Liceul “G. Ibrăileanu”, Iaşi SoluŃie : Din , , 0x y z > şi 1x y z+ + = rezultǎ cǎ 1 ,1 ,1 0x y z− − − > . Eliminând numitorii,

inegalitatea cerutǎ devine ( ) ( ) ( ) ( ) ( )( )4 1 4 1 4 1 3 1 1 1xy z xz y yz x x y z− + − + − ≥ − − − . Efectuând

calculele se ajunge la : ( ) ( )4 4 4 4 4 4 3 3 1xy xyz xz xyz yz xyz x y z yz− + − + − ≥ − − − +

4 4 4 12 3 3 3 3 3 3 3 3xy xz yz xyz y z yz x xy xz xyz⇔ + + − ≥ − − + − + + − , care este echivalentǎ cu

Page 52: De ce să învăŃăm la matematică? Word - 2011 Revista Mici matematicieni...coordonate, de axe ortogonale sau nu. Ceea ce înseamnă…matematică. Reintrând într-un registru

Micii MATEMATICIENI

52

( )9 3 3xy xz yz xyz x y z+ + − ≥ − + + . łinând cont de ipotezǎ şi apoi împǎrŃind cu xyz

obŃinem : 9xy xz yz xyz+ + ≥ ⇔ 1 1 1

9x y z+ + ≥ 9

x y z x y z x y z

x y z

+ + + + + +⇔ + + ≥ , de unde

gǎsim ⇔ 1 1 1 9y z x z x y

x x y y z z+ + + + + + + + ≥ 6

y x z x z y

x y x z y z

⇔ + + + + + ≥

. Fiecare

paratezǎ este 2≥ fiind suma dintre un numǎr şi inversul sǎu, iar prin însumarea lor obŃinem cǎ inegalitatea precedentǎ este adevǎratǎ. Aşadar este adevǎratǎ şi echivalenta sa, cea din enunŃ.

9. 26 : Să se demonstreze că , dacă 2 2 2

DEF

MNP

ADE EF DF

MN NP MP A∆

≥ ≥ ≥

atunci triunghiurile

ascuŃitunghice DEF△ şi MNP△ sunt triunghiuri asemenea. Prof. Ioan Sǎcǎleanu, Liceul ” Ştefan cel Mare”, Hirlău

SoluŃie : Notǎm cu sin

0sin

Dx

M= > ;

sin0

sin

Ey

N= > şi

sin0

sin

Fz

P= > . Aplicând teorema

sinusurilor rezultǎ cǎ / /

2 sin

2 sin

DE R F Rz

MN R P R

⋅= = ⋅

⋅, unde R şi /R sunt razele cercurilor

circumscrise DEF△ , respectiv MNP△ . Analog , se obŃine cǎ /

EF Rx

NP R= ⋅ şi

/

DF Ry

MP R= ⋅ .

Folosind formula 22 sin sin sinDEFA R D E F∆ = ⋅ ⋅ obŃinem cǎ 2 2

/ /

sin sin sin

sin sin sinDEF

MNP

A R D E F Rx y z

A R M N P R∆

= ⋅ ⋅ ⋅ = ⋅ ⋅ ⋅

. Atunci inegalitatea din enunŃ devine,

dupǎ împǎrŃirea cu 2

/

R

R

echivalentǎ cu 2 2 2z x y xyz≥ ≥ ≥ . De aici, avem z x y≥ ≥ şi xz y≤ .

Atunci şi xz x≤ , de unde 1z ≤ şi deci 1y x z≤ ≤ ≤ . Din 1x ≤ , 1y ≤ şi 1z ≤ obŃinem cǎ sin sinD M≤ , sin sinE N≤ şi sin sinF P≤ . Deoarece triunghiurile sunt ascuŃitunghice, iar

funcŃia sinus este crescǎtoare pe 0;2

π

obŃinem ( ) ( )m D m M≤∢ ∢ , ( ) ( )m E m N≤∢ ∢ şi

( ) ( )m F m P≤∢ ∢ . Dacǎ una dintre aceste inegalitǎŃi ar fi stricte atunci am avea

( ) ( ) ( ) ( ) ( ) ( ) 0 0180 180m D m E m F m M m N m P+ + < + + ⇔ <∢ ∢ ∢ ∢ ∢ ∢ , absurd. Prin urmare

( ) ( )m D m M=∢ ∢ , ( ) ( )m E m N=∢ ∢ , deci cele douǎ triunghiuri sunt asemenea.

9. 27 : Sǎ se rezolve în mulŃimea numerelor reale ecuaŃia [ ] { }2009 20092009x x x+ = , unde prin

[ ]x şi { }x înŃelegem partea întreagǎ, respectiv partea fracŃionarǎ a numǎrui real x .

Prof. Dr. Cristian Dinu , Colegiul NaŃional “Carol I”, Craiova SoluŃie : Dacǎ [ )0,1x∈ atunci [ ] 0x = şi { }x x= . EcuaŃia devine 2009 2009 20090x x+ = , adevǎratǎ.

Dacǎ 1x ≥ atunci [ ] 1x ≥ . Avem [ ]20092009 2x x+ ≥ ⇒ { }20092x ≥ , în contradicŃie cu { } [ )0,1x ∈ .

Dacǎ 0x < atunci [ ] 0x < . Avem [ ]20092009 0x x+ < ⇒ { }20090x < , în contradicŃie cu

{ } [ )0,1x ∈ .

Prin urmare, mulŃimea soluŃiilor este [ )0,1S = .

Page 53: De ce să învăŃăm la matematică? Word - 2011 Revista Mici matematicieni...coordonate, de axe ortogonale sau nu. Ceea ce înseamnă…matematică. Reintrând într-un registru

Micii MATEMATICIENI

53

9. 28 : Fie numǎrul real x astfel încât 2x x− ∈ℚ şi 3x ∈ℚ . ArǎtaŃi cǎ x∈ℚ .

Prof. Ion Pǎtraşcu, Colegiul NaŃional “FraŃii Buzeşti”, Craiova SoluŃia 1 : Considerǎm x∈ℝ astfel încât 2x x p− = şi 3x q= unde p∈ℚ şi q∈ℚ . Avem

3 2q p x x x− = − + ( ) ( )2 1 1q p x x x q p x p⇔ − = ⋅ − + ⇔ − = ⋅ + ( )∗ .

Vom demonstra, prin reducere la absurd cǎ 1p ≠ − . Presupunem prin absurd cǎ 1p = − atunci

numǎrul x∈ℝ verificǎ 2 1x x− = − , de unde ecuaŃia 2 1 0x x− + = are soluŃii reale, adicǎ 0∆ ≥ , dar 2 4 3b ac∆ = − = − , aşadar 3 0− ≥ , absurd. Prin urmare 1p ≠ − 1 0p⇔ + ≠ . Din relaŃia ( )∗

gǎsim cǎ 1

q px

p

−= ∈

+ℚ fiind raportul dintre diferenŃa şi suma a douǎ numere raŃionale.

SoluŃia 2 (a redacŃiei) : Pentru 0x = problema se verificǎ uşor. Dacǎ 0x ≠ atunci raportul

numerelor raŃionale 2x x− ∈ℚ şi 3x ∈ℚ este raŃional, adicǎ 2

3

x x

x

−∈ℚ

2

31

x x

x

−⇒ − ∈ℚ

2 3

3

x x x

x

− −⇔ ∈ℚ

( )2

3

1x x x

x

− ⋅ + −⇔ ∈ℚ

2

2

1x x

x

+ −⇔ ∈ℚ , dar 2 1x x+ − ∈ℚ fiind diferenŃǎ

de numere raŃionale. Rezultǎ 2x ∈ℚ şi cum 3x ∈ℚ , atunci 3

2

xx

x= ∈ℚ ca raport de numere

raŃionale.

9. 29 : Fie ABC∆ şi punctele M, N, P astfel încât 1

3AM AB= ⋅����� ����

, 1

2CN CA=���� ����

şi 2BP BC=���� ����

.

DemonstraŃi cǎ 3NP MN=���� �����

. Prof. Aurel Neicu, Liceul ” Ştefan cel Mare”, Hîrlău

SoluŃia 1 (vectorial): łinând cont de datele problemei şi regula lui Chales de adunare a

vectorilor: 1

22

NP NC CB BP CN BC BC CA BC= + + = − − + = − +���� ���� ���� ���� ���� ���� ���� ���� ����

, de unde1

2NP AC BC= +������ ���� ����

(1)

iar ( ) 1 3 33 3 3 3 3 3

2 2 2MN MA AC CN AM AC CA AB AC AC AC AC CB= + + = − + + ⋅ = − + − = − −����� ���� ���� ���� ����� ���� ���� ���� ���� ���� ���� ���� ����

de unde 1

32

MN AC BC= +����� ���� ����

(2). Din (1) şi (2) rezultǎ cerinŃa cerutǎ.

SoluŃia 2 (sintetic) : DefiniŃia înmulŃirii unui vector cu scalari caracterizeazǎ sintetic relaŃiile

vectoriale date. Din 1

3AM AB= ⋅����� ����

rezultǎ cǎ ( )M AB∈ şi 3AB AM= , din 1

2CN CA=���� ����

gǎsim cǎ

N este mijlocul lui ( )AC , iar din 2BP BC=���� ����

C⇒ este mijlocul lui ( )BP . Trebuie demonstrat

cǎ punctele M N P− − sunt coliniare şi cǎ 3NP MN= . Pentru aceasta, considerǎm S , mijlocul lui ( )MP . Cum C este mijlocul lui ( )BP rezultǎ cǎ CS este linie mijlocie în BPM∆ , de unde

CS BM� CS AM⇒ � şi 2

BMCS = ⇒ 2BM CS= 2AB AM CS⇒ − = 3 2AM AM CS⇒ − =

AM CS⇒ = şi cum CS AM� , deducem cǎ AMCS este paralelogram . Din N este mijlocul

lui ( )AC rezultǎ cǎ N este mijlocul lui ( )MS , adicǎ M N P− − sunt coliniare şi mai mult,

14 4

2 2 2

MS MPMN MP MN MN NP MN= = ⋅ ⇒ = ⇒ + = ⇒ 3NP MN= . Deci 3NP MN=

���� �����.

Page 54: De ce să învăŃăm la matematică? Word - 2011 Revista Mici matematicieni...coordonate, de axe ortogonale sau nu. Ceea ce înseamnă…matematică. Reintrând într-un registru

Micii MATEMATICIENI

54

Clasa a X-a 10. 23: Punctele distincte 0A , 1A , 2A , … , 2009A se găsesc în spaŃiu la o distanŃă unele de altele

dată de formula , , , 0, 2009i jA A i j i j i j= + ∀ ≠ = . Câte dintre ele pot fi vârfurile unui triunghi

şi câte triunghiuri se pot forma ? « SoluŃia » autorului: Vom arǎta cǎ orice 3 astfel de puncte sunt coliniare dacǎ şi numai dacǎ unul dintre ele este 0A . Într-adevǎr, condiŃia de coliniaritate

pentru m n pA A A− − : m p m n n pA A A A A A= + este echivalentǎ cu m p m n n p+ = + + + 0 2n⇔ =

0n⇔ = , adicǎ oricare trei puncte, diferite de 0A sunt vârfurile unui triunghi ( )∗ . Prin urmare

se pot forma 32009C triunghiuri.

Nota redacŃiei (Prof. Ioan Sǎcǎleanu): Problema are câteva neajunsuri şi anume: • Din informaŃiile de mai sus rezultǎ cǎ 0 1 2, ,A A A , respectiv 0 1 3, ,A A A sunt coliniare. Prin

urmare 2 3,A A aparŃin dreptei 0 1A A , în contradicŃie cu relaŃia ( )∗ , anume 1 2 3, ,A A A sunt

necoliniare. Aşadar, problema are un “defect” de fond. • Pentru a “recura problema”, vom elimina din enunŃul problemei punctul 0A .

10. 24: Fie 1 2 3A A A∆ . Spunem că un punct P din spaŃiu este ortopunct asociat 1 2 3A A A∆ dacă

{ } { }, , , 1, 2,3i j kPA A A i j k⊥ = . ArătaŃi că următoarele afirmaŃii sunt echivalente:

(1) P este ortopunct asociat 1 2 3A A A∆ ;

(2) 2 2 2 2 2 21 2 3 2 1 3 3 2 1PA A A PA A A PA A A+ = + = + ;

(3) 1 2 3 2 1 3 3 1 2PA A A PA A A PA A A⋅ = ⋅ = ⋅���� ����� ����� ����� ���� �����

(4) 1 2 2 3 3 1PA PA PA PA PA PA⋅ = ⋅ = ⋅���� ����� ����� ���� ���� ����

.

Prof. Ioan Sǎcǎleanu, Liceul ” Ştefan cel Mare”, Hirlău

SoluŃie : Demonstrarea urmǎtoarelor implicaŃii 1 2

4 3

⇒⇑ ⇓⇐

, rezolvǎ problema. Din i j i jA A A A=����� �����

i j i k k jAP PA A A A A⇔ + = +���� ����� ����� ������

j i k k j iPA A A A A AP⇔ − = −����� ����� ������ ����

, pentru orice permutare { } { }, , 1, 2,3i j k = .

Ridicând la pătrat scalar, obŃinem: 2 2 2 22 2j j i k i k k j k j i iPA PA A A A A A A A A AP AP− ⋅ ⋅ + = − ⋅ ⋅ + ⇔����� ����� ������ ����

( ) ( ) { } { }2 2 2 2 2 ; , , 1, 2,3 ( )j i k i j k j i k i j kPA A A PA A A PA A A PA A A i j k+ − + = ⋅ ⋅ − ⋅ = ∗����� ����� ���� ������

1 2⇒ Presupunem că P este ortopunct asociat 1 2 3A A A∆ ⇒ 0i j kPA A A⋅ =���� ������

pentru orice

permutare { } { }, , 1,2,3i j k = . Cum 0i j k j i kPA A A PA A A⋅ = ⋅ =���� ������ ����� �����

rezultă din (1) că

2 2 2 2j i k i j kPA A A PA A A+ = + cu { } { }, , 1,2,3i j k = .

Pentru ( ) ( ), , 1, 2,3i j k = şi ( ) ( ), , 1,3,2i j k = rezultă relaŃia (2).

Prof. Petru Oprişanu, Hirlău

Page 55: De ce să învăŃăm la matematică? Word - 2011 Revista Mici matematicieni...coordonate, de axe ortogonale sau nu. Ceea ce înseamnă…matematică. Reintrând într-un registru

Micii MATEMATICIENI

55

2 3⇒ Din (2) avem 2 2 2 2j i k i j kPA A A PA A A+ = +

( )∗

⇒ j i k i j kPA A A PA A A⋅ = ⋅����� ����� ���� ������

cu

{ } { }, , 1, 2,3i j k = .Pentru ( ) ( ), , 2,1,3i j k = şi ( ) ( ), , 2,3,1i j k = avem 1 2 3 2 1 3PA A A PA A A⋅ = ⋅���� ����� ����� �����

şi

3 1 2 2 3 1PA A A PA A A⋅ = ⋅���� ����� ����� �����

, deci egalitatea (3).

3 4⇒ Din (3): { } { }; , , 1, 2,3j i k i j kPA A A PA A A i j k⋅ = ⋅ = ⇒����� ����� ���� ������ ( ) ( )j i k i j kPA AP PA PA A P PA⋅ + = ⋅ +

����� ���� ����� ���� ����� �����

j i j k i j i kPA AP PA PA PA A P PA PA⇒ ⋅ + ⋅ = ⋅ + ⋅ ⇒����� ���� ����� ����� ���� ����� ���� �����

{ } { }; , , 1,2,3j k i kPA PA PA PA i j k⋅ = ⋅ =����� ����� ���� �����

. Pentru

( ) ( ), , 1,3,2i j k = şi ( ) ( ), , 1, 2,3i j k = rezultă relaŃia (4).

4 1⇒ Din (4), obŃinem : { } { } cu , , 1,2,3j k i kPA PA PA PA i j k⋅ = ⋅ =����� ����� ���� �����

( ) =0j i kPA PA PA⇒ − ⋅����� ���� �����

0i j kA A PA⇒ ⋅ =����� �����

( ) ( ) pentru , , 1,2,3k i jPA A A i j k⇒ ⊥ = . Deci P este ortopunct asociat

1 2 3A A A∆ .

ObservaŃii : 1. Pentru orice triunghi există un singur ortopunct H , situat în planul triunghiului şi, anume

ortocentrul triunghiului.

2. Particularizând P H≡ obŃinem caracterizări ale ortocentrului 1 2 3A A A∆ .

3. MulŃimea ortopunctelor asociate 1 2 3A A A∆ este perpendiculara d în ortocentrul triunghiului

pe planul triunghiului.

" "⊆ Fie P H≠ un ortopunct asociat 1 2 3A A A∆ . Arătăm că ( )1 2 3PH A A A⊥ . Avem

( )i j k i j k j k i j kPA A A PH HA A A PH A A HA A A⋅ = + ⋅ = ⋅ + ⋅���� ������ ���� ���� ������ ���� ������ ���� ������

. Cum i j kPA A A⊥ şi i j kHA A A⊥

rezultă că 0i j k i j kPA A A HA A A⋅ = ⋅ =���� ������ ���� ������

. Rezultă că 0j kPH A A⋅ =���� ������

; deci

; ; , 1,3j kPH A A j k j k⊥ ≠ ∈ . Prin urmare ( )1 2 3PH A A A⊥ , deci P d∈ .

" "⊇ Fie ,P d P H∈ ≠ . Din ( )1 2 3PH A A A⊥ rezultă că 0j k j kPH A A PH A A⊥ ⇔ ⋅ =���� ������

.Dar H

este ortopunct, de unde rezultă că 0i j kHA A A⋅ =���� ������

.Avem 0i j k j k i j kPA A A PH A A HA A A⋅ = ⋅ + ⋅ =���� ������ ���� ������ ���� ������

.

Deci { } { }; , , 1,2,3i j kPA A A i j k⊥ =���� ������

. Prin urmare, P este ortopunct asociat 1 2 3A A A∆ .

10. 25: DeterminaŃi numerele naturale a şi b având produsul 21005 ştiind cǎ verificǎ

2010 2010 2010 2010 2010 2010

2a b a b a b

a b

a C b C b C a C C C+ =

⋅ + ⋅ ⋅ + ⋅ + .

Prof. Aurel Neicu, Liceul ” Ştefan cel Mare”, Hîrlău SoluŃie : Din ipotezǎ avem : 21005a b⋅ = . Notǎm cu 2010

ax C ∗= ∈ℕ şi cu 2010by C ∗= ∈ℕ .

Atunci : 2a b

a x b y b x a y x y+ =

⋅ + ⋅ ⋅ + ⋅ +( ) ( ) ( )( ) ( )( )2a x y bx ay b x y ax by ax by bx ay⇔ + + + + + = + +

⇔ ( ) ( ) ( )( ) ( )( ) ( ) ( ) 0ax ay bx ay ax by bx ay bx by ax by ax by bx ay+ + − + + + + + − + + =

echivalentǎ cu ( )[ ] ( )[ ] 0bx ay ax ay ax by ax by bx by bx ay/ /+ + − − + + + − − =/ / / / / / . Descompunem

Page 56: De ce să învăŃăm la matematică? Word - 2011 Revista Mici matematicieni...coordonate, de axe ortogonale sau nu. Ceea ce înseamnă…matematică. Reintrând într-un registru

Micii MATEMATICIENI

56

expresia gǎsitǎ: ( )( ) 0ay by bx ay ax by− + − − = ( ) ( ) ( ) 0y a b y a b x a b⇔ − − − − =

( ) ( )20y a b y x⇔ − − = . Deoarece 2010

by C= este numǎr natural nenul trebuie analizate

situaŃiile :

� Dacǎ ( )2 0a b a b− = ⇒ = şi cum 21005a b⋅ = 2 2 21005a b⇒ = = , de unde 1005a b= = .

� Dacǎ y x= atunci 2010 2010b aC C= . Din formula combinǎrilor complementare rezultǎ cǎ

2010a b+ = 2010a b⇒ = − . Înlocuind în 21005a b⋅ = obŃinem ( ) 22010 1005b b− ⋅ = , adicǎ

( )22 22010 1005 0 1005 0b b b− + = ⇔ − = . Rezultǎ cǎ 1005a b= = .

10. 26: CalculaŃi 2010 2010a b− ştiind cǎ( ) ( ) ( )2

1 1 1

loglog loga b

a b b aee ea ba b a b

+ =⋅⋅ ⋅

; , 0a b > ,

1b ≠ . Prof. Ioan Sǎcǎleanu, Liceul ” Ştefan cel Mare”, Hirlău

SoluŃie:Avem( ) ( ) ( )

2ln ln ln

lnln ln

a b

a b b a

e e e

a ba b a b+ =

⋅⋅ ⋅, de unde

2

ln ln ln ln ln lna b b a

a b

a b a b a b+ =

+ + +

1 10

ln ln ln ln ln ln ln ln

a b

a a b b a b b a a b a b ⇔ − + − = ⋅ + ⋅ + ⋅ + ⋅ +

. Calculǎm în paranteze:

( )( ) ( )( )ln ln ln ln ln ln ln ln

0ln ln ln ln ln ln ln ln

a a a b a a b b b a b b b a a b

a a b b a b b a a b a b

+ − − + − −+ =

⋅ + ⋅ + ⋅ + ⋅ +. Reducem şi scoatem factor:

( ) ln 1 10

ln ln ln ln ln ln

a b b

a b a a b b b a a b

− ⋅ ⋅ − = + + + ( ) ( )ln ln ln ln ln 0a b b b a a b a a b b⇔ − ⋅ + − − =

( ) ( ) ( )ln ln ln 0a b b a b b a b a⇔ − ⋅ ⋅ − − − ⋅ = i ( ) ( )2ln ln ln 0a b b b a⇔ − ⋅ ⋅ − = . Cum ln 0b ≠ ,

atunci ln lnb a a b= ⇔ = sau ( )2 0a b a b− = ⇔ = . Prin urmare 2010 2010 2010 2010 0a b a a− = − = .

Clasa a XI-a 11. 18: DeterminaŃi perechile de numere întregi ( ),a b ştiind cǎ verificǎ egalitatea

( )2

22 2 2lim

b n n b an a

nL n n a n n b e

⋅ + −+

→∞= + + − − + = .

Prof. Aurel Neicu, Liceul ” Ştefan cel Mare”, Hîrlău

SoluŃie : Limita bazei : ( )2 2

2 2

2lim limn n

n a bn n a n n b

n n a n n b

∞∞−∞ ∞

→∞ →∞

+ −+ + − − + = =

+ + + − +

2 2

22

lim 121 1

1 1n

a bn

n

a bn

n n n n

∞∞

→∞

− ⋅ +/ = = =

⋅ + + + − +/

iar2 1, 01

lim lim, 01

n n

bbn n bna bn an

→∞ →∞

=+ += ==

±∞ ≠+ + este

limita exponentului. Dacǎ 0b = atunci 2

1 22

1 1 1 0 2 02

b a b aL e b a a

− −= = ⇔ = ⇔ = ⇔ = ⇔ = .

Page 57: De ce să învăŃăm la matematică? Word - 2011 Revista Mici matematicieni...coordonate, de axe ortogonale sau nu. Ceea ce înseamnă…matematică. Reintrând într-un registru

Micii MATEMATICIENI

57

Dacǎ 0b ≠ suntem în cazul de nedeterminare 1∞ . Vom nota cu

( )2 2 1x n n a n n b= + + − − + + . Atunci se poate scrie astfel

( )2 2 2

2 2

2 1

1

n n a n n b n n bx

n n a n n b

+ + − − + + − + += ⇔

+ + + − + +

( )2

2 2

1 2

1

a b n n n bx

n n a n n b

− − + − − +=

+ + + − + +, de unde se

gǎseşte forma echivalentǎ cu 2

2 2

1 2

1

n ba b

n n n bx

n n a n n b

−− − +

+ − + =+ + + − + +

. ÎnmulŃind cu n obŃinem cǎ :

2 2 2

1 11 2

1 1 1 11 1 1 1

b

nnx n a bb a b

nn n n n n n n

= ⋅ − − + ⋅/

+ − + ⋅ + + + − + +/

. Trecând la limitǎ ,

obŃinem 1

lim2n

a bn x

→∞

− +⋅ = . Atunci ( )

1 11 1 lim2lim 1 n

bn a bbn x n bx n n ax n an

L x e e→∞

+ − ++ ⋅ ⋅ ⋅⋅ ⋅ ⋅ ++→∞

= + = = . Rezultǎ cǎ

( ) ( ) ( )( )2 22 0 2 4 4 2 2 4ab b a a b b b a b⇔ − + = ⇔ + − − = ⇔ + − + = . Rezolvând în ×ℤ ℤ

ecuaŃia obŃinem perechile ( ) ( ) ( ) ( ) ( ) ( ) ( ){ }, 1; 1 , 0;0 , 1;2 , 9; 3 , 8; 4 , 9; 6a b ∈ − − − − − − − .

11. 19: Legea de mişcare a unui mobil de masă 3 kg este dată de ( ) 25 2 1s t t t= − + . CalculaŃi

viteza,acceleraŃia şi energia cinetică a acestui mobil la momentul 3 sect = . Prof. Teodora Dana Pavel, Şcoala Deleni, Iaşi

SoluŃie : Viteza este ( ) ( ) ( )// 25 2 1 10 2v t s t t t t= = − + = − . La momentul 3t = avem

( )3 10 3 2 28v m s= ⋅ − = . AcceleraŃia este ( ) ( ) ( )// 10 2 10a t v t t= = − = , de unde

( ) 23 10a m s= . Energia cineticǎ se calculeazǎ cu 2 2

2 2 2 23 281176

2 2c

mvE kg s kg s

⋅= = = cǎci

masa este 3m kg= .

11. 20: Fie matricele ( ), nA B∈Μ ℂ cu proprietǎŃile : 7 6 6A A B B A= ⋅ + ⋅ şi 7nB I= . Sǎ se

demonstreze cǎ matricele 7A şi B comutǎ între ele. Prof. Mihai Crăciun, Colegiul NaŃional “M. Sadoveanu”, Paşcani

SoluŃie : A doua egalitate din enunŃ transformǎ prima egalitate astfel : 7 6 6nA A B B A O− ⋅ − ⋅ =

7 6 6 7nA A B B A B I⇔ − ⋅ − ⋅ + = ( ) ( )6 6 6 6

nA A B B A B I⇔ ⋅ − − ⋅ − = . Din distributivitatea la

stânga, gǎsim ( ) ( )6 6nA B A B I− ⋅ − = , adicǎ A B− este inversabilǎ . Prin urmare avem şi cǎ

( ) ( )6 6nA B A B I− ⋅ − = . Dezvoltând obŃinem 7 6 6 7

nA A B B A B I− ⋅ − ⋅ + = . Cum 7nB I= rezultǎ

cǎ 7 6 6A B A A B= ⋅ + ⋅ ( )1 . ÎnmulŃind ( )1 la stânga cu B avem : 7 7 6B A B A B A B⋅ = ⋅ + ⋅ ⋅ , de

unde 7 6B A A B A B⋅ = + ⋅ ⋅ ( )2 . ÎnmulŃind prima egalitate din enunŃ la drepta cu B avem : 7 7 6A B A B B A B⋅ = ⋅ + ⋅ ⋅ , de unde 7 6A B A B A B⋅ = + ⋅ ⋅ ( )3 . Deci 7 7A B B A⋅ = ⋅ q.e.d.

Page 58: De ce să învăŃăm la matematică? Word - 2011 Revista Mici matematicieni...coordonate, de axe ortogonale sau nu. Ceea ce înseamnă…matematică. Reintrând într-un registru

Micii MATEMATICIENI

58

Clasa a XII-a 12. 18: Fie " "⋅ o operaŃie internǎ asociativǎ definitǎ pe mulŃimea G. DemonstraŃi cǎ dacǎ sunt verificate proprietǎŃile: a) existǎ t G∈ astfel încât 2x t= pentru orice x G∈ ; b) pentru orice x G∈ , existǎ 0x G∈ astfel încât 0t x x⋅ =

atunci ( ),G ⋅ este grup comutativ.

Prof. Dr. Cristian Dinu , Colegiul NaŃional “Carol I”, Craiova SoluŃie : Din a) pentru orice x G∈ avem : 2 2t x x x x x x t⋅ = ⋅ = ⋅ = ⋅ , deci ,t x x t x G⋅ = ⋅ ∀ ∈ (1).

Pentru ( )

2a

x t G t t= ∈ ⇒ = (2). Atunci ( )( )( ) (2).

20 0 0

bb asoc

t x t t x t x t x x⋅ = ⋅ ⋅ = ⋅ = ⋅ = . Rezultǎ din ( )1 cǎ

,t x x t x x G⋅ = ⋅ = ∀ ∈ ( )3 , adicǎ t este element neutru. Deoarece x x x x t⋅ = ⋅ = rezultǎ cǎ orice

x G∈ are ca simetric pe x G∈ . Din x G∈ şi y G∈ lege

⇒ x y G⋅ ∈( )( )2

a

x y t⇒ ⋅ =

( )( )x y x y t⇒ ⋅ ⋅ = x y x y t⇒ ⋅ ⋅ ⋅ = ( )x x y x y y x t y⇒ ⋅ ⋅ ⋅ ⋅ ⋅ = ⋅ ⋅ .

2 2asoc

x y x y x y⇒ ⋅ ⋅ ⋅ = ⋅ ( )at y x t x y⇒ ⋅ ⋅ ⋅ = ⋅ , de unde

( )( ) ( )

asoc

t y x t x y⇒ ⋅ ⋅ ⋅ = ⋅( )3y x x y⇒ ⋅ = ⋅ , adicǎ legea este comutativǎ.

Prin urmare ( ),G ⋅ este grup comutativ întrucât sunt îndeplinite toate axiomele grupului

comutativ. 12. 19: Fie f:[0, ∞)→[0, ∞)o funcŃie continuă şi a parametru real strict pozitiv. Să se arate că

funcŃia ( )( )

( )

2x

ax

a

t f t dtF x

t f t dt

⋅=

∫∫

este crescătoare pe intervalul [a, ∞).

Prof. LaurenŃa Doca, Liceul „Ion Neculce”, Tg. Frumos SoluŃie : Cum funcŃia este continuă pe [0, ∞) atunci funcŃiile ( )2t f t şi ( )t f t⋅ sunt continue pe

[ ] [ ), 0,a x ⊂ ∞ , deci sunt integrabile şi admit primitive. Din formula Leibniz-Newton obŃinem :

( ) ( ) ( )2x

at f t dt H x H a= −∫ , ( ) ( )/ 2H x x f x= şi ( ) ( ) ( )

x

atf t dt G x G a= −∫ cu ( ) ( )/G x xf x=

⇒ ( )( ) ( ) ( ) ( ) ( ) ( ) ( ) ( )

( ) ( )

/ /

/2

H x H a G x G a H x H a G x G aF x

G x G a

− ⋅ − − − ⋅ − =−

deoarece

( ) ( ) ( )( ) ( )

H x H aF x

G x G a

−=

−. ObŃinem cǎ ( )

( ) ( ) ( ) ( )

( ) ( )

/ / 2

/2

x x

a aH x tf t dt G x t f t dt

F xG x G a

⋅ − ⋅=

∫ ∫ ,

echivalent cu ( )( ) ( ) ( ) ( )

( ) ( )

( ) ( ) ( )( )( ) ( )

22 2

/2 2

xx x

aa a

xf x xtf t t f t dtx f x tf t dt xf x t f t dtF x

G x G a G x G a

⋅ −⋅ − ⋅ = =− −

∫∫ ∫

Prin urmare , ( )( ) ( ) ( )

( ) ( )/

2

x

axf x tf t x t dt

F xG x G a

⋅ −=

∫. Deoarece t , ( )f t , x t− sunt pozitive, cǎci

[ ],t a x∈ rezultǎ cǎ ( )( ) 0tf t x t− ≥ , de unde ( )( ) 0x

atf t x t dt− ≥∫ şi cum x , ( )f x sunt pozitive

deducem cǎ ( )/ 0F x ≥ , adicǎ funcŃia F este crescătoare pe intervalul [a, ∞).

Page 59: De ce să învăŃăm la matematică? Word - 2011 Revista Mici matematicieni...coordonate, de axe ortogonale sau nu. Ceea ce înseamnă…matematică. Reintrând într-un registru

Micii MATEMATICIENI

59

12. 20: Sǎ se determine funcŃia de douǎ ori derivabilǎ ( ): 0,f → ∞ℝ ştiind cǎ verificǎ

/ / / / / / / / /

2f F

f f F f f f F f f f+ =

⋅ + ⋅ ⋅ + ⋅ + , unde F este o primitivǎ a funcŃiei f .

Prof. Ioan Sǎcǎleanu, Liceul ” Ştefan cel Mare”, Hirlău

SoluŃie : Avem / / / / / / / / / / / /

1 10

f F

f f F f f f f f F f f f

− + − = ⋅ + ⋅ + ⋅ + ⋅ +

, de unde

( )( ) ( )( )/ / / / / / / / / / / /

/ / / / / / / / / / / /0

f f f f f f F f F f F f f f F f

f f F f f f f f F f f f

⋅ + ⋅ − ⋅ − ⋅ ⋅ + ⋅ − ⋅ − ⋅+ =

⋅ + ⋅ + ⋅ + ⋅ +. Cum numitorul este pozitiv

rezultǎ cǎ ( )

( )( )( )

( )( )/ / / /

/ / / / / / / / / / / /0

f f F f f F

f f F f f f f f F f f f

⋅ − ⋅ −− =

⋅ + ⋅ + ⋅ + ⋅ +.

Scoatem factorul comun şi obŃinem: ( )/ /

/ / / / / / / / /

1 10

f f F

f f f f F f f f F f

⋅ − ⋅ − = + ⋅ + ⋅ ⋅ + ⋅

.

Eliminând numitorii obŃinem echivalent: ( ) ( )/ / / / / / / / 0f f F f f F f f f F f⋅ − ⋅ ⋅ + ⋅ − ⋅ − ⋅ = şi

grupând l: ( ) ( ) ( )/ / / / / 0f f F f f F f f F ⋅ − ⋅ − − − = ⇔ ( ) ( )2/ / / / / 0f f F f f⋅ − ⋅ − = . Rezultǎ

cǎ / / /f f= sau / / / / / /f F f F f f f f= ⇒ = ⇒ = ⇒ = . Avem ( ) ( )

//// /

/1 ln

ff x

f= ⇔ = .

Atunci existǎ k∈ℝ astfel încât ( )/ /ln x kf x k f x e += + ⇒ = . Atunci existǎ p∈ℝ :

( ) x kf x e p+= + . Cum ( )limx

f x p→−∞

= şi cum f este crescǎtoare rezultǎ cǎ

( ) ( )Im ; 0;f p= ∞ ⊂ ∞ 0p⇒ ≥ . Prin urmare toate funcŃiile ( ): 0,f → ∞ℝ , ( ) x kf x e p+= + cu

0p ≥ verificǎ condiŃiile din enunŃ.

12. 21: Sǎ se calculeze integrala ( )( )

( )2010

2010, 0,

x

x

e xdx x

x x e

⋅ −∈ ∞

⋅ +∫ .

Prof. Dr. Cristian Dinu , Colegiul NaŃional “Carol I”, Craiova

SoluŃie : Avem ( )( )

( )( )

2010 2009.

2010 2010 2010

20102010xxnot

x x

e x xe xI dx dx

x x e x x e

− ⋅−= =

+ ⋅ +∫ ∫ .

Considerǎm funcŃiile ( ). : 0,f g ∞ →ℝ , ( ) 2010f x x= şi ( ) 2010 xg x x e= + . Aplicând regulile de

derivare se obŃine cǎ ( ) ( ) ( ) ( ) ( )/ / 2010 20092010xf x g x g x f x e x x⋅ − ⋅ = ⋅ − ⋅ . Înlocuind gǎsim cǎ

( ) ( ) ( ) ( )( ) ( )

( )( )

( )( )

( ) ( ) ( )( )

/ / / /

ln ln lnf x g x g x f x g x f x g x

I dx dx dx g x f x k kf x g x g x f x f x

⋅ − ⋅= = − = − + = +

⋅∫ ∫ ∫

Deci ( )( ) 20102010

2010ln 1

x x

x

e x edx k

xx x e

⋅ − = + +

⋅ + ∫ .

Page 60: De ce să învăŃăm la matematică? Word - 2011 Revista Mici matematicieni...coordonate, de axe ortogonale sau nu. Ceea ce înseamnă…matematică. Reintrând într-un registru

Micii MATEMATICIENI

60

PROBLEME PROPUSE

MATEMATICA PITICĂ P. 67 : VerificaŃi, prin calcul egalitǎŃile:

12 42 21 24

12 63 21 36

× = ×

× = ×

12 84 21 48

13 62 31 26

× = ×

× = ×

24 63 42 36

24 84 42 48

× = ×

× = ×

26 93 62 39

36 84 63 48

× = ×

× = ×.

GǎsiŃi o altǎ modalitate de a verifica aceste egalitǎŃi, diferitǎ de cea prin calcul ! Inst. Eleonora-Diana Bucicǎ, Şcoala Dealul Viilor, Cǎtunele, Gorj

P. 68 : Câte picioare au cinci familii de iepuri a câte trei iepuraşi fiecare?

Inst. Gabriela–Liliana Onofrei, Şcoala “Petru Rareş”, Hîrlǎu

P. 69 : AflaŃi x din egalitatea: ( ){ }2011 10 : 4 :3 2 2011 2 2011 2011x− + − × × − = .

Prof. Nicolae Ivăşchescu, Colegiul NaŃional ” FraŃii Buzeşti”, Craiova P. 70 : Rica e gǎinuşa ce îşi hrǎneşte cei doi pui alternantiv, fǎrǎ a face discordie între ei. Din 4 in 4 metri ea gǎseşte câte un gândǎcel. Ştiind cǎ poteca este de 20 metri , aflaŃi câŃi gândǎcei primeşte Roro , puiul cel mic.

Înv. Maria-Tereza Rugină, Şcoala “Petru Rareş”, Hîrlău

P. 71 : Două bucăŃi de pânză au aceeaşi lungime. După ce s-au vândut 36 m din prima şi 50 m din a doua, în prima rămâne de 2 ori mai multă pânză decât în a doua. CâŃi metri de pânză a avut fiecare bucată de pânză.

Prof. Madlena Bulboacă, Liceul Pedagogic ,,Dimitrie łichindeal”, Arad

P. 72 : La un concurs de întrebări au participat două echipe, una cu 4 copii ṣi cealaltă cu 6 copii. Dacă fiecare din copii unei echipe propune câte o întrebare pentru fiecare din copii echipei adverse, câte întrebări vor fi în total?

Inst. Gabriela–Liliana Onofrei, Şcoala “Petru Rareş”, Hîrlǎu P. 73 : Dacǎ adunǎm numǎrul bondarilor cu dublul numǎrului fluturilor care au vizitat dimineaŃǎ trandafirii din grǎdinǎ obŃinem 61 de insecte, iar dacǎ adunǎm numǎrul fluturilor cu dublul numǎrului bondarilor, obŃinem 68 insecte. CâŃi bondari şi câŃi fluturi au fost în grǎdinǎ ?

Înv. Maria-Tereza Rugină, Şcoala “Petru Rareş”, Hîrlău

P. 74 : Un milionar lasă moştenire 1 000 000 euro pentru soŃia sa, cei doi fii şi două fiice. Conform testamentului, soŃia trebuie să primească de două ori mai mult decât un fiu, iar un fiu de două ori mai mult decât o fiică. Ce sumă primeşte fiecare?

Prof . Mirela Munteanu, Şcoala ,,Petru Rareş”, Hîrlău

P. 75 : Câtul a două numere este 6, dar dacă iau din numărul mai mic 5 şi adaug numărului mai mare, câtul lor devine 7. DeterminaŃi cele două numere.

Înv. Maria Ilie, Şcoala ,,Vasile Conta’’ Corp B, Iaşi

P. 76 : Câte pagini are o carte dacǎ la numerotarea ei, cifra 0 a fost folosită de 36 de ori ? Prof. Madlena Bulboacă, Liceul Pedagogic ,,Dimitrie łichindeal”, Arad

Page 61: De ce să învăŃăm la matematică? Word - 2011 Revista Mici matematicieni...coordonate, de axe ortogonale sau nu. Ceea ce înseamnă…matematică. Reintrând într-un registru

Micii MATEMATICIENI

61

P. 77 : Într-un număr natural de 3 cifre, din joacă, Radu inversează cifra zecilor cu cea a unităŃilor. El observă că dacă adună numărul astfel format cu numărul iniŃial, suma obŃinută este

un număr de 4 cifre de forma d192 .Care poate fi cifra unităŃilor sumei obŃinute? Dar numărul iniŃial ?

Inst. Corneliu Constantin Ilie, Şcoala ,,Vasile Conta’’ Corp B, Iaşi

P. 78 : Într-o cutie sunt numai bile de trei culori: roşii, galbene şi negre. Numai 27 dintre ele nu sunt negre, numai 39 nu sunt roşii. Câte bile de fiecare culoare sunt în cutie ştiind cǎ numǎrul bilelor roşii este de douǎ ori mai mic decât numǎrul bilelor negre ?

Prof. Turnea Mihaela, Lic.Teoretic “I.Neculce” Tg.Frumos P. 79 : AflaŃi numǎrul ştiind cǎ dacǎ mǎrim cu 2 dublul vecinilor sǎi obtinem numere pare vecine cu 36 . Înv. Maria-Tereza Rugină, Şcoala “Petru Rareş”, Hîrlău

P. 80 : ArǎtaŃi cǎ pentru orice 0a ≠ are loc egalitatea: ( ){ }: : 2 :1 1 1a a a a a a a+ + ⋅ − − = + .

Înv. Mihaela-Ramona Adam, Şcoala Miroslava, Iaşi P. 81 : MaimuŃele Lulu şi Momo au plecat la cules banane. După ce Lulu îşi dublează numărul de banane, Momo îi ia 8 banane. După trei astfel de ,,operaŃii”, Lulu nu mai are nicio banană. Câte banane a avut la început Lulu? ( Momo a promis că-i va da înapoi bananele lui Lulu)

Prof . Mirela Munteanu, Şcoala ,,Petru Rareş”, Hîrlău P. 82 : Suma a 3 numere naturale este 150. Dacă micşorez jumătatea primului număr cu 10, jumătatea celui de-al doilea cu 20 şi jumătatea celui de-al treilea cu 30 obŃin 3 numere consecutive pare, în ordine crescătoare. Care sunt numerele?

Înv. Maria Ilie, Şcoala ,,Vasile Conta’’ Corp B, Iaşi

P. 83 : Suma a trei numere este 2040. ÎmpǎrŃind primul numǎr la al doilea obŃinem câtul 2 şi 314, iar împǎrŃin pe al doilea la al treilea se obŃine câtul 1 şi restul 114. AflaŃi numerele.

Inst. Marian Ciuperceanu, Colegiul NaŃional “FraŃii Buzeşti”, Craiova

P. 84 : Andrei, Ioana şi Rareş au la un loc 2010 RON. Ei primesc de la bunicul lor câte o sumǎ de bani astfel: Ioana primeşte cu un RON mai mult ca Andrei, iar Andrei cu doi RON mai puŃin decât Rareş. În acest fel, sumele lor devin egale.

a) CâŃi RONI a avut iniŃial fiecare ? b) Ce sumǎ a primit fiecare de la bunicul lor dacǎ au în total 3513 RON ?

Prof. ConstanŃa Tudorache, Şcoala “Şt. Bârsǎnescu”, Iaşi P. 85 : Cei patru copii ai unei familii au împreunǎ 48 ani. DiferenŃele vârstelor copiilor celor mai apropiaŃi, în ordine crescǎtoare, sunt numere consecutive. Sǎ se afle vârstele copiilor, ştiind cǎ în familie existǎ fraŃi gemeni.

Prof. Ioan Sǎcǎleanu, Lic.” Ştefan cel Mare”, Hirlău

P. 86 : AflaŃi numerele de forma abc astfel încât ( ) ( ) ( )7 8 2009 2010a a c− ⋅ − ⋅ + = .

Prof. Nelu Tudorache, Liceul “Vasile Alexandri”, Iaşi P. 87 : Suma vârstelor celor patru copii ai unei familii este de 10 ani. AflaŃi vârstele copiilor ştiind cǎ în familie nu sunt gemeni.

Prof. Aurel Neicu, Lic.” Ştefan cel Mare”, Hîrlău

Page 62: De ce să învăŃăm la matematică? Word - 2011 Revista Mici matematicieni...coordonate, de axe ortogonale sau nu. Ceea ce înseamnă…matematică. Reintrând într-un registru

Micii MATEMATICIENI

62

P. 88 : Cifrele lui abcd verificǎ 2010 2011 11 11 2011 2010a b c d a b c d+ ⋅ + ⋅ + ⋅ = ⋅ + ⋅ + ⋅ + , sǎ

se arate cǎ abcd este numǎr palindromic (adicǎ este egal cu rǎsturnatul sǎu). Prof. Ioan Sǎcǎleanu, Lic.” Ştefan cel Mare”, Hirlău

MATEMATICA GIMNAZIALĂ Clasa a V-a 5.53 : Spunem cǎ douǎ numere de douǎ cifre sunt “gemene” dacǎ produsul cifrelor unuia este egal cu produsul cifrelor celuilalt. GǎsiŃi numerele gemene ale numǎrului 23.

Prof. Ioan Sǎcǎleanu, Lic.” Ştefan cel Mare”, Hirlău 5.54 : CalculaŃi ( ) ( ) ( )2 2b c c a a b+ ⋅ − ⋅ + ştiind cǎ 31a b c+ + = şi 2 3 4 105a b c+ + = .

Prof. Nicolae Ivăşchescu, Colegiul NaŃional ” FraŃii Buzeşti”, Craiova 5.55 : Vârsta lui Mihai este o cincime din vârsta lui Viorel. AflaŃi vârsta celor doi copii ştiind cǎ împreunǎ totalizeazǎ 13 ani.

Prof. Ioan RǎuŃu, Şcoala “Petru Rareş”, Hîrlǎu

5.56 : Suma a şapte numere naturale este egală cu 650. Este posibil ca produsul lor să se termine în 2009 ?

Prof Crăciun Alina, Liceul ,,Miron Costin”, Paşcani

5.57 : Se considerǎ egalitatea: 11 100abc a c= ⋅ + ⋅ . Sǎ se determine numǎrul abc ştiind cǎ dǎ câtul 74 la împǎrŃirea cu 3.

Prof. Ioan Sǎcǎleanu, Liceul ” Ştefan cel Mare”, Hirlău 5.58 : DescoperiŃi regula după care s-a format următorul şir de numere: 2; 35; 6; 32; 10; 29; 14; 26; 18; 23. Folosind aceeaşi regulă alcătuiŃi un şir din zece numere, astfel încât ultimul număr să fie 19, iar penultimul să fie 24.

Înv. Maria CreŃu, Şcoala „Petru Rareş”, Hîrlǎu 5.59 : Se considerǎ egalitatea : 9ab xy− = . DemonstraŃi cǎ a b= dacǎ şi numai dacǎ cifrele x , a şi y sunt consecutive.

Prof. Aurel Neicu, Liceul ” Ştefan cel Mare”, Hirlău 5.60 : Sǎ se scrie numǎrul 32010 2010n = − ca produsul de trei numere naturale consecutive.

Prof. Oana Alexe, Şcoala « Petru Rareş », Hîrlǎu

5.61 : ArǎtaŃi cǎ numǎrul 6 4 3 1 3 1 3 2 3 32 3 4 3 7 4 3n n n n n na + + + += ⋅ − ⋅ + ⋅ ⋅ se divide cu 19 pentru orice numǎr natural n .

Prof. Dr. Cristian Dinu , Colegiul NaŃional “Carol I”, Craiova

5.62 : Sǎ se determine numerele de forma xyzt ştiind cǎ împǎrŃind acest numǎr la yzt obŃinem câtul egal cu 1x + şi restul egal cu 2.x +

Prof. Turnea Mihaela, Liceul “Ion Neculce” Tg.Frumos

Page 63: De ce să învăŃăm la matematică? Word - 2011 Revista Mici matematicieni...coordonate, de axe ortogonale sau nu. Ceea ce înseamnă…matematică. Reintrând într-un registru

Micii MATEMATICIENI

63

5.63 : ScrieŃi numǎrul 2009 ca sumǎ de douǎ pǎtrate perfecte.

Prof. Victor Sǎceanu, Drobeta Turnu Severin

5.64 : Sǎ se determine restul împǎrŃirii numǎrului 201135 100− la numǎrul 175 . Prof. Petru Asaftei, Şcoala Normalǎ “Vasile Lupu”, Iaşi

5.65 : Moş Crǎciun şi-a propus sǎ împartǎ cadouri elevilor clasei a V-a de la Liceul din Hîrlǎu în 2 ore şi 13 minute în durate egale. Totuşi, activitatea sa a durat 2 ore şi 43 minute întrucât timpul acordat primilor cinci copii a fost de 15, 14, 13, 12 şi de 11 minute. a) Care este durata pe care şi-a propus-o Moşul cel mult aşteptat ? b) CâŃi copii sunt în clasa vizitatǎ ?

Prof. Ioan Sǎcǎleanu, Lic.” Ştefan cel Mare”, Hirlău 5.66 : ArătaŃi că numărul 2 3 2009 20101 3 3 3 ... 3 3N = + + + + + + nu este pătrat perfect, dar este divizibil cu un pǎtrat perfect.

Prof. Gabriela Popa , Şcoala nr. 43 „D.A.Sturza”, Iaşi 5.67 : Sǎ se arate cǎ nu existǎ numǎr de douǎ cifre pentru care suma cifrelor sale sǎ fie egalǎ cu suma cifrelor succesorului sǎu.

Prof. Ioan Sǎcǎleanu, Lic.” Ştefan cel Mare”, Hirlău 5.68 : Considerăm toate numerele naturale de trei cifre, scrise numai cu cifre pare, şi le calculăm produsul cifrelor. Adunăm toate aceste produse obŃinute. Care este rezultatul?

Prof. Mârza-Leşan Paraschiva şi Prof. Negrescu Alexandru, Şcoala “Elena Cuza”, Iaşi

5.69 : ArǎtaŃi cǎ existǎ o mulŃime A ce conŃine 2011 numere naturale consecutive şi care satisface simultan condiŃiile:

( )a Existǎ douǎ mulŃimi disjuncte B şi C astfel încât A B C= ∪ ;

( )b Orice element din B nu se poate scrie ca diferenŃa a douǎ elemente din B ;

( )c Orice element din C nu se poate scrie ca diferenŃa a douǎ elemente din C .

Prof. Petru Asaftei, Şcoala Normalǎ “Vasile Lupu”, Iaşi

5.70 : O mulŃime are suma celor 52 elemente ale sale egalǎ cu 2008. DeterminaŃi o astfel de mulŃime care conŃine cele mai multe numere impare.

Prof. Ioan Sǎcǎleanu, Lic.” Ştefan cel Mare”, Hirlău Clasa a VI-a 6. 43 : Un tren porneşte în traseu cu un număr de pasageri. Ştiind că în prima staŃie urcă un pasager, în a doua staŃie coboară doi, în a treia urcă trei, în a patra coboară patru pasageri şi aşa mai departe, să se determine cu câŃi pasageri a pornit trenul dacă în a 28 a staŃie coboară toŃi pasagerii.

Prof. Iuliana Blanariu, Liceul ” Ştefan cel Mare”, Hirlău 6. 44 : AflaŃi câŃi divizori are suma numerelor de forma abc ştiind cǎ 16 8 11a c b⋅ + ⋅ = ⋅ .

Prof. Victor Sǎceanu, Drobeta Turnu Severin

Page 64: De ce să învăŃăm la matematică? Word - 2011 Revista Mici matematicieni...coordonate, de axe ortogonale sau nu. Ceea ce înseamnă…matematică. Reintrând într-un registru

Micii MATEMATICIENI

64

6. 45 : Pe ruta Hîrlǎu-Iaşi, un automobil Opel Astra a consumat pânǎ la Târgu Frumos 25% din benzina pe care o avea în rezervor. Continuând, pânǎ la Podu Iloaie a mai consumat 20% din rest, rǎmânându-i pânǎ la Iaşi 5,6 litri de benzinǎ. Cu câŃi litri de benzinǎ a plecat la drum ?

Prof. Oana Alexe, Şcoala “Petru Rareş”, Hîrlǎu

6. 46 : Se dau unghiurile adiacente AOB∢ şi COB∢ . Bisectoarele lor formeazǎ un unghi cu mǎsura de 600. Ştiind cǎ BOC∢ are masura de douǎ ori mai mare decât AOB∢ , aflaŃi mǎsura unghiului format de bisectoarea unghiului AOB∢ cu semidreapta [OC .

Prof. Marcela Goşman, Şcoala” Ion Creangǎ”, Târgu Frumos 6. 47 : DeterminaŃi mulŃimile { }1 2; ;...; nA a a a= ştiind cǎ toate elementele sale sunt numere

naturale prime şi cǎ între ele are loc egalitǎŃile: 1 2 , 1, 1kk ka a k n+ − = = − .

Prof.Aurel Neicu, Liceul ” Ştefan cel Mare”, Hirlău

6. 48 : Sǎ se demonstreze cǎ numǎrul 1 2 3

1 1 1 1...

2 2 2 2abc+ + + + este subunitar, unde abc verificǎ

egalitatea: 1779abbc abb ab a− − − = . Prof. Popa Gabriela, Şcoala Nr. 43 “D.A.Sturza”, Iaşi

6. 49 : DemonstraŃi cǎ 2 2

2011

a b+∈ℕ dacǎ şi numai dacǎ

2 2

2011

a ba b⋅ − ⋅∈ℕ , unde 2011a b+ = şi

a∈ℕ şi b∈ℕ . Prof. Gheorghe Oancea, Liceul “Ştefan cel Mare”, Hîrlǎu

6. 50 : AflaŃi numerele naturale nenule x, y, z, t, ştiind că 157 1

1301

xy

zt

= ++

+

.

Prof. Mârza-Leşan Paraschiva, Prof. Negrescu Alexandru, Şcoala “Elena Cuza”, Iaşi 6. 51 : ArǎtaŃi cǎ nu existǎ numǎr întreg nenul x pentru care 2011 25 2 0x x x− ⋅ − = .

Prof. Ioan Sǎcǎleanu, Liceul ” Ştefan cel Mare”, Hirlău 6. 52 : Fie unghiurile adiacente ABC∢ şi BCD∢ astfel încât ( ) 120Om ABD =∢ . AflaŃi

măsurile celor două unghiuri ştiind că acestea sunt invers proporŃionale cu 0,01(9) şi 0,1. Prof. Bogdan Dorneanu, Şcoala “ Petru Rareş”, Hîrlǎu

6. 53 : DeterminaŃi m∈ℤ astfel încât ecuaŃia 1 ... 2000x x x m+ − + + − = să aibă soluŃie

unică. Prof. Turnea Mihaela, Liceul “Ion Neculce”, Târgu Frumos

6. 54 : AflaŃi numerele n ∗∈ℕ astfel încât ultima cifrǎ a numǎrului 1 2 3 ...2012 na + + + += sǎ fie 8 .

Prof. Petru Asaftei, Şcoala Normalǎ “Vasile Lupu”, Iaşi

Page 65: De ce să învăŃăm la matematică? Word - 2011 Revista Mici matematicieni...coordonate, de axe ortogonale sau nu. Ceea ce înseamnă…matematică. Reintrând într-un registru

Micii MATEMATICIENI

65

6. 55 : a) ConstruiŃi mediatoarea unui segment dat folosind doar un echer. b) Se dǎ unghiul AOB∢ de mǎsurǎ 0150 . Ştiind cǎ [ ] [ ]AO OB≡ , construiŃi cu ajutorul unui

echer un triunghi echilateral ABC△ . Prof. Nicolae Ivăşchescu, Colegiul NaŃional ” FraŃii Buzeşti”, Craiova

6. 56 : Fie P un punct în interiorul unghiului XOY cu măsura de 075 , iar M şi N simetricele

punctului P faŃă de [OX şi [OY . ArătaŃi 4

MP PNOP

+> .

Prof. Gabriela Popa, Şcoala „ Dimitrie A. Sturdza”, Iaşi Clasa a VII-a 7. 35 : Se considerǎ predicatul matematic: ( ), :P a b ” Suma dintre diferenŃa şi suma numerelor

a şi b este egalǎ cu suma dintre produsul şi câtul numerelor a şi b, unde a∈ℝ şi b ∗∈ℝ ”. a) GǎsiŃi a∈ℝ astfel încât propoziŃia ( ),P a b sǎ fie adevǎratǎ pentru orice b ∗∈ℝ .

b) DeterminaŃi b ∗∈ℝ astfel încât propoziŃia ( ),P a b sǎ fie adevǎratǎ pentru orice a∈ℝ .

Prof. Aurel Neicu, Lic.” Ştefan cel Mare”, Hîrlău

7. 36 : Un elev asociazǎ unor poligoane numere astfel: pǎtratului numǎrul 6, pentagonului numǎrul 10, hexagonului 15, iar heptagonului numǎrul 21. Ce numere credeŃi cǎ va asocia octogonului?

Prof. Nicolae Ivăşchescu, Colegiul NaŃional ”FraŃii Buzeşti”, Craiova

7. 37 : Numerele întregi , ,a b c sunt direct proporŃionale cu numerele întregi , 1, 2 1p p p+ + . DeterminaŃi , ,a b c ştiind cǎ suma pǎtratelor diferenŃelor acestora este 56.

Prof. Gheorghe Oancea, Lic.” Ştefan cel Mare”, Hirlău

7. 38 : Dacǎ cel mai mare divizor comun al numerelor 2ab , 4bc şi 8ca este 6, demonstraŃi cǎ

numǎrul 2 2 2a b c+ + este numǎr iraŃional. Prof. Dr. Cristian Dinu, Colegiul “Carol I”, Craiova

7. 39 : Bisectoarea unghiului A al paralelogramului ABCD intersectează latura [BC] în punctul M, iar bisectoarea unghiului AMC trece prin punctul D. Ştiind că ( ) 0 /36 30m MDC =∢ , aflaŃi

măsurile unghiurilor paralelogramului. Prof Crăciun Mihai, Colegiul National ,,M. Sadoveanu” Paşcani

7. 40 : DeterminaŃi numerele { }/ 1n ∗∈ℕ ştiind că 2

2

5 9 15 3...

1 2 2 3 3 4

n n

n n

− ++ + + + ∈

⋅ ⋅ ⋅ −ℕ .

Prof. Neculai Goşman, Şcoala “G. Ibrǎileanu”, Târgu Frumos, Iaşi

7. 41 : ArǎtaŃi cǎ numǎrul 2008 2011 2012

2009 2011 2010

⋅ +

⋅ + este iraŃional.

Prof. Ramona Darie, Liceul “Ştefan cel Mare”, Hîrlǎu

Page 66: De ce să învăŃăm la matematică? Word - 2011 Revista Mici matematicieni...coordonate, de axe ortogonale sau nu. Ceea ce înseamnă…matematică. Reintrând într-un registru

Micii MATEMATICIENI

66

7. 42 : DeterminaŃi a şi b ştiind cǎ 2011 20102011

2011

1

1

a a a

b b b

+ < < + şi cǎ mulŃimea

{ }; ;2011;2012a b are cardinalul egal cu 2.

Prof. Ioan Sǎcǎleanu, Liceul “Ştefan cel Mare”, Hîrlǎu 7. 43 : În trapezul ABCD cu AB DC� şi 2AB DC= ⋅ se considerǎ E mijlocul segmentului

[ ]AC , { }F DE AB= ∩ , { } BCADG ∩= şi { }H GE AB= ∩ .

Să se arate că a) F este mijlocul lui [ ]AB ; b) 4 DE BG⋅ = ; c) 3 HF AF⋅ = .

Prof. Liviu Barbu, Prof. P. Asaftei, Şcoala Normalǎ, Iaşi 7. 44 : Sǎ se arate cǎ triunghiul ale cǎrui laturi au lungimile a , b , c şi verificǎ relaŃia:

2 2 22 2 2 2 2 2 2 2 2 2

1 1 1 1 1 1 1 1 1 1: a b c

c a c a c b b c b a

− ⋅ + + + ⋅ − + ⋅ ⋅ = este dreptunghic.

Prof. Nicolae Ivăşchescu, Colegiul NaŃional ” FraŃii Buzeşti”, Craiova

7. 45 : Fie ABC∆ un triunghi oarecare cu ( ) 60Om BAC <∢ . În exteriorul triunghiului se

consideră triunghiurile echilaterale ABD şi ACE. Fie punctele S atfel încât AESD să fie paralelogram.

a) ArătaŃi că ( ) 60Om BSC =∢

b) ArătaŃi că ∆BSC este echilateral. Prof. Bogdan Dorneanu, Şcoala “ Petru Rareş”, Hîrlǎu

7. 46 : În pătratul ABCD , considerăm P mijlocul laturii [ ]AB şi [ ]M CP∈ cu DM CP⊥ .

Notăm { }DM BC N∩ = şi { }BM DC E∩ = . Să se arate că:

a) DME EMC≡∢ ∢ ;

b) EF BC� şi 3

DCEF = , unde { }AC DM F∩ = .

Prof. Mirela Marin, Şcoala “Al. VlahuŃǎ”, Iaşi

7. 47 : Fie ABC△ cu 0( ) 90m BAC =∢ , , ,AB c AC b BC a= = = şi [AM bisectoarea interioară a

unghiului A∢ cu [ )M BC∈ . CalculaŃi în funcŃie de a , b şi c :

a) distanŃa de la M la AB ; b) lungimea segmentului [ ]MO , O fiind mijlocul ipotenuzei.

Prof. Gabriela Popa, Şcoala „ Dimitrie A. Sturdza”, Iaşi

Clasa a VIII-a 8. 33 : Pentru ce valori ale lui x∈ℤ ,

2

222

16

33)1)(1(

1

221

1

1)(

x

xxxxx

x

x

x

xxE

+++−+⋅

+−

++

+−

= are inversul un numǎr întreg ?

Prof. Marcela Goşman, Şcoala “Ion Creangǎ”, Tg. Frumos

Page 67: De ce să învăŃăm la matematică? Word - 2011 Revista Mici matematicieni...coordonate, de axe ortogonale sau nu. Ceea ce înseamnă…matematică. Reintrând într-un registru

Micii MATEMATICIENI

67

8. 34 : Fie 8 2 15 8 2 15x = − − + . CalculaŃi 2010( 2 3)x + . Prof. Bogdan Dorneanu, Şcoala “ Petru Rareş”, Hîrlǎu

8. 35 : ArǎtaŃi cǎ x y= unde2

2 2 3 2 2 3

1 1 2

2 2 2 2 4

a b ay

a b a b a b ab a a b ab b

−= − − +

+ − + + + − −, iar

2 2 2

2 2 3 2 2 3 2

6 4 4 9 12 3 2: :

2

b a a b ab ab ax

a b a a b ab a a b

− + − −= − − + −

în condiŃiile de existenŃǎ necesar impuse.

Prof. Nicolae Ivăşchescu, Colegiul NaŃional ”FraŃii Buzeşti”, Craiova

8. 36 : Trei elevi au scris fiecare câte 60 de cuvinte. Analizând listele, s-au şters cuvintele care s-au găsit cel puŃin de două ori. După această operaŃie s-a constatat că un elev a rămas pe listă cu 40 de cuvinte, altul cu 48, iar ultimul mai are pe listă 43. Să se demonstreze că cel puŃin un cuvânt a fost scris de toŃi trei.

Prof. Alina Crăciun, Liceul ,,Miron Costin ”, Paşcani

8. 37 : AflaŃi pǎtratele perfecte ale unui numǎr prim, având forma 24 35,n n− ∈ℕ . Prof. Ioan RǎuŃu, Şcoala “Petru Rareş”, Hîrlǎu

8. 38 : Fie numerele p şi f , reprezentând partea întreagǎ, respectiv partea fracŃionarǎ a

numǎrului real x . Sǎ se determine numerele numerele , ,p f x ştiind cǎ 2010

2011

x fp

+ ⋅= .

Prof. Ioan Sǎcǎleanu, Liceul ”Ştefan cel Mare”, Hirlău

8. 39 : Fie numărul: 22009 2009 ... 2009nA = ⋅ ⋅ ⋅ , n ∗∈ℕ . Să se determine astfel încât numărul să aibă 946 de divizori.

Prof. Mihai Crăciun, Colegiul NaŃional “M. Sadoveanu”, Paşcani

8. 40 : ArǎtaŃi cǎ numǎrul 4 2 22011 2 2010 4021+ ⋅ − nu este un pǎtrat perfect. Prof. Mina Merlǎ, Şcoala łuŃora, Iaşi

8. 41 : RezolvaŃi în mulŃimea numerelor raŃionale ecuaŃia

3 2

2

2011 2011 2010 2010

12011 2011

x x x x

xx

+ − − +=

−+

Prof. Ioan Sǎcǎleanu, Liceul ”Ştefan cel Mare”, Hirlău

8. 42 : Existǎ numere reale distincte care sǎ verifice egalitatea 2 2

1 2

2

a

a b a a b+ =

+ +?

Prof. Aurel Neicu, Lic.” Ştefan cel Mare”, Hîrlău 8. 43 : a) ArǎtaŃi cǎ numǎrul 2011 este numǎr prim.

b) Fie mulŃimea infinitǎ de numere raŃionale 2013 2014 2015

, , ,...2 3 4

A =

. DemonstraŃi cǎ

mulŃimea A conŃine un singur numǎr întreg. Prof. Dr. Cristian Dinu , Colegiul NaŃional “Carol I”, Craiova

Page 68: De ce să învăŃăm la matematică? Word - 2011 Revista Mici matematicieni...coordonate, de axe ortogonale sau nu. Ceea ce înseamnă…matematică. Reintrând într-un registru

Micii MATEMATICIENI

68

MATEMATICA LICEALĂ Clasa a IX-a 9. 30 : DeterminaŃi funcŃia liniară :f →ℝ ℝ , ştiind că, pentru orice x număr real au loc

următoarele relaŃii: ( 1) 2 (2 ) 2 (1) 4 1

2 (2 ) ( 1) 3 (2) 3 4

f x f x f x

f x f x f x

+ − − = − +

− + + = + −.

Prof. Bogdan Dorneanu, Şcoala “ Petru Rareş”, Hîrlǎu 9. 31 : Să se rezolve, în mulŃimea numerelor reale ecuaŃia: 2 22 2 2 2 10 14 0x y xy x y+ − + − + = .

Prof. Ionica Marcovschi, Paşcani, Iaşi 9. 32 : DemonstraŃi cǎ, pentru orice x∈ℝ este adevǎratǎ urmǎtoarea propoziŃie matematicǎ:

( ) ( )( ) ( ) ( )1 2 3 ... 2010 2011 503x x x x x x+ + + + + + + + ≠ − .

Prof. Ioan Sǎcǎleanu, Liceul ” Ştefan cel Mare”, Hirlău

9. 33 : Fie /x∈ℝ ℚ . ArǎtaŃi cǎ numerele 3 2x x− şi 2x x+ nu pot fi simultan raŃionale. Prof. Ion Pǎtraşcu, Colegiul NaŃional “FraŃii Buzeşti”, Craiova

9. 34 : DemonstraŃi cǎ dacǎ 2

sin sin sin sin sin sin

a b

a a b b b a a b a b+ =

⋅ + ⋅ ⋅ + ⋅ + cu ( ), 0;1a b∈

atunci a b= . Prof. Ioan Sǎcǎleanu, Lic.” Ştefan cel Mare”, Hirlău

9. 35 : ArǎtaŃi cǎ: ( ) ( ) ( ) ( )2 x x y y z z y z x z x y x y z⋅ + + ≥ + + + + + , ∀ , ,x y z +∈ℝ .

Prof. Nicolae Ivǎşchescu, Colegiul NaŃional “FraŃii Buzeşti”, Craiova

9. 36 : Dacă , ,a b c +∈ℝ astfel încât 2 2 2a b ab c+ − = , arătaŃi că 2 2ac bc a b+ ≥ + .

Prof. Mârza-Leşan Paraschiva şi Negrescu Alexandru, Şcoala “Elena Cuza”, Iaşi

9. 37 : Fie fracŃia ( )12

10 6 2

1xF x

x x x

+=

− + astfel încât ( ) 7F x = cu 0x > . ArǎtaŃi cǎ

( ) 1F x F

x =

şi calculaŃi valoarea lui ( )2F x x⋅ .

Prof. Doca LaurenŃa, Liceul “Ion Neculce”, Târgu Frumos 9. 38 : Fie ABC△ cu AB c= , AC b= , BC a= . Se notează cu P intersecŃia dintre mediana BD şi bisectoarea (CE a unghiului BCA∢ cu ( )E AB∈ . Să se determine, în funcŃie de

lungimile laturilor triunghiului ABC, numerele reale x şi y astfel încât să aibă loc

relaŃia PA x PB y PC= ⋅ + ⋅���� ���� ����

. Prof. Turnea Mihaela, Liceul “Ion Neculce”, Târgu Frumos

Page 69: De ce să învăŃăm la matematică? Word - 2011 Revista Mici matematicieni...coordonate, de axe ortogonale sau nu. Ceea ce înseamnă…matematică. Reintrând într-un registru

Micii MATEMATICIENI

69

9. 39 : Dacǎ 1 1 1 1

x y z x y z+ + =

+ + arătaŃi că

2011 2011 2011 2011 2011 2011

1 1 1 1

x y z x y z+ + =

+ +.

Prof. Nicolae Ivăşchescu, Colegiul NaŃional ” FraŃii Buzeşti”, Craiova

9. 40 : ArǎtaŃi cǎ are loc inegalitatea: 1 2 3 2011

... 20112011 2010 2009 1

+ + + + > .

Prof. Ioan Sǎcǎleanu, Liceul ” Ştefan cel Mare”, Hirlău

9. 41 : Sǎ se rezolve în ℤ ecuaŃia cu parte întreagǎ: 2

2

3 3 3 2017

3 3 2

x x

x x x

− += + + +

.

Prof. Aurel Neicu, Liceul ” Ştefan cel Mare”, Hirlău

9. 42 : DeterminaŃi numǎrul real a din egalitatea: 0 0

20114

sin10 cos10

a aa

+− = .

Prof. Roxana Mihaela Ropotǎ, Liceul ” Ştefan cel Mare”, Hirlău

Clasa a X-a 10. 27: DeterminaŃi imaginea funcŃiei :f →ℝ ℝ , ( ) 2 3f x arctg x arctgx= − ⋅ .

Prof. Constantin Nastasǎ, Liceul “Ştefan cel Mare”, Hîrlǎu

10. 28: RezolvaŃi în ℂ ecuaŃia: 2011 2011 2011 2011

20111 2 3 2010... 2011

2 3 4 2011

x x x xx

+ + + ++ + + + = − .

Prof. LaurenŃa Doca, Liceul “Ion Neculce”, Târgu Frumos

10. 29: RezolvaŃi ecuaŃia exponenŃialǎ: 2

2 2

2

1

x x

x x x x

e e

xe x e e x e+ =

++ ⋅ + ⋅.

Prof. Ioan Sǎcǎleanu, Lic.” Ştefan cel Mare”, Hirlău

10. 30: Să se rezolve ecuaŃia logaritmicǎ: 33 2 3 6

1log log log

3 23

x xx

⋅ − =

.

Prof. Turnea Mihaela, Liceul “I.Neculce”, Târgu Frumos

Clasa a XI-a

11. 21: Dacă 2 1

0 2A

=

, arǎtaŃi cǎ ( )2det ... nA A A+ + + este un numǎr natural, pǎtrat perfect.

Prof. Turnea Mihaela, Liceul “Ion Neculce”, Târgu Frumos

11. 22: Matricea degeneratǎ ( )2A∈Μ ℝ verificǎ 2011 2019 200922012 2011A A A O− ⋅ + ⋅ = .

DeterminaŃi urma matricea A . Prof. Ioan Sǎcǎleanu, Liceul ” Ştefan cel Mare”, Hirlău

Page 70: De ce să învăŃăm la matematică? Word - 2011 Revista Mici matematicieni...coordonate, de axe ortogonale sau nu. Ceea ce înseamnă…matematică. Reintrând într-un registru

Micii MATEMATICIENI

70

11. 23: Fie ( )RMA 3∈ , .

233

101

34352

+

−−

+−

=

na

a

a

A Să se determine Rna ∈, astfel încât

.AAt = Prof. Madlena Bulboacă, Liceul Pedagogic ,,Dimitrie łichindeal”, Arad

11. 24: DemonstraŃi cǎ are loc inegalitatea: ln 2011 ln 2010 2011 2010

2011 2010 2 2010

− +≥

−.

Prof. Ioan Sǎcǎleanu, Liceul ” Ştefan cel Mare”, Hirlău

Clasa a XII-a

12. 22: Se consideră matricea

=

3̂0̂0̂

0̂2̂0̂

0̂0̂1̂

A şi

=

1̂0̂0̂

0̂1̂0̂

0̂0̂1̂

3I cu elemente din 7Z . Sǎ se

determine mulŃimea { }3 7( ) ( )C A X M Z A X X A= ∈ ⋅ = ⋅ şi matricea nA cu n∈ℕ .

Prof. Maria Both, Grup Şcolar ,,Iuliu Maniu”, Arad 12. 23: Fie [ ] Rbaf →,: o funcŃie continuă care admite derivată de ordin doi continuă şi

.0)()( == bfaf Să se demonstreze că / /( ) ( ) 0b

a

f x f x dx⋅ ≤∫

Prof. Maria Both, Grup Şcolar ,,Iuliu Maniu”, Arad 12. 24: DeterminaŃi funcŃiile : p pf →ℤ ℤ , p numǎr prim astfel încât ( ) ( ) ( )f x y f x f y+ = +

pentru orice , px y∈ℤ .

Prof. Aurel Neicu, Liceul ” Ştefan cel Mare”, Hirlău

12. 25: DeterminaŃi primitivele funcŃiei ( ): 1,f ∞ → ℝ , ( )( )

2010

22011ln

1

xf x x

x= ⋅

−.

Prof. Ioan Sǎcǎleanu, Liceul ” Ştefan cel Mare”, Hirlău

∗∗∗

Zâmbetul ŞtiinŃei

Karl Friedrich Gauss (1777-1855), princeps mathematicerum este considerat ca fiind cel de-al treilea geniu matematic (ca succesiune în timp) al omenirii, după Arhimede şi Newton. În matematică a fost un copil precoce. Următoarea întâmplare arată că la vârsta de 3 ani ştia să facă uşor calcule mintale. Într-o sâmbătă, tatăl său făcea socoteli pentru plăŃile lucrătorilor din subordine, fără să observe că micul Karl îi urmărea mersul operaŃiilor. Când

Page 71: De ce să învăŃăm la matematică? Word - 2011 Revista Mici matematicieni...coordonate, de axe ortogonale sau nu. Ceea ce înseamnă…matematică. Reintrând într-un registru

Micii MATEMATICIENI

71

şi-a încheiat adunările copilul îi spuse: "ai greşit tăticule; uite acolo la adunarea aceea trebuie pus ..."; şi când tatăl său a refăcut calculul a văzut că viitorul matematician avea dreptate, cifra indicată de el fiind cea bună. Tot aşa la vârsta de 10 ani uimea pe învăŃătorul său care propunea adunări de zeci de numere în progresie aritmetică. Gauss, fără să-i fi explicat cineva despre suma numerelor în progresie aritmetică, dădea rezultatul exact şi în timp scurt, aplicând intuitiv formula de însumare a acestor progresii aritmetice. Este de remarcat că după ce s-a stabilit la Göttingen nu a mai ieşit din acest oraş decât o singură dată, pentru a se duce la un congres international la care Alexander von Humboldt îl invitase spre a-l prezenta ca cel mai mare matematician al lumii din acel moment.

Culeasǎ de Neicu Mara

Concursul de creaŃie matematicǎ al revistei “Cea mai frumoasă problemă”

te invită să îŃi pui la încercare intuiŃia, perspicacitatea , creativitatea în conceperea de probleme originale la ediŃia a IV-a din februarie 2012. Acum ai ocazia să propui şi tu probleme, nu numai să rezolvi problemele propuse de alŃii. Aşadar, este o invitaŃie la efort, care va fi încununată de satisfacŃii pe măsură, pentru acei elevi care au înŃeles că matematica nu înseamnă numai probleme “încruntate” de calcul, mai mult sau mai puŃin asemănătoare, ci înseamnă creativitate, imaginaŃie, efort de gândire, toate grefate pe o solidă pregătire teoretică. La ediŃia a III-a din februarie 2010, din pǎcate nu au fost participanŃi.

∗∗∗ Concursul se adresează tuturor elevilor (clasele I-XII) . Elevii pot participa doar cu probleme originale. Problemele care nu sunt originale nu vor fi publicate sau nu vor participa la premiere. Fiecare problemă propusă trebuie sǎ fie însoŃită de rezolvarea completă. ExpediaŃi problemele folosind una din variantele:

• prin poştă , pe adresa Liceul Teoretic “ Ştefan cel Mare” , Hîrlău , str. Mihai Eminescu, nr. 5 , cu menŃiunea Pentru concursul “Cea mai frumoasǎ problemǎ”

• direct prof. Ioan Săcăleanu • prin e-mail, pe adresa : [email protected]

În luna februarie a fiecărui an vor fi stabiliŃi câştigătorii pentru fiecare clasă . Va fi premiat autorul celei mai originale . Alte informaŃii găsiŃi pe site-ul liceului http://hirlau.licee.edu.ro/

Page 72: De ce să învăŃăm la matematică? Word - 2011 Revista Mici matematicieni...coordonate, de axe ortogonale sau nu. Ceea ce înseamnă…matematică. Reintrând într-un registru

Micii MATEMATICIENI

72

Rubrica rezolvitorilor

Au dat soluŃii corecte la problemele propuse în nr. 4 următorii elevi:

Şcoala “Petru Rareş”, Hîrlǎu clasa a VIII a (prof.Ioan RǎuŃu): Poruşniuc Cosmin(12) IvǎnuŃǎ Roxana(10) Florea Mihaela (12) Coroeanu Ioana(10) şi Munteanu Vlad(10)

Liceul teoretic “Ştefan cel Mare”, Hîrlǎu clasa a V a (prof.Ioan Sǎcǎleanu): Puhǎ Alexandru(14) Ciobanu Laura(14) Pavǎl Maria(14) Bortaş IonuŃ(12) Bârzu Antonia(12) Cǎlin Constantin(12) Nastasǎ Georgiana(12) Loghin Andrei(12) Munteanu Claudia(10) Aursulesei Laura(10) Suruniuc Constantin(12) clasa a VI a (prof. Gheorghe Oancea): Agheorghiesei Tudor (10) Cozma Roxana(10) Cernescu Bogdan(10) Ciubuc Remus(10) Curca Iuliana(10) Ciobanu CodruŃa(10) Cojocaru Elisabeta(10) Pricop Adelin(10) Dolhǎscu Alexandru(10) clasa a VII a (prof.Ioan Sǎcǎleanu): Bobîrnǎ Costin(10) Murariu Maria(10) Nicu Andrei(10) Ruginǎ Rareş(10) Ciubotariu Raluca(10) Poruşniuc Iuliansa(10) Lǎcǎtuşu Alexandra(10) Petrei Dragoş(10) Sǎcǎleanu Emilian(10) Izmanǎ Petru10) Coman Daniela(10) clasa a VIII a (prof.Ioan Sǎcǎleanu): Cǎlinescu Ana(10) Buzilǎ Bianca(10) Porusniuc Andrei Nicolae(10) Mititelu Melissa Florina(10) Ifrim Rares Cristian(10) HuŃanu Mǎdǎlina(10) Scripcariu Gabriel(10) Ifrim Rareş(10) Sandu Adelina(10) Neicu Mara(10). clasa a IX a); (prof. Ramona Darie): Maftei Simona(8) IvǎnuŃǎ Andreea(8) Spiridon Alexandra(8); (prof.Ioan Sǎcǎleanu): Matei Madalina(10) Paval Simona(10) Pletan Denisa(10) Ceuca Razvan(10) Pintilii Alina(10) Spiridon Gabriela(14); (prof. Iuliana Blanariu): Gheorghinǎ Adriana(10) Ungureanu Elena(9). clasa a X a (prof. Ioan Sǎcǎleanu):Puhǎ Rǎzvan(12) Ponor Cosmina(12) Ciubotariu Evelina(12) (prof .Iuliana Blanariu): Chiriac Roxana(10) Amarghioalei Ana-Maria(10) Asoltanei Elena(10) Pinzaru Florin(10) Urcaciu Sinziiana(10) clasa a XI a (prof. Iuliana Blanaru): Dorcu LucreŃia Irina (10) Baiceanu C. Florin Constantin(10) (prof. Ramona Darie): Asoltanei Dumitru(10). clasa a XII a (prof.Ramona Darie): Cǎlin Mihaela(10) Apetrei Valentin (10) Manolie Victor (10) (prof.Ioan Sǎcǎleanu): AtîrgoviŃoaiei Anca(10), Atasiei Florentina(10), Oprea Otilia (10), Loghin Mǎdǎlina (10), GavriluŃǎ Alexandra(10).

În atenŃia elevilor !

Numele elevilor ce vor trimite redacŃiei soluŃii corecte la problemele din rubrica Probleme propuse vor fi menŃionate în Rubrica rezolvitorilor. Se va Ńine seama de următoarele reguli: 1. Pot trimite soluŃii la minim 10 probleme propuse în acest număr ; pe o foaie va fi redactată

soluŃia unei singure probleme; 2. Elevii din clasele III—VI au dreptul să trimită soluŃii la problemele propuse până la clasa lor

şi pentru orice clasă mai mare. Elevii din clasele VII-XII pot trimite soluŃii la problemele propuse pentru clasa lor, pentru orice clasă mai mare şi din două clase mai mici , imediat anterioare.

3. Vor fi menŃionate următorele date personale: numele şi prenumele, telefon-email, clasa, şcoala, localitatea şi profesorul clasei.

4. Plicul cu probleme rezolvate se va trimite prin poştă pe adresa RedacŃiei: Liceul “Ştefan cel Mare”, Hîrlău, str. Mihai Eminescu, nr. 5 sau va fi adus direct prof. Ioan Săcăleanu.